Blackboard EMQs Mixed Flashcards

1
Q

An unconscious 35-year-old man who has a capillary blood glucose of 1.5 mmol/L.

What would be the most appropriate next step?
A. Gastric lavage
B. Commence CPR
C. Intravenous dextrose
D. Inhaled anticholinergic
E. CT scan brain
F. IV antibiotics
G. Endotracheal intubation
H. Intravenous naloxone
I. Precordial thump
J. Intramuscular glucagon
K. Lumbar puncture
L. DC cardioversion
A

C. Intravenous dextrose

This patient is profoundly hypoglycaemic (symptoms of hypoglycaemia are present when glucose drops <3mmol/L). Patients with either long standing DM or on beta blockers may become unaware of hypoglycaemia and become profoundly hypoglycaemic before symptoms develop. In DM, hypoglycaemia is usually secondary to insulin or oral hypoglycaemics. Non DM causes of hypoglycaemia include insulinomas, alcohol, liver failure and Addison’s disease. Treatment is corrective and for someone this profoundly hypoglycaemic, IV dextrose is needed. Care should be taken when administering such high % dextrose IV due to the risk of skin necrosis if administered incorrectly (if the IV leaks). IM glucagon is only used if IV access cannot be established.

How well did you know this?
1
Not at all
2
3
4
5
Perfectly
2
Q

A 55-year-old man found collapsed at home who, on arrival at hospital, has no palpable pulse or recordable blood pressure.

What would be the most appropriate next step?
A. Gastric lavage
B. Commence CPR
C. Intravenous dextrose
D. Inhaled anticholinergic
E. CT scan brain
F. IV antibiotics
G. Endotracheal intubation
H. Intravenous naloxone
I. Precordial thump
J. Intramuscular glucagon
K. Lumbar puncture
L. DC cardioversion
A

B. Commence CPR

This patient has cardiac arrest and the first thing you need to do is commence CPR. It is thought to work by raising intrathoracic pressure and providing direct cardiac compression. 30 compressions (at 100/min) and 2 breaths for a total of 5 cycles (2 minutes), makes up one cycle of CPR. Compressions are the first priority, breaths may follow but compressions are paramount. Further management depends on whether the patient has a shockable rhythm (pulseless CT or VF) or non-shockable rhythm (PEA or asystole). Do you know when a precordial thump can be used?

How well did you know this?
1
Not at all
2
3
4
5
Perfectly
3
Q

An 18-year-old woman found unconscious at home. She has needle “track” marks in her arms, a respiratory rate of 10/min. and pinpoint pupils.

What would be the most appropriate next step?
A. Gastric lavage
B. Commence CPR
C. Intravenous dextrose
D. Inhaled anticholinergic
E. CT scan brain
F. IV antibiotics
G. Endotracheal intubation
H. Intravenous naloxone
I. Precordial thump
J. Intramuscular glucagon
K. Lumbar puncture
L. DC cardioversion
A

H. Intravenous naloxone

Opiate OD signs include CNS depression, miosis and apnoea. Finding small constricted pupils in someone who is unconscious is highly indicative. Naloxone is indicated both therapeutically and diagnostically. If there is a response, then it is diagnostic. Another diagnosis should be sought if the patient is unresponsive. IV is the preferred route of administration although naloxone can be given IM or SC if IV access cannot be established. Ventilatory support is key with 100% oxygen. You can check out Toxbase for a full database on poisons and treatments.

How well did you know this?
1
Not at all
2
3
4
5
Perfectly
4
Q

A 34-year-old woman who complained of a severe headache on waking and then collapsed.

What would be the most appropriate next step?
A. Gastric lavage
B. Commence CPR
C. Intravenous dextrose
D. Inhaled anticholinergic
E. CT scan brain
F. IV antibiotics
G. Endotracheal intubation
H. Intravenous naloxone
I. Precordial thump
J. Intramuscular glucagon
K. Lumbar puncture
L. DC cardioversion
A

E. CT scan brain

A CT head is indicated here in this possible SAH. This may show hyperdense areas in the basal cisterns, major fissures and sulci.

How well did you know this?
1
Not at all
2
3
4
5
Perfectly
5
Q

An 18-year-old known asthmatic with a respiratory rate of 50 and inaudible breath sounds on auscultation.

What would be the most appropriate next step?
A. Gastric lavage
B. Commence CPR
C. Intravenous dextrose
D. Inhaled anticholinergic
E. CT scan brain
F. IV antibiotics
G. Endotracheal intubation
H. Intravenous naloxone
I. Precordial thump
J. Intramuscular glucagon
K. Lumbar puncture
L. DC cardioversion
A

G. Endotracheal intubation

This is life threatening asthma that has not responded to therapy. There is cyanosis and respiratory acidosis despite the tachypnoea of 50/min. This patient is clearly too dyspnoeic to speak and oxygen saturation may be quite low. Pulsus paradoxus may also be observed. This patient should be intubated (mechanical ventilation is required) and transfered to ICU and given supplemental oxygen. IV corticosteroids should also be used and heliox therapy considered (helium-oxygen).

How well did you know this?
1
Not at all
2
3
4
5
Perfectly
6
Q

A 28-year-old female with recent onset of depression takes 50 capsules, 500 mg each of paracetamol tablets. In several days, the liver is most likely to show what?

A. Portal hypertension
B. Hepatocellular carcinoma
C. Cirrhosis
D. Portal chronic inflammation
E. Cholecystitis
F. Hepatitis C virus infection
G. Extensive necrosis
H. Mallory weiss tear of oesophagus
I. Hepatitis B virus infection
A

G. Extensive necrosis

Paracetamol OD can occur after a single large OD or repeated ODs. Often, the patient is asymptomatic at initial presentation but if untreated may cause liver injury over the 2-4 days after ingestion, including fulminant liver failure. Paracetamol is the most frequent intentional OD drug in this country. The risk of liver damage is increased after taking drugs which induce CYP 450. Inducers include St John’s wort, barbiturates, phenytoin, tetracycline, chronic alcohol use and carbamazepine. A serum paracetamol level is important to order as early as possible, but at the earliest 4 hours post-ingestion.Treatment if indicated is with N-acetylcysteine with the level based on a paracetamol treatment graph.

How well did you know this?
1
Not at all
2
3
4
5
Perfectly
7
Q

A 40-year-old male has a long history of chronic alcoholism. His liver is firm on palpation. An abdominal CT scan reveals that the liver has changes consistent with cirrhosis. He joins Alcoholics Anonymous and stops drinking. Despite his continued abstinence from alcohol, he remains at risk for development of which disease?

A. Portal hypertension
C. Cirrhosis
D. Portal chronic inflammation
E. Cholecystitis
F. Hepatitis C virus infection
G. Extensive necrosis
H. Mallory weiss tear of oesophagus
I. Hepatitis B virus infection
A

B. Hepatocellular carcinoma

Patients with cirrhosis, especially those with alcoholic liver disease, are at a high risk of developing HCC. Cirrhosis is irreversible so despite stopping drinking, he is still at risk of HCC (hepatoma). Patients with cirrhosis should be screened for HCC with serum AFP and USS at 6 month intervals.

How well did you know this?
1
Not at all
2
3
4
5
Perfectly
8
Q

A 40-year-old female, rather overweight, has developed right upper quadrant pain and fever. What is the most likely diagnosis?

A. Portal hypertension
B. Hepatocellular carcinoma
C. Cirrhosis
D. Portal chronic inflammation
E. Cholecystitis
F. Hepatitis C virus infection
G. Extensive necrosis
H. Mallory weiss tear of oesophagus
I. Hepatitis B virus infection
A

E. Cholecystitis

Cholecystitis is acute GB inflammation caused by an obstruction at the cystic duct. It occurs as a major complication of gallstones and classically presents with RUQ pain and fever. Gallstones in EMQs classically involves the Fs (Fat, Forty, Female, Fertile, Fair). USS is the definitive initial investigation. HIDA scanning and MRI may help if the diagnosis remains unclear. Treatment is with cholecystectomy.

How well did you know this?
1
Not at all
2
3
4
5
Perfectly
9
Q

A 58-year-old man, who smoked 30 cigarettes a day, presents with a 6-week history of cough, malaise, anorexia and weight loss. Past medical history includes hypertension for which he has taken lisinopril and bendrofluazide for 4 years.

What is the most likely diagnosis?
A. Carcinoma of bronchus
B. Postnasal drip
C. Foreign body
D. Angiotensin converting enzyme inhibitor
E. Oesophageal reflux
F. Sarcoidosis
G. COPD
H. Tuberculosis
I. Asthma
J. Bronchiectasis
A

A. Carcinoma of bronchus

The history of smoking and weight loss point to a bronchial carcinoma. Initial investigation is with a CXR. Diagnosis relies on pathological confirmation from a tissue sample, often obtained from bronchoscopy. First line treatment aims at surgical resection if possible. Small cell lung cancer is treated with chemotherapy and is associated with SIADH and ectopic ACTH. Non-small cell lung cancer is more often associated with clubbing. Squamous cell carcinoma is associated with PTHrp release and is treated with radiotherapy. Adenocarcinomas are usually located peripherally in the lung and are more common in non-smokers although most cases are still associated with smoking. The paraneoplastic syndromes may include Lambert-Eaton myasthenic syndrome.

How well did you know this?
1
Not at all
2
3
4
5
Perfectly
10
Q

A 45-year-old woman who smokes 25 cigarettes a day is reviewed in the diabetic clinic. She has had a dry cough for 2 months. She is on numerous tablets as her diabetes is complicated by microalbuminuria and hypertension. Her GP had given her a course of antibiotics 2 weeks previously.

What is the most likely diagnosis?
A. Carcinoma of bronchus
B. Postnasal drip
C. Foreign body
D. Angiotensin converting enzyme inhibitor
E. Oesophageal reflux
F. Sarcoidosis
G. COPD
H. Tuberculosis
I. Asthma
J. Bronchiectasis
A

D. Angiotensin converting enzyme inhibitor

A dry cough is a side effect of ACE inhibitors due to the build up of bradykinin which is normally degraded by ACE. ARB such as losartan will be indicated in this case. ARBs are insurmountable antagonists of AT1 receptors for angiotensin II, preventing its renal and vascular effects.

How well did you know this?
1
Not at all
2
3
4
5
Perfectly
11
Q

A 40-year-old Afro-Caribbean woman presents with bilateral parotid swelling, and painful nodules on the front of the shins. She has a dry cough and slight shortness of breath on exertion.

What is the most likely diagnosis?
A. Carcinoma of bronchus
B. Postnasal drip
C. Foreign body
D. Angiotensin converting enzyme inhibitor
E. Oesophageal reflux
F. Sarcoidosis
G. COPD
H. Tuberculosis
I. Asthma
J. Bronchiectasis
A

F. Sarcoidosis

Sarcoidosis is a chronic multisystem disease with an unknown aetiology. The painful (mauve) nodules are erythema nodosum. Lupus pernio is another typical skin manifestation of sarcoidosis presenting with indurated plaques with discoloration on the face. Parotid enlargement is a classic feature (involvement of exocrine glands). The dry cough and SOB on exertion indicate pulmonary involvement.
CXR will typically show bilateral hilar lymphadenopathy and CXR findings are used in the staging of disease. Additionally, serum calcium and ACE levels may be raised. A transbronchial biopsy is essential for diagnosis in most cases and shows the presence of non-caseating granulomas. Black people have a higher lifetime risk of sarcoidosis, as do those of Scandinavian origin. The mainstay of treatment for severe disease involves systemic corticosteroids.

How well did you know this?
1
Not at all
2
3
4
5
Perfectly
12
Q

An 18-year-old man presents with a night-time cough and shortness of breath while playing football. This has got progressively worse over the previous 2 months.

What is the most likely diagnosis?
A. Carcinoma of bronchus
B. Postnasal drip
C. Foreign body
D. Angiotensin converting enzyme inhibitor
E. Oesophageal reflux
F. Sarcoidosis
G. COPD
H. Tuberculosis
I. Asthma
J. Bronchiectasis
A

I. Asthma

SOB and the cough, which may wake the patient from sleep combined with the patient’s age and progessive course suggest asthma. Examination can show an expiratory wheeze but may be normal and treatment is step-wise based on BTS guidelines. It is worth noting that in severe exacerbations, the chest may be silent. Night symptoms occur in more severe asthma and symptoms can be exacerbated by exercise. Diagnosis is supported by PEFR variation of at least 20% over 3 days in a week over several weeks or an increase of at least 20% to treatment. Look up the BTS guidelines for more information.

How well did you know this?
1
Not at all
2
3
4
5
Perfectly
13
Q

A 30-year-old man, a lifelong non-smoker, presents with a history of at least 6 months of purulent sputum. He has had regular chest infections since an attack of measles at the age of 14.

What is the most likely diagnosis?
A. Carcinoma of bronchus
B. Postnasal drip
C. Foreign body
D. Angiotensin converting enzyme inhibitor
E. Oesophageal reflux
F. Sarcoidosis
G. COPD
H. Tuberculosis
I. Asthma
J. Bronchiectasis
A

J. Bronchiectasis

Bronchiectasis is permanent bronchi dilatation due to bronchial wall damage and loss of elasticity. It is often as a consequence of recurrent/severe infections and most present with chronic productive mucopurulent cough. The most common identifiable cause is CF. Chest CT is the diagnostic test. Diagnosis is aided by sputum analysis. Have a think about what you would expect to hear on ascultation of the chest.

How well did you know this?
1
Not at all
2
3
4
5
Perfectly
14
Q

A 19-year-old student complaining of amenorrhoea for 9 months. Weight loss, generalised weakness and depression. She has started a vegan diet a year ago. Her BMI is 16.

What is the most likely diagnosis?
A. Depression
B. Hyperthyroidism
C. Malignancy
D. Cardiac failure
E. Infestation with helminths
F. Tuberculosis
G. HIV
H. Diabetes mellitus
I. Liver failure
J. Addison's disease
K. Anorexia nervosa
L. Renal failure
M. Malabsorption
A

K. Anorexia nervosa

Anorexia nervosa often occurs in women which is a key risk factor. Puberty/adolescence and obsessive personality traits are additional risks. There is a higher incidence reported in western cultures (presumably skinny models on TV, adverts etc…) and studies on immigrants moving to a western culture exhibited a higher incidence. There is also postulated to be a genetic contribution from twin studies.Those who in higher socioeconomic classes are more affected. BMI is usually <17.5 and the patient often has a fear of weight gain and will refuse intervention to gain weight. The patient often fails to acknowledge how thin they are and can indicate how some parts of her body are ‘big’. Amenorrhoea is also a common complaint. Often anorexics eat vegetarian or vegan diets and you should inquire about the patient’s eating habits.There are two subtypes of AN. Restrictive where the patient will diet and exercise or the bingeing/purging types where there is also calorie restriction but ocassional binges and purging afterwards. There is also a tendency to laxative, diuretic and enema abuse.

How well did you know this?
1
Not at all
2
3
4
5
Perfectly
15
Q

A 17-year-old man returning from holiday in Africa. He presents with abdominal pain, weight loss, pruritis and a wheeze. Blood film showed eosinophilia.

What is the most likely diagnosis?
A. Depression
B. Hyperthyroidism
C. Malignancy
D. Cardiac failure
E. Infestation with helminths
F. Tuberculosis
G. HIV
H. Diabetes mellitus
I. Liver failure
J. Addison's disease
K. Anorexia nervosa
L. Renal failure
M. Malabsorption
A

E. Infestation with helminths

The only option on the list that would give eosinophilia is infection with helminths. There is a risk factor here, having returned from Africa where the sanitation, hygiene and agricultural practices may leave much to be desired. Testing for stool ova and parasites will be needed to see exactly which helminth is causing this infection, although this is not very sensitive for strongyloides larvae. IgG serology can be used with >95% sensitivity if stool samples are negative in the case of strongyloides. This could be strongyloides or ascariasis or a rarer organism. Treatment will be with antihelminths depending on the organism. Ivermectin for strongyloides is the drug of choice. Other antihelmintic agents include albendazole, mebendazole and pyrantel pamoate.

How well did you know this?
1
Not at all
2
3
4
5
Perfectly
16
Q

A 70-year-old man with a history of 10kg weight loss over the previous 3 months. More recently, he developed acute lower back pain. He presents to A&E with coughing and sputum. Chest x-ray shows left lower lobe pneumonia.

What is the most likely diagnosis?
A. Depression
B. Hyperthyroidism
C. Malignancy
D. Cardiac failure
E. Infestation with helminths
F. Tuberculosis
G. HIV
H. Diabetes mellitus
I. Liver failure
J. Addison's disease
K. Anorexia nervosa
L. Renal failure
M. Malabsorption
A

C. Malignancy

This person likely has lung cancer which has resulted in the significant weight loss of 10kg. This is post-obstructive pneymonia which is common in lung cancer patients and is caused, most of the time, by a large and centrally obstructing tumour. It is essential to relieve this obstruction in this case and many techniques can be tried. There is also bone pain here in the lower spinal column which is due to metastases. The prognosis here is not good.

How well did you know this?
1
Not at all
2
3
4
5
Perfectly
17
Q

A 25-year-old woman with fatigue and weight loss. She gives a history of frequent loose stools with abdominal pain. Full blood count revealed iron deficiency anaemia.

What is the most likely diagnosis?
A. Depression
B. Hyperthyroidism
C. Malignancy
D. Cardiac failure
E. Infestation with helminths
F. Tuberculosis
G. HIV
H. Diabetes mellitus
I. Liver failure
J. Addison's disease
K. Anorexia nervosa
L. Renal failure
M. Malabsorption
A

M. Malabsorption

There is frequent loose stools here and abdominal pain. Combined with the IDA, this points to malabsorption. This could well be a presentation of coeliac disease – IDA is one of the most common clinical presentations and abdominal pain and diarrhoea are common. Coeliac disease is a systemic autoimmune condition triggered by dietary gluten peptides found in grains. It is a relatively common condition. The only treatment is a strict gluten-free diet for life.

How well did you know this?
1
Not at all
2
3
4
5
Perfectly
18
Q

A 40-year-old African refugee has noticed recent weight loss. Although he attributed this to stress you are concerned when you detect generalised lymphadenopathy. Blood count shows neutropenia and thrombocytopenia.

What is the most likely diagnosis?
A. Depression
B. Hyperthyroidism
C. Malignancy
D. Cardiac failure
E. Infestation with helminths
F. Tuberculosis
G. HIV
H. Diabetes mellitus
I. Liver failure
J. Addison's disease
K. Anorexia nervosa
L. Renal failure
M. Malabsorption
A

G. HIV

HIV is a retrovirus and there are two types, HIV 1 which is the main virus responsible and HIV 2 which is restricted to parts of West Africa. Weight loss is common in HIV and if more than 10% body weight is lost of BMI reduces to 18.5, this is an indication of more severe immunocompromise. Weight loss in HIV may result from malnutrition, co-existent TB infection or HIV wasting syndrome, the latter being an AIDS defining illness. Generalised lymphadenopathy is also common and is characterised by the painless enlargement of 2 more more non-contiguous sites of >1cm for >3 months. Neutropenia is also seen due to CD4 deficiency and thrombocytopenia may also be seen along with an anaemic picture.

There are WHO (stage 1-4) and CDC criteria used in clinical staging. This patient needs to have a CD4 count, HBV and HCV screen, VDRL (syphilis), tuberculin skin test (TB) and CXR. HIV viral load will also be assessed. Prophylaxis and immunisations should be considered against infections such as hepatitis, influenza, PCP and TB. When to initiate HAART depends on the clinical stage, CD4 and co-morbidities. This patient will need to be started on HAART. Classes of antiretrovirals include NRTIs, NNRTIs, protease inhibitors, fusion inhibitors and integrase inhibitors.

How well did you know this?
1
Not at all
2
3
4
5
Perfectly
19
Q

A 25-year-old woman returning from Australia presents with acutely painful left calf. Ultrasound confirms deep vein thrombosis extending above the popliteal veins. She has recently missed a period.

What is the most appropriate management?
A. Embolectomy
B. Subcutaneous low molecular weight heparin
C. Reassure and discharge
D. Anti-embolism stocking
E. Fondaparinux (FXa inhibitor)
F. Intravenous heparin 
G. Start warfarin therapy
H. Subcutaneous low molecular weight heparin
I. Check INR and continue warfarin
J. Vena cava filter
K. Observation in hospital
A

B. Subcutaneous low molecular weight heparin

Women developing a DVT during pregnancy can be treated with heparin or LMWH. However, LMWH is preferred due to more dependable pharmacokinetics. The agents you will commonly hear include enoxaparin, dalteparin and tinzaparin.
You may see patients treated with unfractionated heparin instead of LMWH - this is done for patients at a higher bleeding risk because unfractionated heparin is more easily reversible with protamine.

How well did you know this?
1
Not at all
2
3
4
5
Perfectly
20
Q

A 30-year-old man developed acute pain in the right calf after a game of squash. He has marked calf tenderness but no swelling. Doppler ultrasound is negative.

What is the most appropriate management?
A. Embolectomy
B. Subcutaneous low molecular weight heparin
C. Reassure and discharge
D. Anti-embolism stocking
E. Fondaparinux (FXa inhibitor)
F. Intravenous heparin 
G. Start warfarin therapy
H. Subcutaneous low molecular weight heparin
I. Check INR and continue warfarin
J. Vena cava filter
K. Observation in hospital
A

C. Reassure and discharge

This is likely to be a musculoskeletal injury in an otherwise fit, young man. Eccentric contraction causes more frequent injury. Inadequate warm-up before exercise can predispose to muscle injuries.

How well did you know this?
1
Not at all
2
3
4
5
Perfectly
21
Q

A 50-year-old woman taking non-steroidal anti-inflammatory drugs for arthritis presented with a history of sudden onset pain behind her right knee leading to pain down the calf. Ultrasound confirms a Baker’s cyst.

What is the most appropriate management?
A. Embolectomy
B. Subcutaneous low molecular weight heparin
C. Reassure and discharge
D. Anti-embolism stocking
E. Fondaparinux (FXa inhibitor)
F. Intravenous heparin 
G. Start warfarin therapy
H. Subcutaneous low molecular weight heparin
I. Check INR and continue warfarin
J. Vena cava filter
K. Observation in hospital
A

C. Reassure and discharge

A popliteal cyst is an accumulation of synovial fluid which in this case has resulted from this woman’s arthritis. This is an accumulation of synovial fluid behind the knee, usually in response to injury or inflammation. It will self-resolve but the underlying cause should be addressed i.e. arthritis. First line treatment for grade 1 or 2 injuries is with RICE: rest, ice, compression and elevation followed by gentle mobilisation. Adjunctive analgesia can be offered with paracetamol. Treatment is conservative, particularly if asymptomatic. Surgery is only indicated in those with extensive symptoms where conservative and percutaneous treatments have failed. Corticosteroid injections (intra-articular) can also be considered.

How well did you know this?
1
Not at all
2
3
4
5
Perfectly
22
Q

You are asked to see a patient with acute chest pain 5 days after total hip replacement. BP 120/80, HR 93. A PE is confirmed. The patient has a previous history of heparin-induced thrombocytopenia.

What is the most appropriate management?
A. Embolectomy
B. Subcutaneous low molecular weight heparin
C. Reassure and discharge
D. Anti-embolism stocking
E. Fondaparinux (FXa inhibitor)
F. Intravenous heparin 
G. Start warfarin therapy
H. Subcutaneous low molecular weight heparin
I. Check INR and continue warfarin
J. Vena cava filter
K. Observation in hospital
A

E. Fondaparinux (FXa inhibitor)

A factor Xa antagonist is preferred if the patient has or has had heparin-induced thrombocytopenia. If the patient has a low BP then systemic thrombolysis would be indicated to prevent possible cardiac arrest.

How well did you know this?
1
Not at all
2
3
4
5
Perfectly
23
Q

A woman who is on warfarin for a confirmed right calf DVT develops increasing pain and swelling of that leg. This is the third time this has happened. Repeat imaging shows thrombus limited to the calf.

What is the most appropriate management?
A. Embolectomy
B. Subcutaneous low molecular weight heparin
C. Reassure and discharge
D. Anti-embolism stocking
E. Fondaparinux (FXa inhibitor)
F. Intravenous heparin 
G. Start warfarin therapy
H. Subcutaneous low molecular weight heparin
I. Check INR and continue warfarin
J. Vena cava filter
K. Observation in hospital
A

H. Subcutaneous low molecular weight heparin

Patients with recurrent thromboembolism despite on warfarin should be given heparin or LMWH. LMWH is again the primary option for reasons discussed. They should be given for at least 5 days until INR is between 2 and 3 (the target INR). Fondaparinux would be used instead if there was a high chance of delayed HIT. Warfarin is continued and efforts must be made to work out how this patient has developed a DVT despite on warfarin therapy. There may be subtherapeutic dosing, the presence of a malignancy or antiphospholipid syndrome. If there is documented thrombosis progression despite adequate anticoagulation, without HIT and other causes are excluded, an IVC filtre may be indicated but evidence of its efficacy have been debated by recent studies.

How well did you know this?
1
Not at all
2
3
4
5
Perfectly
24
Q

A 40 year old lady with multiple painful lumps in her breast, which are painful & tender pre-menstrually.

What is the most likely diagnosis?
A. Duct Ectasia
B. Lipoma
C. Sebaceous cyst
D. Carcinoma of the breast
E. Gynaecomastia
F. Fibroadenoma
G. Breast Abscess
I. Breast Cyst
A

H. Fibroadenosis

Fibrocystic breasts are characterised by ‘lumpy’ breasts associated with pain which fluctuates with the menstrual cycle (it is worse during the luteal phase of menses). Risk factors include obesity, nulliparity, HRT and late onset menopause and first childbirth. It is a diagnosis of exclusion, and is considered to be an exaggerated physiological phenomenon rather than a disease (54% of clinically normal breasts are found on autopsy to have fibrocystic changes). Symptoms typically arise between the 3rd and 4th decases of life. There may also be a nipple discharge, which can be suspicious if bloody or profuse etc and may indicate the presence of an intraductal papilloma, cancer, or duct ectasia. Cysts can be aspirated if symptomatic (asymptomatic or small ones do not require intervention). If the aspirate is straw coloured and completely aspirated, there is no need for cytology, but if the aspirate is bloody, cytology or biopsy is needed to exclude cancer. There is improvement of mastalgia and cysts at menopause and until then it runs a chronic relapsing course.

How well did you know this?
1
Not at all
2
3
4
5
Perfectly
25
Q

A 25 year old lady with a discrete, non-tender, mobile lump in one breast.

What is the most likely diagnosis?
A. Duct Ectasia
B. Lipoma
C. Sebaceous cyst
D. Carcinoma of the breast
E. Gynaecomastia
F. Fibroadenoma
G. Breast Abscess
H. Fibroadenosis
I. Breast Cyst
A

F. Fibroadenoma

This is a fibroadenoma which tends to be asymptomatic and found incidentally, typically in a patient <40 years old. It is a mobile mass (sometimes called breast mice), firm, painless and can also be described as smooth and rubbery. It is benign with epithelial and stromal elements.

How well did you know this?
1
Not at all
2
3
4
5
Perfectly
26
Q

A 35 year old lady is generally unwell with a tachycardia & a fever. A segment of the right breast is painful, tender, red & warm

What is the most likely diagnosis?
A. Duct Ectasia
B. Lipoma
C. Sebaceous cyst
D. Carcinoma of the breast
E. Gynaecomastia
F. Fibroadenoma
G. Breast Abscess
H. Fibroadenosis
I. Breast Cyst
A

G. Breast Abscess

Breast abscess presents with mastalgia and fever. Breast infection typically affects women who are lactating and the most commonly implicated pathogen is staphylococcus aureus. The painful, hard and red lump indicates the development of an abscess. Antibiotic therapy is indicated with surgical intervention such as aspiration and drainage with possible duct excision. Prompt management of mastitis when it presents will usually lead to a good timely resolution and prevent the development of complications such as an abscess. An USS can help to identify the underlying abscess which usually forms a hypoechoic lesion. Needle aspiration can be used both therapeutically and diagnostically and can be guided by ultrasound.

How well did you know this?
1
Not at all
2
3
4
5
Perfectly
27
Q

A 14 year old boy with bilateral breast enlargement.

What is the most likely diagnosis?
A. Duct Ectasia
B. Lipoma
C. Sebaceous cyst
D. Carcinoma of the breast
E. Gynaecomastia
F. Fibroadenoma
G. Breast Abscess
H. Fibroadenosis
I. Breast Cyst
A

E. Gynaecomastia

This is a boy who has enlarged breasts. Normal to see gynaecomastia in puberty. Other causes include liver disease and as a side effect of drugs such as digoxin, spironolactone and cimetidine.

How well did you know this?
1
Not at all
2
3
4
5
Perfectly
28
Q

A 40 year old lady with a green nipple discharge & tender lumpiness beneath the areola.

What is the most likely diagnosis?
A. Duct Ectasia
B. Lipoma
C. Sebaceous cyst
D. Carcinoma of the breast
E. Gynaecomastia
F. Fibroadenoma
G. Breast Abscess
H. Fibroadenosis
I. Breast Cyst
A

A. Duct Ectasia

Duct ectasia happens because the lactiferous duct gets blocked. Ectasia means widening. It can mimic breast cancer as the discharge can be bloody sometimes and signs can include nipple inversion. The green nipple discharge is typical in EMQs. It is a self limiting condition.

How well did you know this?
1
Not at all
2
3
4
5
Perfectly
29
Q

A 60 year old man with a history of occupational exposure in building & demolition industry presents with shortness of breath. On examination there are signs of a pleural effusion and the patient is clubbed.

What would be the most appropriate investigation?
A. Colonoscopy
B. Bronchoscopy
C. Sputum culture
D. Abdominal ultrasound
E. Echocardiogram
F. Stool culture
G. Lung funtion tests
H. Chest x-ray
A

H. Chest x-ray

Idiopathic pulmonary fibrosis (previously known as Cryptogenic fibrosing alveolitis) progresses over several years and is characterised by pulmonary scar tissue formation and dyspnoea. Patients complain of a non-productive cough and typically reproducible and predictable SOB on exertion. This man’s work means he comes into contact with small organic or inorganic dust particles which is thought to be implicated in the cascade of events leading to IPF. Another risk factor is cigarette smoking which significantly increases the risk of IPF. The mean age of diagnosis is 60-70. CXR here will show reticular opacities. A high resolution CT scan can also be done if it was an option on this list.

How well did you know this?
1
Not at all
2
3
4
5
Perfectly
30
Q

A 19 year old woman with a past history of cardiac surgery in infancy presents with symptoms of decreasing exercise tolerance. On examination there is cyanosis & clubbing.

What would be the most appropriate investigation?
A. Colonoscopy
B. Bronchoscopy
C. Sputum culture
D. Abdominal ultrasound
E. Echocardiogram
F. Stool culture
G. Lung funtion tests
H. Chest x-ray
A

E. Echocardiogram

The only cardiac investigation here is an echocardiogram. Cardiovascular causes of clubbing include cyanotic congenital heart disease, infective endocarditis and atrial myxoma. This could well be Eisenmenger’s syndrome caused by shunt reversal.

How well did you know this?
1
Not at all
2
3
4
5
Perfectly
31
Q

A 35 year old woman with history of recurrent lower abdominal pain, bloody diarrhoea & passing mucus PR. On examination there is lower abdominal tenderness & clubbing.

What would be the most appropriate investigation?
A. Colonoscopy
B. Bronchoscopy
C. Sputum culture
D. Abdominal ultrasound
E. Echocardiogram
F. Stool culture
G. Lung funtion tests
H. Chest x-ray
A

A. Colonoscopy

This woman has inflammatory bowel disease (which is a cause of clubbing), which by the history is more likely to be UC than CD, whereby the mainstay of treatment is with 5-ASA. A colonoscopy is required to assess the extent of disease and for a definitive diagnosis. Biopsy in CD will show transmural granulomatous inflammation. CD can affect the whole GIT but favours the TI and proximal colon and is macroscopically characterised by skip lesions. UC on the other hand is characterised by the presence of crypt abscesses, which is pathognomic. CD risk is increased 3-4 fold by smoking whereas smoking seems protective in UC. The mainstay of treatment in CD is with steroids and azathioprine to revent relapses and for those suffering side effects of steroid treatment. TNF-alpha inhibitors also have a role. Surgery in CD is only indicated in a small number of patients who bleed, for bowel perforation and cases of complete obstruction. The aim is to rest distal disease by temporarily diverting faecal flow.While a stool culture may be useful in ruling out infectious gastroenteritis it will not provide the diagnosis here.

How well did you know this?
1
Not at all
2
3
4
5
Perfectly
32
Q

A 50 year old woman who is a heavy smoker presents with shortness of breath & weight loss. On examination she is clubbed. The chest x-ray shows a perihilar shadow.

What would be the most appropriate investigation?
A. Colonoscopy
B. Bronchoscopy
C. Sputum culture
D. Abdominal ultrasound
E. Echocardiogram
F. Stool culture
G. Lung funtion tests
H. Chest x-ray
A

B. Bronchoscopy

The history of smoking and weight loss point to a bronchial carcinoma. Whilst the initial investigation is with a CXR, diagnosis relies on pathological confirmation from a tissue sample, often obtained from bronchoscopy. Non-small cell lung cancer is more often associated with clubbing.

How well did you know this?
1
Not at all
2
3
4
5
Perfectly
33
Q

A 37 year old Somali doctor with a history of coughing up about half a cup of fresh blood on 4-5 occasions over the period of 2 months. He had lost 5kg in weight and had drenching night sweats almost daily. Chest x-ray shows a large cavitating nodule in the left upper zone.

What is the most likely diagnosis?
A. Pulmonary aspergillosis
B. Thrombocytopenia
C. Pulmonary embolus
D. Arterio-venous malformation
E. Asthma
F. Bronchiectasis
G. Tuberculosis
H. Sickle cell crisis
I. Acute left ventricular failure
J. Carcinoma of bronchus
K. Inhaled foreign body
A

G. Tuberculosis

This patient has pulmonary TB. Risk factors include HIV infection, exposure to infection and returning from or being born in a high risk region such as Somalia, or other areas of Africa, Asia and Latin America. The night sweats here, weight loss and haemoptysis are all suggestive. The CXR finding of a cavitating lesion and upper zone changes are also consistent with TB. If TB is suspected, the patient should be placed in isolation and 3 sputum samples cultured for AFB being the gold standard of diagnosis. Culture takes several weeks so sputum smears will be done before culture results are known. Interferon-gamma release assays (IGRAs) are now used by some hospitals to rapidly determine a patient’s TB status. All patients who have TB should be tested for HIV within 2 months of diagnosis.

How well did you know this?
1
Not at all
2
3
4
5
Perfectly
34
Q

A 55 year old lorry driver, a ‘fit’ smoker of 20-30 cigarettes a day, presents to his GP with a history of coughing up 2 streaks of blood on separate mornings in the previous week. He has had a morning cough with small amounts of sputum over the previous 10 years and admits on questioning that he may have lost weight and has had to tighten his trouser belt. The ESR was 75 and chest x-ray showed collapse of the right middle lobe.

What is the most likely diagnosis?
A. Pulmonary aspergillosis
B. Thrombocytopenia
C. Pulmonary embolus
D. Arterio-venous malformation
E. Asthma
F. Bronchiectasis
G. Tuberculosis
H. Sickle cell crisis
I. Acute left ventricular failure
J. Carcinoma of bronchus
K. Inhaled foreign body
A

J. Carcinoma of bronchus

The weight loss, respiratory symptoms of haemoptysis and the history of smoking makes this likely to be bronchial carcinoma. Lobar collapse here may be a consequence of obstruction. First line treatment aims at surgical resection if possible. Small cell lung cancer is treated with chemotherapy and is associated with SIADH and ectopic ACTH. Non-small cell lung cancer is more often associated with clubbing. Squamous cell carcinoma is associated with PTHrp release and is treated with radiotherapy. Adenocarcinomas are usually located peripherally in the lung and are more common in non-smokers although most cases are still associated with smoking. The paraneoplastic syndromes may include Lambert-Eaton myasthenic syndrome.

How well did you know this?
1
Not at all
2
3
4
5
Perfectly
35
Q

A 50 year old cleaner, with a history of pertussis as a child, had a fever and increasing shortness of breath with sweating on exertion. She has had a cough productive of yellow sputum for 25 years but this is recently blood stained. She is a non-smoker. She has come to her GP for a further course of antibiotics and he refers her for a chest X-ray because he thinks she is clubbed.

What is the most likely diagnosis?
A. Pulmonary aspergillosis
B. Thrombocytopenia
C. Pulmonary embolus
D. Arterio-venous malformation
E. Asthma
F. Bronchiectasis
G. Tuberculosis
H. Sickle cell crisis
I. Acute left ventricular failure
J. Carcinoma of bronchus
K. Inhaled foreign body
A

F. Bronchiectasis

Bronchiectasis is permanent bronchi dilatation due to bronchial wall damage and loss of elasticity. It is often as a consequence of recurrent/severe infections and most present with chronic productive mucopurulent cough. The most common identifiable cause is CF but this history here of pertussis is responsible in this instance. Chest CT is the diagnostic test. Diagnosis is aided by sputum analysis. Not only should the GP have this diagnosis in mind but should know whether the patient is actually clubbed, instead of ‘thinking’ she is clubbed. Bronchiectasis is a cause of clubbing.

How well did you know this?
1
Not at all
2
3
4
5
Perfectly
36
Q

A 25 year old public relations executive has gained weight on the combined oral contraceptive and is reluctant to give up smoking 10-15 cigarettes a day. She presents at an A&E department with acute left sided pleuritic chest pain, slight shortness of breath and a cough with a little sputum containing smears of blood. Chest X-ray is unremarkable.

What is the most likely diagnosis?
A. Pulmonary aspergillosis
B. Thrombocytopenia
C. Pulmonary embolus
D. Arterio-venous malformation
E. Asthma
F. Bronchiectasis
G. Tuberculosis
H. Sickle cell crisis
I. Acute left ventricular failure
J. Carcinoma of bronchus
K. Inhaled foreign body
A

C. Pulmonary embolus

The underlying pathophysiology of PE is based on Virchow’s triad. SOB is a common symptom and there may also be pleuritic chest pain and haemoptysis. Strong risk factors include recent surgery, DVT, obesity, prolonged bed rest, malignancy, previous VTE, pregnancy and the thrombophilias such as factor V Leiden. The oral contraceptive pill is associated with an increased risk of VTE but is a weak risk factor. However, the rest of this history still makes the diagnosis in this case. CXR may be normal like in this case or may have findings suggestive of PE such as band atelectasis, hemidiaphragm elevation, Fleischner’s sign, Westermark’s sign and Hampton hump. ECG may be normal, or may show tachycardia, new RAD, new RBBB or the classical S wave in I, Q wave with T inversion in III. Various clinical probability scores exist for PE and D-dimer can be used to exclude PE as a diagnosis.

How well did you know this?
1
Not at all
2
3
4
5
Perfectly
37
Q

A 45 year old woman who is HIV positive reports easy bruising, frequent nose bleeds and coughing up small streaks of blood. She is otherwise well at present. Chest X-ray shows no abnormality. A full blood count shows Hb of 10.5g/dl, WBC 5.0 x 109/l and platelet count 28 x 109/l.

What is the most likely diagnosis?
A. Pulmonary aspergillosis
B. Thrombocytopenia
C. Pulmonary embolus
D. Arterio-venous malformation
E. Asthma
F. Bronchiectasis
G. Tuberculosis
H. Sickle cell crisis
I. Acute left ventricular failure
J. Carcinoma of bronchus
K. Inhaled foreign body
A

B. Thrombocytopenia

HIV is implicated in idiopathic thrombocytopenic purpura, which is a condition of abnormally low platelet count of unknown cause. This question makes this dead easy by giving you the platelet count as 28 x109/L and all you need to do is to appreciate that a count under 150 x109/L is defined as thrombocytopenia. As well as the count, the patient has obvious symptoms of thrombocytopenia here with bruising, haemoptysis and epistaxis.

How well did you know this?
1
Not at all
2
3
4
5
Perfectly
38
Q

A 30 year old man appears jaundiced. FBC and peripheral blood smear is reported as normal. INR is normal. LFTs show elevated unconjugated bilirubin. Liver enzymes are normal.

What is the most likely diagnosis?
A. Liver secondaries
B. Primary biliary cirrhosis
C. Alcoholic cirrhosis
D. Gilbert’s syndrome
E. Paracetamol poisoning
F. Carcinoma of head of pancreas
G. Autoimmune haemolytic anaemia
H. Cholelithiasis
I. Viral hepatitis A
A

D. Gilbert’s syndrome

Gilbert’s occurs in an asymptomatic patient, often as an incidental finding or mild jaundice occuring in adolescence/young adult age. There is elevated unconjugated BR with other liver tests being normal. The blood smear is also normal with normal reticulocyte count, and normal Hb indicating that this is not due to haemolysis. It is a common syndrome and is not really a disease, more a physiological variant. No treatment is needed and this condition is due to decreased UDPGT activity leading to decreased conjugation of unconjugated bilirubin, leading to elevated levels.

How well did you know this?
1
Not at all
2
3
4
5
Perfectly
39
Q

A 20 year old woman complains of breathlessness and appears pale. Hb 7.3 g/dl, WBC 7.0 x 109/L, Platelets 100 x 109/l. Unconjugated bilirubin is elevated. Direct antiglobulin test is positive.

What is the most likely diagnosis?
A. Liver secondaries
B. Primary biliary cirrhosis
C. Alcoholic cirrhosis
D. Gilbert’s syndrome
E. Paracetamol poisoning
F. Carcinoma of head of pancreas
G. Autoimmune haemolytic anaemia
H. Cholelithiasis
I. Viral hepatitis A
A

G. Autoimmune haemolytic anaemia

This woman is complaining of symptoms of anaemia with SOB and pallor. Haemolytic anaemia can be either hereditary or acquired. Hereditary can be divided into 3 broad groups – either inherited defects in the membrane such as hereditary spherocytosis, enzyme deficiencies such as G6PDH deficiency or abnormal Hb production such as that seen in sickle cell anaemia and thalassaemia. Acquired can be either immune or non-immune in cause. Autoimmune HA here is due to autoantibodies, which occurs most often as part of another autoimmune process like SLE or RA or related to a lymphoproliferative disorder such as CLL. FBC here shows a low Hb consistent with the diagnosis, though it is important here to also request MCHC and reticulocyte count. Both of these would be increased in haemolytic anaemia. Increased breakdown of haem leads to an unconjugated hyperbilirubinaemia, though not >70-85, which would indicate some degree of liver impairment as well. LDH would also be raised, which can be helpful if there is no concurrent tissue damage. Haptoglobin (which binds free Hb) will also be low, and these two markers combined is 90% specific for HA. The giveaway for AIHA here is the positive DAT or Coombs’ test. This test detects IgG or complement bound to RBCs (hence a positive result suggests an immune cause of HA). The patient’s RBCs are washed and mixed with antiserum for IgG and C3d. The presence of IgG often indicates the presence of a warm antibody, whereas C3d suggests a cold antibody.

How well did you know this?
1
Not at all
2
3
4
5
Perfectly
40
Q

A 55 year old man complains of abdominal distension. Hb 10.0 g/dl, WBC 7.0 x 109/l, Platelets 100 x 109/l. The patient is hyponatraemic and hypokalaemic. Urea and creatinine are normal. Serum albumin is low. AST is elevated at 250 and ALT is elevated to 125. ALP and GGT are also elevated.

What is the most likely diagnosis?
A. Liver secondaries
B. Primary biliary cirrhosis
C. Alcoholic cirrhosis
D. Gilbert’s syndrome
E. Paracetamol poisoning
F. Carcinoma of head of pancreas
G. Autoimmune haemolytic anaemia
H. Cholelithiasis
I. Viral hepatitis A
A

C. Alcoholic cirrhosis

This is a case of alcoholic cirrhosis. AST and ALT are elevated in all forms of alcoholic liver disease, both to <300. The true upper limit of normal AST and ALT can be considered to be 30 units/L for men and 19 units/L for women. The AST/ALT ratio is classically >2, which is seen in about 70% of cases, and AST is usually above ALT level. The elevated ALP here may represent cholestasis. GGT is a more sensitive marker than AST or ALT for heavy alcohol use and elevation here suggests the cause is alcohol. The anaemia here in alcoholic liver disease can be due to many causes like iron deficiency, folate deficiency and hypersplenism. Thrombocytopenia may be secondary to alcohol induced BM suppression, folate deficiency or hypersplenism. Those with advanced cirrhosis frequently are also hyponatraemic and hypokalaemia is also seen. Urea and creatinine is normal in this patient but elevation may suggest the presence of hepatorenal syndrome. Cirrhosis is the end stage of chronic liver disease and the abdominal distension may be a sign of ascites.

How well did you know this?
1
Not at all
2
3
4
5
Perfectly
41
Q

A 30 year old man who has returned from a month-long business trip to India has been complaining of abdominal pain and nausea for a week. He has also vomitied and remembers having a meal of shellfish from a street vendor. He woke up this morning and noticed he was going yellow. His ALT is 5000 and total bilirubin 139.

What is the most likely diagnosis?
A. Liver secondaries
B. Primary biliary cirrhosis
C. Alcoholic cirrhosis
D. Gilbert’s syndrome
E. Paracetamol poisoning
F. Carcinoma of head of pancreas
G. Autoimmune haemolytic anaemia
H. Cholelithiasis
I. Viral hepatitis A
A

I. Viral hepatitis A

Hepatitis A is primarily transmitted via the faecal-oral route. After the virus is consumed and absorbed, it replicates in the liver and is excreted in the bile (to be re-transmitted). Transmission usually precedes symptoms by about 2 weeks and patients are non-infectious 1 week after onset of jaundice. The history can reveal risk factors such as living in an endemic area, contact with an infected person, homosexual sex or a known food-borne outbreak. This is classically, in EMQs, associated with shellfish which is harvested from sewage contaminated water. If the patient has other liver diseases such as HBV or HCV or cirrhosis then there is a higher risk of fulminant HAV infection. The clinical course of HAV consists of a pre-icteric phase, lasting 5-7 days, consisting characteristically of N&V, abdominal pain, fever, malaise and headache. Rarer symptoms may be present such as arthralgias and even severe thrombocytopenia and signs that may be found include splenomegaly, RUQ tenderness and tender hepatomegaly as well as bradycardia. The icteric phase is characterised by dark urine, pale stools, jaundice and pruritis. When jaundice comes on, the pre-icteric phase symptoms usually diminish, and jaundice typically peaks at 2 weeks. However, a fulminant course runs in <1% of patients with worsenining jaundice and encephalopathy. Serum transaminases may reach in excess of 10,000 units, although there is little correlation between the level and disease severity. ALT is typically higher than AST.

How well did you know this?
1
Not at all
2
3
4
5
Perfectly
42
Q

A 45 year old obese woman complains of intermittent abdominal pain with jaundice. Hb 13.1 g/dl, WBC 8.3 x 109/l, Platelets 200 x 109/l, Na 140, K 4.1, Urea 5.2, Creatinine 88, ALP 580, Bilirubin 264, ALT 70

What is the most likely diagnosis?
A. Liver secondaries
B. Primary biliary cirrhosis
C. Alcoholic cirrhosis
D. Gilbert’s syndrome
E. Paracetamol poisoning
F. Carcinoma of head of pancreas
G. Autoimmune haemolytic anaemia
H. Cholelithiasis
I. Viral hepatitis A
A

H. Cholelithiasis

Gallstones, or cholelithiasis, is highly prevalent but most are asymptomatic. Symptoms occur when the cystic or bile duct get obstructed or when the gallstones erode through the gallbladder. The biliary pain here is caused by transient cystic duct obstruction or passage/obstruction through the bile duct. The typical presentation is of RUQ or epigastric pain often after food which increases in intensity and lasts for several hours. The jaundice here suggests choledocholithiasis (the presence of at least one gallstone in the CBD). Gallstones in EMQs classically involves the Fs (Fat, Forty, Female, Fertile, Fair) which reflects the risk factors such as older age, female sex, obesity, rapid weight loss, pregnancy and drugs. USS is the definitive initial investigation. HIDA scanning and MRI may help if the diagnosis remains unclear. Definitive treatment is with laparoscopic cholecystectomy for those with symptoms. Complications of gallstones include cholecystitis, cholangitis and pancreatitis and symptoms may indeed overlap between these conditions.

How well did you know this?
1
Not at all
2
3
4
5
Perfectly
43
Q

A 45 year old woman from Jamaica presents with a 6 month history of weight loss and a two week history of fever. On examination she had cervical lymphadenopathy. Her calcium was raised at 3.0 and CXR showed bilateral hilar lymphadenopathy.

What is the most likely diagnosis?
A. Post immunisation
B. Malaria
C. HIV infection
D. Hodgkin’s lymphoma
E. Appendicitis
F. Systemic lupus erythematosis
G. Pyelonephritis
H. Sarcoidosis
I. Drug reaction
J. Influenza
K. Glandular fever
L. Gastric carcinoma
M. Pneumonia
N. Tuberculosis
A

H. Sarcoidosis

Sarcoidosis is a chronic multisystem disease with an unknown aetiology. Lymphadenopathy is a common presentation and nodes are enlarged but non-tender, typically involving the cervical and submandibular nodes. Although uncommon, the patient may present with unexplained modest weight loss (which is often mistaken for TB or lymphoma along with the abnormal CXR) and a low-grade fever. CXR will typically show bilateral hilar lymphadenopathy and CXR findings are used in the staging of disease. Additionally, serum calcium and ACE levels may be raised. A transbronchial biopsy is essential for diagnosis in most cases and shows the presence of non-caseating granulomas. Black people have a higher lifetime risk of sarcoidosis, as do those of Scandinavian origin. The mainstay of treatment for severe disease involves systemic corticosteroids. Skin manifestations include erythema nodosum which are tender erythematous nodules and lupus pernio presenting with indurated plaques with discoloration on the face.

How well did you know this?
1
Not at all
2
3
4
5
Perfectly
44
Q

A 25 year old man with a 3 day history of high temperatures, aching limbs and neck discomfort. Apart from temperature of 39℃and some mild conjunctivitis, examination was normal. Antibiotics were prescribed but had had no effect.

What is the most likely diagnosis?
A. Post immunisation
B. Malaria
C. HIV infection
D. Hodgkin’s lymphoma
E. Appendicitis
F. Systemic lupus erythematosis
G. Pyelonephritis
H. Sarcoidosis
I. Drug reaction
J. Influenza
K. Glandular fever
L. Gastric carcinoma
M. Pneumonia
N. Tuberculosis
A

J. Influenza

This is an acute respiratory tract infection caused by seasonal viral influenza A or B, hence antibiotics would have no effect. It is characterised by respiratory symptoms including rhinorrhoea, cough, fever, chills, headache and myalgia. Recent upper respiratory tract infection has led to associated viral conjuncitivitis. The examination in this case is of course otherwise unremarkable. Antigenic change presents a challenge in creating new vaccines as there is no incremental protection from previous vaccinations. There have been 4 pandemics since 1918, the recent one being ‘swine flu’ in April 2009, caused by new gene rearrangement of human, avian and swine influenza.

How well did you know this?
1
Not at all
2
3
4
5
Perfectly
45
Q

A 50 year woman normally resident in the UK returned from visiting relatives in Pakistan. She described intermittent fevers with rigors, diarrhoea and severe headaches. She is mildly jaundiced.

What is the most likely diagnosis?
A. Post immunisation
B. Malaria
C. HIV infection
D. Hodgkin’s lymphoma
E. Appendicitis
F. Systemic lupus erythematosis
G. Pyelonephritis
H. Sarcoidosis
I. Drug reaction
J. Influenza
K. Glandular fever
L. Gastric carcinoma
M. Pneumonia
N. Tuberculosis
A

B. Malaria

In the Western world, almost all cases of malaria occurs in travellers so an adequate travel history is crucial or the diagnosis may be missed. Patients typically present with non-specific symptoms such as a fever, sweats, chills and myalgia. This can also include the diarrhoea which this patient is experiencing and headaches. The jaundice here suggests falciparum infection, which is always the cause in severe disease. This woman has just returned from an endemic area. Sometimes EMQs will describe patterns of fevers occuring at regular intervals of 48-72 hours associated with P. vivax, P. ovale and P. malariae infections but in most patients there is no specific pattern. Hepatosplenomegaly is a common presenting sign although not common at presentation in a first world setting. Thrombocytopenia is common with falciparum infection and a mild degree of anaemia ais commonly seen. WCC can be high, low or normal. The severity of malaria depends partly on the species and also on host immunity. Therefore those who live in endemic areas may develop minimal symptoms due to IgG antibody and cell-mediated immunity and physiological tolerance of parasitaemia. Pregnant women affected by P. falciparum are also susceptible to the complications of pregnancy due to placental parasite sequestration. Treatment of malaria in pregnancy must be managed with an ID specialist and should be treated with IV antimalarial therapy. The test of choice is Giesma-stained thick and thin blood smears. Thick films sensitively detect parasites whereas thin films allow species identification and calculation of parasitaemia to guide treatment. Studies have shown that for P falciparum, the most effective treatment is artesunate which is more effective than quinine without the risk of cinchonism. Numerous studies such as the AQUAMAT study in The Lancet showing that quinine should no longer be the established treatment of choice.

How well did you know this?
1
Not at all
2
3
4
5
Perfectly
46
Q

A 22 year old man presented with a two week history of fever and drenching night sweats. He had experienced severe itching during this time. Examination was normal except for swollen supraclavicular lymph nodes. CXR showed a mediastinal mass.

What is the most likely diagnosis?
A. Post immunisation
B. Malaria
C. HIV infection
D. Hodgkin’s lymphoma
E. Appendicitis
F. Systemic lupus erythematosis
G. Pyelonephritis
H. Sarcoidosis
I. Drug reaction
J. Influenza
K. Glandular fever
L. Gastric carcinoma
M. Pneumonia
N. Tuberculosis
A

D. Hodgkin’s lymphoma

This is a case of lymphoma. Reed-Sternberg cells are binucleate cells characteristically seen in Hodgkin’s lymphoma. Hodgkin’s is localised to a single group of nodes (normally the cervical and/or supraclavicular) and extranodal involvement is rare. Mediastinal involvement is common. Spread is contiguous and B symptoms may be present such as a low grade fever, weight loss and night sweats. Pruritis may be found in approximately 10% of cases but has no prognostic significance. 50% of cases is associated with EBV infection and distribution is bimodal with peaks in young and old. There is classically pain in lymph nodes on alcohol consumption.

How well did you know this?
1
Not at all
2
3
4
5
Perfectly
47
Q

A 35 year old pregnant woman developed a temperature with chills and increased urinary frequency. She is tender in the right loin and has vomited. Dipstick urinalysis is positive for leukocytes, nitrites and blood.

What is the most likely diagnosis?
A. Post immunisation
B. Malaria
C. HIV infection
D. Hodgkin’s lymphoma
E. Appendicitis
F. Systemic lupus erythematosis
G. Pyelonephritis
H. Sarcoidosis
I. Drug reaction
J. Influenza
K. Glandular fever
L. Gastric carcinoma
M. Pneumonia
N. Tuberculosis
A

G. Pyelonephritis

Acute onset fever with chills, flank pain, vomiting and positive urine dipstick all point to the diagnosis of acute pyelonephritis. Urinalysis is highly sensitive but not very specific. Pregnancy is a risk factor for complicated disease as the enlarging uterus compresses the ureters and hormonal changes increase the likelihood of obstructive uropathy. In uncomplicated pyelonephritis, the most common cause is E. coli and gram stain will typically reveal gram negative rods, either E. coli, Proteus or Klebsiella. Gram positive cocci that could be implicated include enterococci and staphylococci. Older patients can often also present non-specifically. Treatment should start before culture results are received to prevent the patient from deteriorating, with empirical antibiotics.

How well did you know this?
1
Not at all
2
3
4
5
Perfectly
48
Q

A 50 year old woman presented with a temperature and aching joints, 2 days prior to her departure on holiday to Egypt. On examination she had a tender swelling on her left upper arm.

What is the most likely diagnosis?
A. Post immunisation
B. Malaria
C. HIV infection
D. Hodgkin’s lymphoma
E. Appendicitis
F. Systemic lupus erythematosis
G. Pyelonephritis
H. Sarcoidosis
I. Drug reaction
J. Influenza
K. Glandular fever
L. Gastric carcinoma
M. Pneumonia
N. Tuberculosis
A

A. Post immunisation

Travel to Egypt may necessitate vaccinations for diseases such as typhoid and HAV. This is a side-effect of the vaccine and the history here points towards this. Vaccine side effects tend to be uncommon although this depends on the specific vaccine given. For example, some 10-30% of people will experience mild side effects such as muscle pain and headache after being given the yellow fever vaccine.

How well did you know this?
1
Not at all
2
3
4
5
Perfectly
49
Q

A 35 year old woman complains of weight loss, tremors, palpitations, a large appetite and diarrhoea. Examination revealed tachycardia over 120 beats/min and exophthalmos. She appeared anxious.

What is the most likely diagnosis?
A. Infected dermoid cyst
B. Anaplastic carcinoma
C. Anxiety state
D. Myxoedema
E. Toxic multinodular goitre
F. Thyroglossal cyst
G. Pituitary failure
H. Viral thyroiditis
I. Grave’s disease
J. Papillary thyroid carcinoma
K. Medullary thyroid carcinoma
L. Carotid body tumour
A

I. Grave’s disease

This patient is obviously hyperthyroid with the weight loss despite an increased appetite, tremor, palpitations, tachycardia and diarrhoea. This patient has Graves’ disease, which is the most common cause of hyperthyroidism in countries with an adequate iodine intake. Peripheral manifestations, such as in this case exopthalmos (and pretibial myxoedema, hyperthyroid acropachy), do not occur with other causes of hyperthyroidism. Where there is pretibial myxoedema, there is almost always opthalmopathy too. Treatment aims to normalise thyroid function and is achieved by radioactive iodine, antithyroid medications or with surgery. They are all effective and relatively safe options. Symptomatic therapy is given with beta blockers such as propranolol.

How well did you know this?
1
Not at all
2
3
4
5
Perfectly
50
Q

50 yr old woman complains of weight loss, tremor of hands, palpitations, large appetite, diarrhoea and an irregular goitre in the neck. The patient is tachycardiac and eye examination revealed no exopthalmos or lid lag.

What is the most likely diagnosis?
A. Infected dermoid cyst
B. Anaplastic carcinoma
C. Anxiety state
D. Myxoedema
E. Toxic multinodular goitre
F. Thyroglossal cyst
G. Pituitary failure
H. Viral thyroiditis
I. Grave’s disease
J. Papillary thyroid carcinoma
K. Medullary thyroid carcinoma
L. Carotid body tumour
A

E. Toxic multinodular goitre

This patient is again obviously hyperthyroid with the symptoms she is presenting with but does not have the peripheral stigmata of Graves’. This is a case of toxic multinodular goitre characterised by the irregular goitre, rather than the smooth goitre of Graves’. There may be substernal extension. It is most common in older patients and is associated with head and neck irradiation and iodine deficiency. TSH is the initial screening test and if supressed, T4/T3 levels are measured. A thyroid scan and uptake will show multiple hot and cold areas consistent with areas of autonomy and areas of suppression. Definitive treatment is commonly given in the form of radioactive iodine.

How well did you know this?
1
Not at all
2
3
4
5
Perfectly
51
Q

24 yr old woman complains of weight loss, tremor of hands, palpitations and loss of appetite. Examination revealed tachycardia and she appeared nervous. Investigation revealed TSH 1.0 (range 0.3-4.5) and ESR within normal limits.

What is the most likely diagnosis?
A. Infected dermoid cyst
B. Anaplastic carcinoma
C. Anxiety state
D. Myxoedema
E. Toxic multinodular goitre
F. Thyroglossal cyst
G. Pituitary failure
H. Viral thyroiditis
I. Grave’s disease
J. Papillary thyroid carcinoma
K. Medullary thyroid carcinoma
L. Carotid body tumour
A

C. Anxiety state

TSH is normal so there is no thyroid disorder here despite the patient’s symptoms and the patient also appears nervous. Anxiety would explain the patient’s symptoms here and weight loss may be incorporated as part of an anxiety disorder.

How well did you know this?
1
Not at all
2
3
4
5
Perfectly
52
Q

A 24 year old woman complains of weight loss, tremor of hands, palpitations and pain in neck radiating to the left ear and tiredness. Examination revealed tender swelling in neck, tachycardia of 120 beats/min and fever of 38oC. TSH <0.05 (range 0.3-4.5) and ESR 60mm/hr.

What is the most likely diagnosis?
A. Infected dermoid cyst
B. Anaplastic carcinoma
C. Anxiety state
D. Myxoedema
E. Toxic multinodular goitre
F. Thyroglossal cyst
G. Pituitary failure
H. Viral thyroiditis
I. Grave’s disease
J. Papillary thyroid carcinoma
K. Medullary thyroid carcinoma
L. Carotid body tumour
A

H. Viral thyroiditis

This woman has de Quervain’s (viral) thyroiditis (which can also be called subacute granulomatous thyroiditis) which is inflammation of the thyroid characterised by a triphasic course where there is transient thyrotoxicosis followed by a hypothyroid phase before a return to euthyroidism. The thyrotoxic phase (symptoms of hyperthyroidism may are present here) is characterised by pain and tenderness of the thyroid, which tends to be larger, firm and tender to touch. This woman also has a fever and neck pain. This is a self-limiting condition and no specific treatment is needed though NSAIDs and beta blockers can be used for symptomatic relief. Roughly 30-40% describe a prior viral infection.

How well did you know this?
1
Not at all
2
3
4
5
Perfectly
53
Q

An 18 year old woman complains of a swelling in the neck. Examination revealed the swelling is painless, non-tender. There is no weight loss, no tremors of hands or palpitations. Swelling moves up and down with protrusion and intrusion of tongue respectively. Investigation revealed normal TSH and ESR.

What is the most likely diagnosis?
A. Infected dermoid cyst
B. Anaplastic carcinoma
C. Anxiety state
D. Myxoedema
E. Toxic multinodular goitre
F. Thyroglossal cyst
G. Pituitary failure
H. Viral thyroiditis
I. Grave’s disease
J. Papillary thyroid carcinoma
K. Medullary thyroid carcinoma
L. Carotid body tumour
A

F. Thyroglossal cyst

This midline neck swelling moves up on tongue protrusion (and swallowing) making this a thyroglossal cyst. It is a cyst that forms from a remnant thyroglossal duct and can hence develop anywhere along the length of this embryological duct, which is a midline structure between the foramen caecum at the back of the tongue and the thyroid gland.

How well did you know this?
1
Not at all
2
3
4
5
Perfectly
54
Q

A 50 yr old man complains of a lump in neck on the left side. Examination revealed painless, non tender lump on the left side of the neck which moves on swallowing. Investigation revealed normal TSH and ESR. Fine needle aspiration revealed ‘Orphan Annie’ eyes and psammoma bodies.

What is the most likely diagnosis?
A. Infected dermoid cyst
B. Anaplastic carcinoma
C. Anxiety state
D. Myxoedema
E. Toxic multinodular goitre
F. Thyroglossal cyst
G. Pituitary failure
H. Viral thyroiditis
I. Grave’s disease
J. Papillary thyroid carcinoma
K. Medullary thyroid carcinoma
L. Carotid body tumour
A

J. Papillary thyroid carcinoma

TSH is normal and the examination findings point to a carcinoma, which most commonly presents like this – an asymptomatic nodule. If TSH were suppressed then this would suggest hyperthyroidism, or a hot nodule, in which case the incidence of cancer is very low. If TSH, like this case, is normal, then a FNA is indicated and cytology may then help to tell cancer type. Histology cannot distinguish follicular adenomas and carcinomas but is used for diagnosis of follicular carcinoma (when combined with other features) instead of cytology. There are 5 types of thyroid cancer: papillary (most common), follicular, medullary (about a quarter are familial e.g. MEN), anaplastic (worst prognosis) and lymphoma. Orphan Annie eyes and psammoma bodies are seen in papillary cancer. Average 10 year survival is >90%. This is a hard question.

How well did you know this?
1
Not at all
2
3
4
5
Perfectly
55
Q

A 20 year old male with generalised oedema. He has noticed ‘frothy’ urine recently. Liver function tests show plasma albumin of 15g/l.

What is the most appropriate investigation?
A. 24hr urinary protein
B. Lymph node biopsy
C. Venous duplex scan
D. Echocardiogram
E. Creatinine clearance
F. Liver function tests
G. Drug history
H. Full blood count
I. Urea &amp; electrolytes
J. Polysomnography
K. Pelvic ultrasound
L. V/Q scan
M. Blood film examination
A

A. 24hr urinary protein

This man has nephrotic syndrome, which is defined by the presence of proteinuria (>3.5g/24 hours), hypoalbuminaemia (<30g/L) oedema, and hyperlipidaemia. You should note that this is different to nephritic syndrome which is typically defined by acute kidney injury, hypertension and an active urinary sediment (RBCs and RBC casts). This is a constellation of several symptoms which can have many causes. The likely cause differs depending on how old the patient is and whether the patient has co-morbidities like diabetes. The most common cause in children is minimal change disease, whereas the most common cause in young adults is focal segmental glomerulosclerosis. Membranous nephropathy is most common in older people and you should suspect diabetic nephropathy as a cause in those with long standing DM. Note also that those who present with proteinuria without the other components needed to define nephrotic syndrome are described as having ‘nephrotic range proteinuria’.
In terms of physiology, the glomerular filtration barrier is disrupted by disease leading to glomerular proteinuria (the scale of which is linked to how severe the condition is and renal function). The loss of albumin leads to hypoalbuminaemia. The liver then tries to compensate for the protein loss by increases albumin synthesis, and this also increases synthesis of LDL, vLDL and lipoprotein A, leading to lipid abnormalities. The patient is also in a hypercoagulable state due to the loss of coagulatiion inhibitors in urine and increased synthesis of clotting factors in the liver. A 24-hour urine protein collection would establish proteinuria and whether it is in the nephrotic range. If tis is impractical then the protein/creatinine ratio can be obtained.

How well did you know this?
1
Not at all
2
3
4
5
Perfectly
56
Q

A 60 year old woman complained of a painful swollen left leg on arrival at Heathrow airport. On examination her leg is tender with shiny skin. She is on the oral contraceptive pill.

What is the most appropriate investigation?
A. 24hr urinary protein
B. Lymph node biopsy
C. Venous duplex scan
D. Echocardiogram
E. Creatinine clearance
F. Liver function tests
G. Drug history
H. Full blood count
I. Urea &amp; electrolytes
J. Polysomnography
K. Pelvic ultrasound
L. V/Q scan
M. Blood film examination
A

C. Venous duplex scan

This is a DVT as I am sure you have already figured out.The recent presumably long-haul air travel is a strong risk factor though the exact mechanism of this risk is currently unclear, but may involve pressure changes, dehydration and mobility. This woman also has another risk factor with the use of oestrogen-containing oral contraceptives, although this is a small individual risk. The risk here is greatest in the first year of use. Other strong risk factors include recent surgery, especially orthopaedic surgery, active malignancy, pregnancy, obesity and coagulopathies such as factor V Leiden. A Wells score is determined in all patients with a suspected DVT with the condition being likely if the score is 2 or more. If the Wells score is <2 then a D-dimer level is indicated. A compression ultrasound of the proximal deep venous system is the first line investigation in those where there is a high clinical suspicion. However, Doppler venous flow studies can be used when all other tests are unavailable although this test has a low sensitivity and requires a trained technician. A low flow in the veins is indicative of a DVT.

How well did you know this?
1
Not at all
2
3
4
5
Perfectly
57
Q

A 45 year old publican developed bilateral limb oedema and noticed that his trousers were becoming tight. On examination he had gynaecomastia and Duputyren’s contractures

What is the most appropriate investigation?
A. 24hr urinary protein
B. Lymph node biopsy
C. Venous duplex scan
D. Echocardiogram
E. Creatinine clearance
G. Drug history
H. Full blood count
I. Urea &amp; electrolytes
J. Polysomnography
K. Pelvic ultrasound
L. V/Q scan
M. Blood film examination
A

F. Liver function tests

This is a sign of decompensation of hepatic cirrhosis which has caused peripheral oedema due to hypoalbuminaemia. The gynaecomastia is a sign of chronic liver disease (tender and firm enlarged breast bud) as well as the Dupuytren’s contracture of alcoholic liver disease. Cirrhosis is the end-stage of chronic liver disease, in this case due to alcohol. Cirrhosis results in hepatic insufficiency and portal hypertension. Other complications include ascites, variceal bleeds, jaundice, hepatic encephalopathy, hepatorenal syndrome and the development of HCC. The only curative option after decompensation is a timely referral for liver transplantation. Management is aimed at treating the underlying liver disease – this man needs to stop drinking.

How well did you know this?
1
Not at all
2
3
4
5
Perfectly
58
Q

A 60 year old female with uncontrolled hypertension, whose medication has been recently changed by he GP presents with bilateral ankle swelling. Clinical examination of her chest and cardiovascular system were normal.

What is the most appropriate investigation?
A. 24hr urinary protein
B. Lymph node biopsy
C. Venous duplex scan
D. Echocardiogram
E. Creatinine clearance
F. Liver function tests
H. Full blood count
I. Urea &amp; electrolytes
J. Polysomnography
K. Pelvic ultrasound
L. V/Q scan
M. Blood film examination
A

G. Drug history

A new change in medication here has led to peripheral oedema. The examination is normal which would lead you away from suspecting causes such as heart failure (plus the patient does not seem to be complaining of any other symptoms such as SOB). The JVP is normal as well. These agents which can cause bilateral oedema include CCBs, especially the dihydropyridines and agents which cause vasodilation such as minoxidil or diazoxide. Drugs which act to block renal prostaglandin synthesis such as NSAIDs can also lead to renal salt and water retention which would also contribute to peripheral oedema. There are other medications associated with peripheral oedema such as antidepressants, oestrogens and steroids. The clinical suspicion here should very much lie towards the new medication, which has coincided with the onset of oedema. This diagnosis would be supported by improvement in oedema with the withdrawal of the offending drug.

How well did you know this?
1
Not at all
2
3
4
5
Perfectly
59
Q

A 70 year old woman with swelling of her right leg. She gives a history of recent weight loss and lower abdominal discomfort. Examination shows pitting oedema of the entire leg and a possible right sided pelvic mass.

What is the most appropriate investigation?
A. 24hr urinary protein
B. Lymph node biopsy
C. Venous duplex scan
D. Echocardiogram
E. Creatinine clearance
F. Liver function tests
G. Drug history
H. Full blood count
I. Urea &amp; electrolytes
J. Polysomnography
K. Pelvic ultrasound
L. V/Q scan
M. Blood film examination
A

K. Pelvic ultrasound

There is a possible mass in the pelvis which is most likely malignant given the history. This needs to be investigated by pelvic ultrasound. This is likely secondary lymphoedema due to malignancy with the weight loss and abdominal discomfort. This is initially pitting. Stemmer’s sign may be useful, which is the inability to pinch and lift a fold of skin at the base of the second toe.

How well did you know this?
1
Not at all
2
3
4
5
Perfectly
60
Q

A 60 year old man who is a heavy smoker presents with a longstanding history of cough and breathlessness. He has recently noticed increasing difficulty in walking due to leg swelling. On examination he is centrally cyanosed and has bilateral wheezes on chest auscultation.

What is the most appropriate investigation?
A. 24hr urinary protein
B. Lymph node biopsy
C. Venous duplex scan
D. Echocardiogram
E. Creatinine clearance
F. Liver function tests
G. Drug history
H. Full blood count
I. Urea &amp; electrolytes
J. Polysomnography
K. Pelvic ultrasound
L. V/Q scan
M. Blood film examination
A

D. Echocardiogram

This is cor pulmonale in a patient with a history of COPD. There is peripheral oedema and wheezes on chest examination. Cor pulmonale is right heart failure secondary in this case to long standing COPD, caused by chronic hypoxia and pulmonary vascular vasoconstriction secondary to this, giving pulmonary hypertension and right sided heart failure. Signs aside from lower extremity oedema also include hepatomegaly, a loud P2 and engorged neck veins. The first test to order is an echocardiogram which will slow elevated pulmonary arterial pressure. Further tests may include spirometry and CT chest.

How well did you know this?
1
Not at all
2
3
4
5
Perfectly
61
Q

A 30 year old female smoker has atrophic waxy yellowish plaques on her shins with surrounding erythema.

What is the most likely diagnosis?
A. Diabetes
B. Scleroderma
C. Ulcerative colitis
D. Hyperthyroidism
E. Dermatomyositis
F. Amyloidosis
G. Reactive arthritis
H. Rheumatoid arthritis
I. Hypothyroidism
J. Coeliac disease
A

A. Diabetes

This is necrobiosis lipoidica which is a necrotising condition usually occuring in those with diabetes (and can be called necrobiosis lipodica diabeticorum). Most people with the condition are diabetics but only a small proportion of diabetics have the condition. It tends to appear on the shins, often bilaterally, although it can occur on other sites. They are often asymptomatic (due to associated neuropathy) but may be painful and ulcerate if injured. It tends to have the appearance of an erthematous area of hardened and raised skin (initially a reddish brown), progressing to a yellowish tint. Try and have a look at some photos online. These lesions may display the Koebner phenomenon.

How well did you know this?
1
Not at all
2
3
4
5
Perfectly
62
Q

A 35 year old woman has tight shiny skin on her hands & around her mouth. There are also telangiectasia on her face. She has had Raynaud’s phenomenon for the past 5 years and a history of GORD.

What is the most likely diagnosis?
A. Diabetes
B. Scleroderma
C. Ulcerative colitis
D. Hyperthyroidism
E. Dermatomyositis
F. Amyloidosis
G. Reactive arthritis
H. Rheumatoid arthritis
I. Hypothyroidism
J. Coeliac disease
A

B. Scleroderma

This is scleroderma, or systemic sclerosis. It is a multi-system disease with initially non-specific symptoms such as tiredness, hand swelling, musculoskeletal symptoms and Raynaud’s phenomenon. The cause is currently unknown. There are two main types – the limited cutaneous type and the diffuse cutaneous tupe. The limited cutaneous form tends to have less severe internal organ involvement and a better prognosis. Telangiectasia is commonly found, most often on the fingers, palms, face and mucous membranes. They can also be found in the stomach, where they are referred to as ‘watermelon stomach’. Characteristic findings of this condition also include sclerodactyly and thickening of the skin proximal to the MCP jpints. GORD, digital ulcers and lung involvement may also be found. Clinical course depends on the extent of both vascular and fibrotic complications. Vascular involvement includes Raynaud’s, ischaemic digital ulcers and hypertensive crisis as well as pulmonary arterial hypertension. Fibrosis can affect lungs, heart and GIT.

How well did you know this?
1
Not at all
2
3
4
5
Perfectly
63
Q

An 18 year old with a history of episodes of diarrhoea, in between which she suffers no symptoms, has painful pustular lesions which are ulcerating.

What is the most likely diagnosis?
A. Diabetes
B. Scleroderma
C. Ulcerative colitis
D. Hyperthyroidism
E. Dermatomyositis
F. Amyloidosis
G. Reactive arthritis
H. Rheumatoid arthritis
I. Hypothyroidism
J. Coeliac disease
A

C. Ulcerative colitis

This is pyoderma gangrenosum, which presents with multiple lesions, most commonly affecting the lower extremity and is linked to UC although it can be seen less commonly in Crohn’s, and is also seen in conditions such as RA and the myeloid dyscrasias. However, the diarrhoea here in the history makes this UC. These lesions start as tender papules or vesicles which develop into painful ulcers with a dusky purple edge and surrounding induration and erythema. The base may contain granulation tissue and lesions heal with atrophic scars. This patient will need a colonoscopy to investigate for UC. Skin culture may also reveal infection, which can occur. Physical trauma such as debridement is contraindicated as it can cause severe exacerbations.

How well did you know this?
1
Not at all
2
3
4
5
Perfectly
64
Q

A 28 year old woman has indurated plaques of reddish-yellow thickened skin over both shins. She also has areas of depigmentation on her hands & face.

What is the most likely diagnosis?
A. Diabetes
B. Scleroderma
C. Ulcerative colitis
D. Hyperthyroidism
E. Dermatomyositis
F. Amyloidosis
G. Reactive arthritis
H. Rheumatoid arthritis
I. Hypothyroidism
J. Coeliac disease
A

D. Hyperthyroidism

This is pretibial myxoedema and vitiligo in a woman with Graves’ disease. Graves’ disease is the most common cause of hyperthyroidism. Peripheral manifestations such as ophthalmopathy, pretibial myxoedema and hyperthyroid acropachy do not occur with other causes of hyperthyroidism. Pretibial myxoedema is almost always associated with ophthalmopathy. Treatment aims to normalise thyroid function and is achieved by radioactive iodine, antithyroid medications or with surgery. They are all effective and relatively safe options. Symptomatic therapy is given with beta blockers such as propranolol.

How well did you know this?
1
Not at all
2
3
4
5
Perfectly
65
Q

A 65 year old man with painful shoulders & hips has purple swollen eyelids & red scaly papules over the knuckles.

What is the most likely diagnosis?
A. Diabetes
B. Scleroderma
C. Ulcerative colitis
D. Hyperthyroidism
E. Dermatomyositis
F. Amyloidosis
G. Reactive arthritis
H. Rheumatoid arthritis
I. Hypothyroidism
J. Coeliac disease
A

E. Dermatomyositis

Dermatomyositis is an idiopathic myopathy which is characterised by hallmark cutaneous lesions such as a heliotrope rash and Gottron’s papules. Gottron’s papules are seen here and this is a pathognomic sign characterised by violet to dusty-red papules and plaques over the dorsal surface of the knuckles, wrists, elbows, knees and malleoli. The surface may be slightly scaling and telangiectasia may develop within the lesions. The periorbital violet rash in a symmetrical involvement is also a very characteristic sign and there may also be associated periorbital oedema. Skin manifestations are treated with topical antipruritics and topical corticosteroids. Photoprotection is advised for all patients – the rash often develops in sun exposed areas and some patients also report photosensitivity. UV radiation is a strong risk factor for this disease. Some patients only have cutaneous involvement whereas others may have associated lung disease, heart involvement, oesophageal involvement of underlying malignancy. Muscle disease is initially treated with high dose prednisolone followed by immunosuppresants and IV immunoglobulins in refractory cases.

How well did you know this?
1
Not at all
2
3
4
5
Perfectly
66
Q

A 21 year old Jamaican gentleman was referred to A&E by his GP, who on investigation following a chest infection found an incidental finding of a raised bilirubin of 120micromols/L. His LFTs are within normal ranges. O/E he was found to be jaundiced but he said that he had “always been that way”.

What is the most likely diagnosis?
A. Gilbert's syndrome
B. Hepatitis C
C. Primary biliary cirrhosis
D. Hepatocellular carcinoma
E. Hepatorenal syndrome
F. Hepatitis E
G. Hepatitis D
H. Hepatitis A
I. Hepatitis B
J. Portal hypertension
K. Gallstones
L. Pyogenic abscess
M. Alcoholic liver disease
N. GORD
A

A. Gilbert’s syndrome

Gilbert’s occurs in an asymptomatic patient, often as an incidental finding or mild jaundice occuring in adolescence/young adult age. There is elevated unconjugated BR with other liver tests being normal. The blood smear is also normal with normal reticulocyte count, and normal Hb indicating that this is not due to haemolysis. It is a common syndrome and is not really a disease, more a physiological variant. No treatment is needed and this condition is due to decreased UDPGT activity leading to decreased conjugation of unconjugated bilirubin, leading to elevated levels.

How well did you know this?
1
Not at all
2
3
4
5
Perfectly
67
Q

Mrs C is a 55 year old mother of two. Who has been feeling increasingly tired over the past few months. Investigations showed an ALT of 497. Her past medical history includes type 2 diabetes and arthritis. Her only other hospital admission was 15 years ago was when her youngest girl was born when she had major complications and needed several blood transfusions. After a few more tests the doctor prescribes peginterferon and ribavirin.

What is the most likely diagnosis?
A. Gilbert's syndrome
B. Hepatitis C
C. Primary biliary cirrhosis
D. Hepatocellular carcinoma
E. Hepatorenal syndrome
F. Hepatitis E
G. Hepatitis D
H. Hepatitis A
I. Hepatitis B
J. Portal hypertension
K. Gallstones
L. Pyogenic abscess
M. Alcoholic liver disease
N. GORD
A

I. Hepatitis C

The most common route which HCV is transmitted is through illicit IVDU. Following the acute exposure, most patients (55-85%) will go on to develop chronic hepatitis C. Most infections are asymptomatic. Treatment involves, usually, pegylated interferon and ribavirin with the aim of getting rid of viraemia. Triple therapy is used for the most common genotype 1, and this consists of the above two drugs as well as a HCV protease inhibitor. Long term complications of infection include cirrhosis or HCC.
The first case is one of transfusion-associated HCV, which occured before donor screening was put in place. This is currently an incredibly rare occurence in western countries. The serum transaminases, especially ALT are the easiest measure of disease activity (but are neither very sensitive nor very specific). The association between ALT level and the degree of chronic liver disease is however weak and ALT level does not reflect the progression to cirrhosis. Pegylated interferon itself can also cause mild ALT elevation. This is not HBV because of the last line which gives away the treatment, consistent with HCV. If this information was not given, this would still be more likely to be HCV than HBV as HCV has a greater chance of establishing chronic infection. However, there may be no differentiating signs or symptoms between the two. HBV surface antigen test would be positive if it was HBV and HCV antibody test negative.
The second case has two risk factors and has presented with classic signs of liver disease with ascites as a sign of decompensation. Tattoos are associated with HCV infection as is homosexuality. It is really important to note that HCV is not sexually transmitted. This may sound like a paradox… but HCV is trasmitted by blood, not sex. The reason homosexual intercourse is a risk factor for HCV and HCV is increasing in incidence among homosexual men in London is likely due to traumatic anal intercourse. Hence, the virus is still transmitted by blood. This is what the majority of studies seem to suggest, although there is still a bit of contention here, as there is with whether HIV can be transmitted through oral sex. HBV markers are negative here. Surface antigen appears 2-10 weeks after exposure and is usually detectable after 4-6 months with persistence indicating chronic infection. E antigen is a soluble viral protein sound in serum in the early acute phase of infection and usually disappears soon after or at the peak in ALT levels. Persistence again suggest chronicity.

How well did you know this?
1
Not at all
2
3
4
5
Perfectly
68
Q

Mr H is a 46 year old homosexual tattoo artist who presents to his GP because he has noticed worsening abdominal swelling and dark stool. On examination you notice spider naevi on the upper chest and gynaecomastia. Initial blood tests demonstrate negative HBsAg and HBeAg.

What is the most likely diagnosis?
A. Gilbert's syndrome
B. Hepatitis C
C. Primary biliary cirrhosis
D. Hepatocellular carcinoma
E. Hepatorenal syndrome
F. Hepatitis E
G. Hepatitis D
H. Hepatitis A
I. Hepatitis B
J. Portal hypertension
K. Gallstones
L. Pyogenic abscess
M. Alcoholic liver disease
N. GORD
A

B. Hepatitis C

The most common route which HCV is transmitted is through illicit IVDU. Following the acute exposure, most patients (55-85%) will go on to develop chronic hepatitis C. Most infections are asymptomatic. Treatment involves, usually, pegylated interferon and ribavirin with the aim of getting rid of viraemia. Triple therapy is used for the most common genotype 1, and this consists of the above two drugs as well as a HCV protease inhibitor. Long term complications of infection include cirrhosis or HCC.
The first case is one of transfusion-associated HCV, which occured before donor screening was put in place. This is currently an incredibly rare occurence in western countries. The serum transaminases, especially ALT are the easiest measure of disease activity (but are neither very sensitive nor very specific). The association between ALT level and the degree of chronic liver disease is however weak and ALT level does not reflect the progression to cirrhosis. Pegylated interferon itself can also cause mild ALT elevation. This is not HBV because of the last line which gives away the treatment, consistent with HCV. If this information was not given, this would still be more likely to be HCV than HBV as HCV has a greater chance of establishing chronic infection. However, there may be no differentiating signs or symptoms between the two. HBV surface antigen test would be positive if it was HBV and HCV antibody test negative.
The second case has two risk factors and has presented with classic signs of liver disease with ascites as a sign of decompensation. Tattoos are associated with HCV infection as is homosexuality. It is really important to note that HCV is not sexually transmitted. This may sound like a paradox… but HCV is trasmitted by blood, not sex. The reason homosexual intercourse is a risk factor for HCV and HCV is increasing in incidence among homosexual men in London is likely due to traumatic anal intercourse. Hence, the virus is still transmitted by blood. This is what the majority of studies seem to suggest, although there is still a bit of contention here, as there is with whether HIV can be transmitted through oral sex. HBV markers are negative here. Surface antigen appears 2-10 weeks after exposure and is usually detectable after 4-6 months with persistence indicating chronic infection. E antigen is a soluble viral protein sound in serum in the early acute phase of infection and usually disappears soon after or at the peak in ALT levels. Persistence again suggest chronicity.

How well did you know this?
1
Not at all
2
3
4
5
Perfectly
69
Q

Miss K is a 45 year old retired kitchen lady who presents to A+E with abdominal discomfort which has been getting worse over the past few weeks. She says she feels tired, bloated and has lost her appetite. Findings show antimitochrondrial antibodies are present and serum alkaline phosphatase is raised.

What is the most likely diagnosis?
A. Gilbert's syndrome
B. Hepatitis C
C. Primary biliary cirrhosis
D. Hepatocellular carcinoma
E. Hepatorenal syndrome
F. Hepatitis E
G. Hepatitis D
H. Hepatitis A
I. Hepatitis B
J. Portal hypertension
K. Gallstones
L. Pyogenic abscess
M. Alcoholic liver disease
N. GORD
A

C. Primary biliary cirrhosis

Primary biliary cirrhosis (PBC) is a chronic condition where the intrahepatic small bile ducts are progressively damaged (and eventually lost) occuring on a background of portal tract inflammation. Fibrosis develops, ultimately leading to cirrhosis (which is defined as fibrosis with nodular regeneration). It is widely believed to be autoimmune in aetiology as almost all patients have antimitochrondrial antibodies. This is the give away point in this question which is otherwise a non-specific presentation. Elevated ALP suggests cholestasis.

How well did you know this?
1
Not at all
2
3
4
5
Perfectly
70
Q

Mr S is a 70 year old retired semi professional footballer. He presents with pain in the right hypochondrium which does not radiate. On further questioning he says he has noticed a 5kg weight loss in the past couple of months and he says he has completely lost his appetite. He drinks an average of 1-2 units of alcohol per week and does not smoke. O/E gross ascites is present and obs chart shows he has been pyrexial since admission.

What is the most likely diagnosis?
A. Gilbert's syndrome
B. Hepatitis C
C. Primary biliary cirrhosis
D. Hepatocellular carcinoma
E. Hepatorenal syndrome
F. Hepatitis E
G. Hepatitis D
H. Hepatitis A
I. Hepatitis B
J. Portal hypertension
K. Gallstones
L. Pyogenic abscess
M. Alcoholic liver disease
N. GORD
A

D. Hepatocellular carcinoma

HCC usually arises in patients with cirrhosis, such as this patient, who has evidence of alcohol use and decompensation with ascites. The weight loss and the site of pain are also indicative. The RUQ pain is due to hepatomegaly, inflammation and stretching of the liver capsule. The ascites is associated with cirrhosis. A liver mass should be present on abdominal imaging and elevated AFP would be a corroborative finding. Treatment depends on the stage and prognosis. If a patient is previously known to have cirrhosis or are at a high risk then they should really be screened for a hepatoma by obtaining serum AFP and USS of the liver every 6 months. Treatment options would include: resection, transplant, chemo-embolisation and percutaneous ablation.

How well did you know this?
1
Not at all
2
3
4
5
Perfectly
71
Q

A 47 year old presents with a sharp central chest pain, worse on lying down, with a two week history of fever and malaise.

What is the most likely diagnosis?
A. GORD
B. Pericardial effusion
C. Stable angina
D. Pulmonary embolism
E. Unstable angina
F. Pleural effusion
G. Myocardial infarction
H. Pericarditis
I. Pneumonia
J. Pneumothorax
K. Emphysema
L.Peptic ulcer
M. Fractured rib
N. Asthma
O. Ventricular fibrillation
P. Aortic dissection
A

H. Pericarditis

This patient has presented with pericarditis. Symptoms include a sharp and severe chest pain retrosternally which is worse on inspiration and when supine, relieved by sitting forwards. The classical finding on examination is a friction rub which is said to sound like ‘walking on snow’. There may be diffuse ST elevations on ECG, an effusion on echocardiography and blood results suggesting inflammation. Complications include tamponade and constrictive pericarditis. Prior viral infection is a risk factor with the most common pericardial infection being viral. Bacterial purulent pericarditis also occurs. The inflammation is due either to direct viral attack or immune mediated damage. Other risk factors include male gender, post-MI (both ‘early’ and Dressler’s), post-pericardiotomy syndrome, neoplasm from local tumour invasion, uraemia and autoimmune conditions such as RA and SLE.

How well did you know this?
1
Not at all
2
3
4
5
Perfectly
72
Q

A 25 year old presents with chest pain, worse on inspiration and associated with a productive cough.

What is the most likely diagnosis?
A. GORD
B. Pericardial effusion
C. Stable angina
D. Pulmonary embolism
E. Unstable angina
F. Pleural effusion
G. Myocardial infarction
H. Pericarditis
J. Pneumothorax
K. Emphysema
L.Peptic ulcer
M. Fractured rib
N. Asthma
O. Ventricular fibrillation
P. Aortic dissection
A

I. Pneumonia

This case of pleuritic chest pain and a productive cough is classic pneumonia. The most specific and sensitive test is a CXR (PA and lateral) and initial treatment of a CAP is empirical with antibiotics. Often diagnosis is made solely on history and examination findings. Management is guided by the patient’s CURB-65 score.

How well did you know this?
1
Not at all
2
3
4
5
Perfectly
73
Q

A 54 year old with severe sudden onset pain mainly felt between the scapulae.

What is the most likely diagnosis?
A. GORD
B. Pericardial effusion
C. Stable angina
D. Pulmonary embolism
E. Unstable angina
F. Pleural effusion
G. Myocardial infarction
H. Pericarditis
I. Pneumonia
J. Pneumothorax
K. Emphysema
L.Peptic ulcer
M. Fractured rib
N. Asthma
O. Ventricular fibrillation
P. Aortic dissection
A

P. Aortic dissection

The tearing chest pain suggests aortic dissection. There may also be interscapular pain with dissection of the descending aorta. Dissecting aneurysms are either type A, which involves the ascending aorta, or type B. Type A dissections require urgent surgery whereas type B can be managed medically if it is not complicated by end organ ischaemia. BP differential between the 2 arms is a hallmark feature. Pulse differences may also be present in the lower limbs. There may also be the diastolic murmur of AR in proximal dissections. A CT scan is indicated as soon as a diagnosis of aortic dissection is suspected and should be from the chest to the pelvis to see the full extent of the dissecting aneurysm. What you will see is the intimal flap. MRI is more sensitive and specific but is more difficult to obtain acutely.

How well did you know this?
1
Not at all
2
3
4
5
Perfectly
74
Q

Progressive retrosternal pain in a 45 year old patient known to have a hiatus hernia.

What is the most likely diagnosis?
A. GORD
B. Pericardial effusion
C. Stable angina
D. Pulmonary embolism
E. Unstable angina
F. Pleural effusion
G. Myocardial infarction
H. Pericarditis
I. Pneumonia
J. Pneumothorax
K. Emphysema
L.Peptic ulcer
M. Fractured rib
N. Asthma
O. Ventricular fibrillation
P. Aortic dissection
A

A. GORD

This patient has GORD characterised by heartburn and regurgitation of acid. It is more severe at night when the patient is lying flat and also when the patient is bending over. Risk factors include obesity and hiatus hernia. Diagnosis is generally clinical and can also be achieved by a diagnostic trial of a PPI. Normally an upper GI endoscopy is reserved for complications such as strictures, Barrett’s or cancer, or for atypical features. An OGD may show oesophagitis or Barrett’s (red velvety), however OGD may be normal. Manometry and pH monitoring may also be performed, but in this case, this patient will probably just have a therapeutic and diagnostic trial of a PPI instead of an OGD.

How well did you know this?
1
Not at all
2
3
4
5
Perfectly
75
Q

A 26 year old professional basketball player presents with central chest pain and breathlessness, with the pain worse on inspiration.

What is the most likely diagnosis?
A. GORD
B. Pericardial effusion
C. Stable angina
D. Pulmonary embolism
E. Unstable angina
F. Pleural effusion
G. Myocardial infarction
H. Pericarditis
I. Pneumonia
J. Pneumothorax
K. Emphysema
L.Peptic ulcer
M. Fractured rib
N. Asthma
O. Ventricular fibrillation
P. Aortic dissection
A

J. Pneumothorax

This patient has developed a primary pneumothorax. Primary pneumothoraces occur in young people without known lung conditions. The main investigation is a CXR and pneumothoraces are classified by the BTS as large (>2cm visible rim between the lung margin and the chest wall) or small (<2cm). If the patient is clinically stable, they can be observed and given oxygen – an invasive approach is not necessary and the oxygen will increase the rate of pneumothorax reabsorption. A pneumothorax, if left, is something that resolves by itself over time by reabsorption. Cavemen (and cavewomen) who got pneumothoraces all those years ago didn’t just walk around breathless until they died. If large however, percutaneous needle aspiration is required (IV cannula 2nd intercostal space, or 3rd, at the MCL). A CXR should be obtained after this procedure. If this fails, a chest drain should be inserted.

How well did you know this?
1
Not at all
2
3
4
5
Perfectly
76
Q

A 22 year old man walks into A&E, he was involved in a fight in a bar. As he left the bar he complained of drowsiness and double vision worst when walked down stairs.

Which nerve is most likely to have been affected?
A. Cranial nerve Vb
B. Cranial nerve XI
C. Optic tract
D. Cerebellum
E. Cranial nerve IV
F. Cranial nerve XII
G. Cranial nerve IX
H. Cranial nerve II
I. Cranial nerve Va
J. Cranial nerve VIII
K. Cranial nerve Vc
L. Cranial nerve III
M. Cranial nerve I
N. Cranial nerve X
O. Cranial nerve VI
P. Cranial nerve VII
A

E. Cranial nerve IV

This man is complaining of double vision. You should already be thinking III, IV or VI which are the cranial nerves responsible for eye movements. CN IV innervates the superior oblique muscle which controls depression, intorsion and adduction of the eye. This is vertical diplopia (worse when walking down the stairs) hence the cause is a lesion in CN IV. The trochlear nucleus is located in the midbrain at the level of the inferior colliculus and fasciles decussate at the medullary velum to exit at the dorsal midbrain. The nerve enters the orbit through the superior orbital fissure. Testing is done at the bedside by asking the patient to follow the examiner’s finger with the eyes, while observing eye movements and asking the patient to report any diplopia. Remember that diplopia is maximal in the direction of action of the paralysed muscle (which makes sense if you think about it) and the outer image is the image which is false, and disappears when the ipsilateral eye is covered up.

How well did you know this?
1
Not at all
2
3
4
5
Perfectly
77
Q

Shortly after recovering from thyroid surgery, Mrs Sugav complains of voice loss and a dry persistent cough.

Which nerve is most likely to have been affected?
A. Cranial nerve Vb
B. Cranial nerve XI
C. Optic tract
D. Cerebellum
E. Cranial nerve IV
F. Cranial nerve XII
G. Cranial nerve IX
H. Cranial nerve II
I. Cranial nerve Va
J. Cranial nerve VIII
K. Cranial nerve Vc
L. Cranial nerve III
M. Cranial nerve I
N. Cranial nerve X
O. Cranial nerve VI
P. Cranial nerve VII
A

N. Cranial nerve X

The vagus nerve contains both visceral efferents and afferents and has 3 main motor branches. It innervates muscles of the larynx and pharynx, with the exception of stylopharyngeus and tensor veli palatini. It also mediates sensory input from these areas as well as the external auditory canal. It also conveys visceral afferent input from the thoracic and abdominal viscera and supplies parasympathetic output to these regions as well. The nerve has been damaged with thyroid surgery and there is voice loss and a cough as a response. Signs of CN X damage include palatal droop, deviation of the uvula contralaterally and an absent gag reflex, as the efferent limb of this reflex is formed by the vagus nerve.

How well did you know this?
1
Not at all
2
3
4
5
Perfectly
78
Q

A patient is diagnosed with bulbar palsy. On examination, his tongue was deviated to one side.

Which nerve is most likely to have been affected?
A. Cranial nerve Vb
B. Cranial nerve XI
C. Optic tract
D. Cerebellum
E. Cranial nerve IV
G. Cranial nerve IX
H. Cranial nerve II
I. Cranial nerve Va
J. Cranial nerve VIII
K. Cranial nerve Vc
L. Cranial nerve III
M. Cranial nerve I
N. Cranial nerve X
O. Cranial nerve VI
P. Cranial nerve VII
A

F. Cranial nerve XII

Bulbar palsy would give progressive symptoms such as difficulty swallowing, chewing or talking and muscle weakness. Tongue wasting, weakness and fasciculations may also be seen and an absent gag reflex may be found. The examination here shows tongue deviation in line with a CN XII lesion. The tongue deviates ipsilaterally to the lesion. There may also be tongue weakness, fasciculation at rest, flaccidity, atrophy or the inability to move the tongue rapidly from side to side or up and down. The 12th cranial nerve is purely motor in function, moving and altering the shape of the tongue by innervating the intrinsic and extrinsic tongue muscles.

How well did you know this?
1
Not at all
2
3
4
5
Perfectly
79
Q

A 79 year old lady was brought to A&E after a fall. On examination, when asked to close her eyes, her eyeballs rolled upwards and she was unable to close her eyes.

Which nerve is most likely to have been affected?
A. Cranial nerve Vb
B. Cranial nerve XI
C. Optic tract
D. Cerebellum
E. Cranial nerve IV
F. Cranial nerve XII
G. Cranial nerve IX
H. Cranial nerve II
I. Cranial nerve Va
J. Cranial nerve VIII
K. Cranial nerve Vc
L. Cranial nerve III
M. Cranial nerve I
N. Cranial nerve X
O. Cranial nerve VI
P. Cranial nerve VII
A

P. Cranial nerve VII

This is someone who cannot close their eyes. Eye closure relies on orbicularis oculi which is innervated by the facial nerve (VII). It is the most common CN mononeuropathy and the most common cause of a peripheral facial palsy is Bell’s palsy. This is why when testing CN VII you ask a patient to screw their eyes up tightly and resist you trying to open them as well as asking them to do other seemingly odd things like raising their eyebrows against resistance, showing you their teeth and pursing their lips. The facial nerve is both motor and sensory (via the nervus intermedius).The nerve exits the cranium through the stylomastoid foramen and enters the parotid, where it splits into 5 branches – temporal, zygomatic, buccal, mandibular and cervical.

How well did you know this?
1
Not at all
2
3
4
5
Perfectly
80
Q

A middle aged lady brought into A&E after a RTA, on examination was found unable to discriminate between smells. Her nasal passages were found to be clear.

Which nerve is most likely to have been affected?
A. Cranial nerve Vb
B. Cranial nerve XI
C. Optic tract
D. Cerebellum
E. Cranial nerve IV
F. Cranial nerve XII
G. Cranial nerve IX
H. Cranial nerve II
I. Cranial nerve Va
J. Cranial nerve VIII
K. Cranial nerve Vc
L. Cranial nerve III
M. Cranial nerve I
N. Cranial nerve X
O. Cranial nerve VI
P. Cranial nerve VII
A

M. Cranial nerve I

This woman has dysnosmia (distortions of smell) caused by dysfunction of cranial nerve I, the olfactory nerve. Anosmia refers to the complete loss of smell and hyposmia refers to the partial loss of smell. This is a clinical diagnosis using odour identification tests where patients are asked to identify pre-defined smells. Specialist centres can do olfactory evoked potentials. The odorants are transported to bipolar cells located in the olfactory neuroepithelium and these cells synapse with olfactory bulb glomeruli. The paired olfactory bulbs are located at the base of the frontal lobe overlying the cribriform plate, and during trauma like a road traffic accident, the connections at the cribriform plate can be broken.

How well did you know this?
1
Not at all
2
3
4
5
Perfectly
81
Q

A 40 year old gentleman comes to A&E, with severe pain in his right loin, radiating to the groin. His PMH includes several episodes of UTIs.

What is the most likely diagnosis?
A. Rheumatoid arthritis
B. Nephrotic syndrome
C. Prostate carcinoma
D. Polycystic kidneys
E. Bladder stones
F. Bladder carcinoma
G. Ureteric stones
H. Glomerulonephritis
I. Appendicitis
J. SLE
K. UTI
L. Urinary tract obstruction
M. Renal cell carcinoma
N. Gout
O. Acute tubular necrosis
A

G. Ureteric stones

This patient has renal colic which classically presents with severe flank pain radiating to the groin. Infection may complicate renal calculi. Microscopic haematuria is present in up to 90% of cases. Up to 85% of stones are visible on a plain KUB although urate stones are radiolucent. If the stone is radio-opaque, calcification will be seen within the urinary tract. In pregnancy though, a renal USS is first line. The IVP has now been replaced by the CT scan which is the new diagnostic standard. A non-contrast helical (or spiral) CT is preferred due to high sensitivity and specificity and acurately determines presence, site and size of stones. Stones are analysed after they are extracted or when they are expelled to check their composition.

How well did you know this?
1
Not at all
2
3
4
5
Perfectly
82
Q

A 70 year old gentleman presents with increased urine frequency and nocturia up to 5 times/night. On direct questioning, it is also noted that he also has urinary hesitancy and terminal dribbling, and that he thinks he has recently lost weight.

What is the most likely diagnosis?
A. Rheumatoid arthritis
B. Nephrotic syndrome
D. Polycystic kidneys
E. Bladder stones
F. Bladder carcinoma
G. Ureteric stones
H. Glomerulonephritis
I. Appendicitis
J. SLE
K. UTI
L. Urinary tract obstruction
M. Renal cell carcinoma
N. Gout
O. Acute tubular necrosis
A

C. Prostate carcinoma

This patient has symptoms of urinary tract outfow obstruction – namely nocturia, frequency and hesitancy. Haematuria and dysuria may be complaints in late-stage disease. Prostate cancer is uncommon in men under 50. Weight loss and other systemic symptoms of disease such as bone pain and lethargy is associated with advanced metastatic disease and is otherwise uncommonly seen. Not all cases of malignancy classically features weight loss. Management of this condition is either expectant or definitive depending on life expectancy. Key investigations are PSA and DRE and diagnosis is confirmed on a trans-rectal ultrasound-guided needle biopsy. The normal prostate is about the size of a walnut and during examination, the size, consistency, symmetry and the presence of any nodules should be noted. A PSA normally ranges from 0-4 microgams/L however this varies with age and race. Prostate cancer is graded on the Gleason score (2-10), which is the sum total of two scales from 1 to 5 which is based on how differentiated the tumour’s predominant cell lines are. It is 95% of the time an adenocarcinoma as it is a malignant growth orginating from the glandular cells of the prostate.

How well did you know this?
1
Not at all
2
3
4
5
Perfectly
83
Q

A 35 year old lady presents with severe fatigue, malaise, nausea and joint pains worsening over the past few months. She has no other significant PMH apart from having had 3 first trimester miscarriages, She has a Hb of 7g/dl and creatinine of 280mmol/l.

What is the most likely diagnosis?
A. Rheumatoid arthritis
B. Nephrotic syndrome
C. Prostate carcinoma
D. Polycystic kidneys
E. Bladder stones
F. Bladder carcinoma
G. Ureteric stones
H. Glomerulonephritis
I. Appendicitis
J. SLE
K. UTI
L. Urinary tract obstruction
M. Renal cell carcinoma
N. Gout
O. Acute tubular necrosis
A

J. SLE

This lady has systemic lupus erythematosus. SLE is a multi-system generalised disorder most commonly affecting women of reproductive age. This patient has associated antiphospholipid syndrome with a history of miscarriages and antiphospholipid antibiodies would be expected to be positive here. Lupus patients also commonly have renal manifestations and hence show elevated urea and creatinine. Anaemia may well be autoimmune here, with Coombs test being positive. SLE is characterised by the presence of ANA and most frequently involves the skin and joints. Most present with tiredness, the typical rash, which is not present here, and/or musculoskeletal symptoms like joint pain. Arthralgia is common and can be similar to RA but is non-erosive. The classic rash of lupus is the malar (butterfly) rash which presents as erythema over the cheeks and bridge of the nose, sparing the nasolabial folds. A photosensitive rash is classic, which occurs after sun exposure and there may also be a discoid rash which presents as erythematous raised patches with keratotic scaling. Symptoms and signs may accumulate over time.

How well did you know this?
1
Not at all
2
3
4
5
Perfectly
84
Q

An 80 year old woman who has been bed bound in hospital for a few months has become increasingly confused over the last two days and complains of abdominal tenderness.

What is the most likely diagnosis?
A. Rheumatoid arthritis
B. Nephrotic syndrome
C. Prostate carcinoma
D. Polycystic kidneys
E. Bladder stones
F. Bladder carcinoma
G. Ureteric stones
H. Glomerulonephritis
I. Appendicitis
J. SLE
L. Urinary tract obstruction
M. Renal cell carcinoma
N. Gout
O. Acute tubular necrosis
A

K. UTI

Confusion is not an uncommon presentation of UTI in the elderly. 10% of women >70 have a UTI. This is the cause of this elderly lady’s confusion. Her set of symptoms (confusion and abdominal tenderness) are not explained by any of the other conditions given on the list. A UTI, in uncomplicated cases is most commonly caused by E coli. However, this case could be a hospital acquired infection due to, for example, catheterisation and organisms such as Proteus and Pseudomonas are all possible. It is diagnosed with a dipstick and urine MC+S from an MSU sample. Have a think about what the dipstick would show. Antibiotic therapy should be guided by local sensitivities and guidelines, or MC+S results. Nitrofurantoin or co-trimoxazole could be used.

How well did you know this?
1
Not at all
2
3
4
5
Perfectly
85
Q

A 30 year old man was admitted for a severe infection a few days ago. His BP has now dropped, he has become tachycardic and his urine output has decreased. Blood test discloses a creatinine level of 800.

What is the most likely diagnosis?
A. Rheumatoid arthritis
B. Nephrotic syndrome
C. Prostate carcinoma
D. Polycystic kidneys
E. Bladder stones
F. Bladder carcinoma
G. Ureteric stones
H. Glomerulonephritis
I. Appendicitis
J. SLE
K. UTI
L. Urinary tract obstruction
M. Renal cell carcinoma
N. Gout
O. Acute tubular necrosis
A

O. Acute tubular necrosis

Acute tubular necrosis is caused by either ischaemia or toxic injury to tubular epithelial cells in the kidney, which causes cell death or detachment from the BM. A history of low BP, fluid depeletion or toxin exposure is usually present. This patient has low BP and is fluid depleted, with compensatory tachycardia. This hypovolaemic state has resulted in ischaemic renal injury, which should have a good outcome once the hypovolaemia is corrected here with IV saline. The treatment is supportive.

How well did you know this?
1
Not at all
2
3
4
5
Perfectly
86
Q

A 73 year old woman is recovering in hospital 3 days after an operation to replace her right hip. On going to the toilet she suddenly becomes extremely short of breath and rapidly loses consciousness. Her BP is 60/20mmHg.

What is the most likely diagnosis?
A. Pancreatitis
B. Exacerbation of COPD
C. Pulmonary Embolus
D. Appendicitis
E. Viral pneumonia
F. Occulomotor (III) nerve palsy
G. Diabetic amyotrophy
H. GORD
I. Abducent (VI) nerve palsy
J. Pneumothorax
K. Biliary colic
L. Bowel obstruction
A

C. Pulmonary Embolus

This patient should be thrombolysed immediately if not contraindicated due to her haemodynamically unstable and critical state. Treatment should not be delayed in this obvious PE. This patient is now at a serious risk of cardiac arrest. Anticoagulation should also be started. The underlying pathophysiology is based on Virchow’s triad. SOB is a common symptom and there may also be chest pain and haemoptysis. This patient has had recent surgery, particularly orthopaedic surgery, which is a strong risk factor for PE. Other strong risk factors include DVT, obesity, prolonged bed rest, malignancy, previous VTE, pregnancy and the thrombophilias such as factor V Leiden. ECG may be normal, or may show tachycardia, new RAD, new RBBB or the classical S wave in I, Q wave with T inversion in III. Various clinical probability scores exist for PE and D-dimer can be used to exclude PE as a diagnosis.

The study of choice is a CTPA with direct visualisation of the thrombus. If there is a contraindication to a CT scan such as contrast allergy or pregnancy, then a V/Q scan is indicated. If a V/Q scan is not possible, alternatives such as MRA can be requested. It is worth noting that in patients with cardiopulmonary disease, these tests may not be accurate. A TTE can also be used to detect RV strain seen with PE.

How well did you know this?
1
Not at all
2
3
4
5
Perfectly
87
Q

A 77-year old retired coal miner has a 30-year history of cough, mostly productive of sputum. He suddenly becomes breathless after a bout of coughing and complains of right sided chest pain. On examination, he is cyanosed, the trachea is deviated to the left and no breath sounds are audible over part of the right side of the chest.

What is the most likely diagnosis?
A. Pancreatitis
B. Exacerbation of COPD
C. Pulmonary Embolus
D. Appendicitis
E. Viral pneumonia
F. Occulomotor (III) nerve palsy
G. Diabetic amyotrophy
H. GORD
I. Abducent (VI) nerve palsy
J. Pneumothorax
K. Biliary colic
L. Bowel obstruction
A

J. Pneumothorax

This patient has developed a right sided tension pneumothorax and will need emergency intervention in the form of the insertion of a large bore cannula into the 2nd intercostal space in the MCL of the affected side. This will need to be followed by the insertion of a chest drain.

Otherwise if this was not a tension pneumothorax, for secondary spontaneous pneumothoraces, if large enough for a chest drain or the patient is clinically unstable, chest drain insertion is indicated. Simple aspiration success rate is reduced in secondary spontaneous pneumothoraces. Primary pneumothoraces occur in young people without known lung conditions. This patient has pulmonary fibrosis. Those who suffer recurrent pneumothoraces may have to undergo pleurodesis to stick the parietal and visceral pleural together by an inflammatory reaction.

How well did you know this?
1
Not at all
2
3
4
5
Perfectly
88
Q

A 62 year old male diabetic presents with a sudden onset of double vision. He is noted to have ptosis and a deviation of the right eye down and to the right. The pupils appear equal in size and are reactive to light.

What is the most likely diagnosis?
A. Pancreatitis
B. Exacerbation of COPD
C. Pulmonary Embolus
D. Appendicitis
E. Viral pneumonia
F. Occulomotor (III) nerve palsy
G. Diabetic amyotrophy
H. GORD
I. Abducent (VI) nerve palsy
J. Pneumothorax
K. Biliary colic
L. Bowel obstruction
A

F. Occulomotor (III) nerve palsy

A complete (or surgical) third nerve palsy presents with compete ptosis, a dilated pupil and the eye turned down and out and can be caused by a PCA aneurysm. This is an emergency and the patient needs a neurosurgical opinion with MRI/angiography. A partial pupil-sparing (or medical) third nerve palsy presents with partial ptosis without pupillary signs and can be caused by infarction of the nerve i.e. mononeuritis multiplex, due to possible DM or vasculitis like GCA.

Remember from anatomy that CNIII innvervates all the mucles of the eye except SO and LR. CNIII also supplies levator palpebrae superioris and the sphincter pupillae which causes miosis. If the cause is surgical and compressive, the parasympathetic nerve is affected early on, as it lies at the surface of the third nerve. In an infarctive medical third nerve palsy, the centre infarcts, which is affected more than the surface of the nerve. Hence, the pupil is spared and there is partial ptosis.

How well did you know this?
1
Not at all
2
3
4
5
Perfectly
89
Q

A 4 year old boy, profuse vomiting, no motions for 2 days, severe abdominal pain which is intermittent in nature. Abdomen distended with increased bowel sounds – high pitched in nature.

What is the most likely diagnosis?
A. Pancreatitis
B. Exacerbation of COPD
C. Pulmonary Embolus
D. Appendicitis
E. Viral pneumonia
F. Occulomotor (III) nerve palsy
G. Diabetic amyotrophy
H. GORD
I. Abducent (VI) nerve palsy
J. Pneumothorax
K. Biliary colic
L. Bowel obstruction
A

L. Bowel obstruction

Bowel obstruction has several causes such as adhesions or cancer in the older patient. In children common causes of small bowel obstruction include appendicitis, intussusception, intestinal atresia and volvulus and these should be kept in mind in this case. The profuse vomiting, distended and painful abdomen and tinkling high pitched bowel signs are all indicative. The proximal segment of bowel dilates and distal bowel collapses. Completely obstructed patients generally require surgery. If, on AXR, air is seen to be seeping past the obstruction then the obstruction is partial. As a standard, all patients should be made NBM and given supplemental oxygen, IV fluids and NG decompression (to reduce flow/gastric contents/air towards the obstruction), unless they are rushed off for an emergency laparotomy because, for example, they have complete SBO and are peritonitic.

How well did you know this?
1
Not at all
2
3
4
5
Perfectly
90
Q

A 30 year old banker came in with pain in the epigastric region which radiated to his back. The pain improves slightly on sitting forward. He says he can’t keep anything down. He is tachycardic, pyrexial, jaundiced, and has a rigid abdomen.

What is the most likely diagnosis?
A. Pancreatitis
B. Exacerbation of COPD
C. Pulmonary Embolus
D. Appendicitis
E. Viral pneumonia
F. Occulomotor (III) nerve palsy
G. Diabetic amyotrophy
H. GORD
I. Abducent (VI) nerve palsy
J. Pneumothorax
K. Biliary colic
L. Bowel obstruction
A

A. Pancreatitis

This patient has acute pancreatitis and is peritonitic. He has vomited (can’t keep anything down) and is describing mid-epigastric pain radiating around to the back, which in itself is almost diagnostic. Tachycardia here is as a result of hypovolaemia. Jaundice here is suggestive of gallstone aetiology with obstruction to the common bile duct, though pancreatic oedema can itself cause jaundice. Nausea and vomiting is not uncommon and can occur with agitation and confusion. Complicated haemorrhagic pancreatitis may exhibit Cullen’s sign, Grey-Turner’s sign and Fox’s sign. Make sure you know what these are and you are familiar with the causes of acute pancreatitis (GET SMASHED). Those caused by hypocalcaemia may also display Chvostek’s sign and Trousseau’s sign.

Key to diagnosis is serum amylase or lipase levels which are massively elevated. Prognostic criteria are outlined in Ranson’s criteria applied on admission and after 48 hours, or the modified Glasgow score which you can find in your Oxford Handbook. An abdominal CT is however the most sensitive and specific study and findings may include enlargement of the pancreas with irregular contours, necrosis, pseudocysts and peripancreatic fat obliteration.

For interest, urinary trypsinogen-2 is now considered a better screening test than amylase but is not currently clinically used.

91
Q

A 7 year old girl presents with at 39.3oC fever, rash and neck stiffness. You suspect raised intracranial pressure, though a CT scanner is not available to confirm this.

What is the next step in her management?

A. Transvaginal ultrasound
B. Lumbar puncture
C. AXR
D. ABGs
E. CXR
F. Urinary beta hCG
G. FBC
H. MRI Head
I. Blood cultures
J. PET scan
K. Laparoscopy
L. CT head
M. Helical CT scan
N. Compression USS
A

I. Blood cultures

You suspect that this little girl has meningitis with fever, nuchal rigidity and presumably a petechial/purpuric rash, typically associated with meningococcal meningitis. If there is evidence of raised intracranial pressure, then a CT head should be considered before LP. There is this suspicion in the question, but the radiographers have gone home. That’s not to say that if you suspect raised ICP and feel an LP would not be safe to do, that you should go ahead and perform a lumbar puncture anyway. In this case, as a responsible doctor, you would order blood cultures as LP is clearly clinically unsafe. This is less accurate as the result may be influenced by previous antimicrobials and positive blood cultures have been reported in only 40-70% of clinically suspected meningococcal infection. But, unless you want to get struck off by the GMC and imprisoned for causing uncal herniation… it’s the best choice you have. The main diagnostic investigation for meningitis is still to do is an LP except you simply cannot do it in this case. This would show low CSF glucose, raised CSF protein and positive CSF culture or gram stain or meningococcal antigen. Given the signs and symptoms here are very striking, you would not delay treatment. Until you know which organism is causing the symptoms and its sensitivities, empirical therapy can be started with, for example, vancomycin and ceftriaxone. Once the organism is confirmed and you know sensitivities, you can begin targeted antibiotic therapy. For example, if the infection is confirmed to be Neisseria and is penicillin sensitive, you can start benzylpenicillin or ampicillin intravenously. All patients should receive adequate supportive care throughout.

92
Q

From the same flight an over weight 33 year old female presents with increasing pain in her left leg. What is the first choice of investigation?

A. Transvaginal ultrasound
B. Lumbar puncture
C. AXR
D. ABGs
E. CXR
F. Urinary beta hCG
G. FBC
H. MRI Head
I. Blood cultures
J. PET scan
K. Laparoscopy
L. CT head
M. Helical CT scan
N. Compression USS
A

N. Compression USS

This is a DVT as I am sure you have already figured out.The recent presumably long-haul air travel is a strong risk factor though the exact mechanism of this risk is currently unclear, but may involve pressure changes, dehydration and mobility. She is also obese which is a big risk. Other strong risk factors include recent surgery, especially orthopaedic surgery, active malignancy, pregnancy and coagulopathies such as factor V Leiden. A Wells score is determined in all patients with a suspected DVT with the condition being likely if the score is 2 or more. If the Wells score is <2 then a D-dimer level is indicated. A compression ultrasound of the proximal deep venous system is the first line investigation in those where there is a high clinical suspicion. However, Doppler venous flow studies can be used when all other tests are unavailable although this test has a low sensitivity and requires a trained technician. A low flow in the veins is indicative of a DVT.

93
Q

A 40 year old man presents with acute colicky pain in his back radiating to his left flank. He has dysuria. What is the best diagnostic test?

A. Transvaginal ultrasound
B. Lumbar puncture
C. AXR
D. ABGs
E. CXR
F. Urinary beta hCG
G. FBC
H. MRI Head
I. Blood cultures
J. PET scan
K. Laparoscopy
L. CT head
M. Helical CT scan
N. Compression USS
A

M. Helical CT scan

This patient has renal colic which classically presents with severe flank pain radiating to the groin. Microscopic haematuria is present in up to 90% of cases. Up to 85% of stones are visible on a plain KUB although urate stones are radiolucent. If the stone is radio-opaque, calcification will be seen within the urinary tract. In pregnancy, a renal USS is first line. The IVP has now been replaced by the CT scan which is the new diagnostic standard. A non-contrast helical (or spiral) CT is preferred due to high sensitivity and specificity and acurately determines presence, site and size of stones. Stones are analysed after they are extracted or when they are expelled to check their composition. It is worth noting that in all females of child bearing age, a urine pregnancy test is necessary to exclude an ectopic pregnancy.

94
Q

30 year old female with previous history of ectopic pregnancy presents with right iliac fossa pain and some vaginal bleeding. What is the first line investigation?

A. Transvaginal ultrasound
B. Lumbar puncture
C. AXR
D. ABGs
E. CXR
F. Urinary beta hCG
G. FBC
H. MRI Head
I. Blood cultures
J. PET scan
K. Laparoscopy
L. CT head
M. Helical CT scan
N. Compression USS
A

F. Urinary beta hCG

This is a pregnancy test – urinary hCG testing, which is the first line investigation. Serum hCG testing can also be used. This is likely to be another ectopic pregnancy. Risk factors here are previous ectopic pregnancy, pain in the RIF and PV bleeding. Once pregnancy is confirmed, then the next step is to perform a high-resolution transvaginal ultrasound scan (TVUS) to determine where the pregnancy is located. Transabdominal is less sensitive but can be used. Many cases will however be PUL, or pregnancy of unknown location, whereby ultrasound cannot see anything. However, ultrasound is very much operator skill dependent. Treatment is either expectant, medical with methotrexate or surgical with for example, salpingectomy or salpingostomy. A complication of ectopic pregnancy is rupture, which may present in shock from blood loss with unusual patterns of referred pain from blood in the peritoneal cavity.

95
Q

30 year old vegan presents with a history of looking increasingly pale. On examination there is glossitis, jaundice and splenomegaly. There is also some loss of proprioception, impaired vibration sense and paraethesia in the limbs. She also has thyroid disease and SLE.

What is the most likely diagnosis?
A. B12 deficiency
B. Iron deficiency
C. Acute myeloblastic leukaemia
D. Hodgkin's lymphoma
E. Folate deficiency
F. Infectious mononucleosis
G. Acute lymphoblastic leukaemia
H. Multiple myeloma
I. Hepatocellular carcinoma
A

A. B12 deficiency

The possible causes in this patient of B12 deficiency is decreased dietary intaken due to the vegan diet. Pallor and glossitis are late signs but this patient is looking pale and there is also glossitis. The patient here is clearly anaemic. There is loss of proprioception and vibration indicating degeneration of the dorsal column-medial lemniscus pathway. The patient may also have a positive Romberg as a result. Parasthesias are also common in B12 deficiency indicating peripheral neuropathy. It is important to remember that hepatic B12 stores last for many years so B12 deficiency depends on chronic deficiency over a long period of time. In general, deficiency is caused by decreased intake, decreased gastric breakdown from food or malabsorption from the GIT. Serum B12 is a useful investigation to initially conduct and will guide further investigations and treatment.

96
Q

18 year old girl. Feeling tired all the time, looks pale. Has heavy periods. Nails look spoon-shaped.

What is the most likely diagnosis?
A. B12 deficiency
B. Iron deficiency
C. Acute myeloblastic leukaemia
D. Hodgkin's lymphoma
E. Folate deficiency
F. Infectious mononucleosis
G. Acute lymphoblastic leukaemia
H. Multiple myeloma
I. Hepatocellular carcinoma
A

B. Iron deficiency

This very concise description points to IDA. IDA is a microcytic hypochromic anaemia characterised by low serum iron, high TIBC and low transferrin saturation and serum ferritin. There is spooning of the nails (koilonychia) which is a sign. Nail chances begin with thinning and then flattening before spooning occurs. Pallor is also a sign of anaemia. Patients can often complain of faitgue, cravings, hair loss and there may also be glossitis and angular stomatitis. Initial treatment is with oral iron. The cause here is most likely bleeding from heavy periods in this case though further investigations should be done if a cause is not obviously present. Bleeding is the principle cause of IDA. IDA is not an end diagnosis and has many causes and this is something that is important to bear in mind at all times. Worldwide, the most common cause is hookworm infection, and in the UK, menstrual losses in women. Causes of IDA can be broadly divided into 4 categories: decreased intake, increased loss, increased requirements (such as in pregnancy), and unknown.

97
Q

74 year old male who has chronic myeloid leukaemia. Recent examination reveals anaemia, neutropenia, thrombocytopenia as well as hepatosplenomegaly and lymphadenopathy. Trephine biopsy shows Auer rods.

What is the most likely diagnosis?
A. B12 deficiency
B. Iron deficiency
C. Acute myeloblastic leukaemia
D. Hodgkin's lymphoma
E. Folate deficiency
F. Infectious mononucleosis
G. Acute lymphoblastic leukaemia
H. Multiple myeloma
I. Hepatocellular carcinoma
A

C. Acute myeloblastic leukaemia

CML may transform into AML or ALL in what is called a ‘blast crisis’. AML predominantly affects older adults with characteristically, abnormal blasts present in peripheral blood and normal haematopoiesis reduced. AML is basically the clonal expansion of myeloid blast cells in the bone marrow, peripheral blood or extramedullary tissue. Bone marrow blasts of at least 20% are diagnostic, obtained from BM biopsy. This patient has the features of cytopenia with anaemia, neutropenia and thrombocytopenia. BM trephine biopsy yielding myeloid blasts with Auer rods is pathognomic. Auer rods are needle shaped granular material found in the cytoplasmic matrix. Hepatosplenomegaly is also a feature which may be present.

98
Q

68 year old male. Complains of fever, night sweats and weight loss. On examination has hepatosplenomegaly and peripheral lymphadenopathy. On further investigation, he was also found to have intra-abdominal and pelvic lymphadenopathy.

What is the most likely diagnosis?
A. B12 deficiency
B. Iron deficiency
C. Acute myeloblastic leukaemia
D. Hodgkin's lymphoma
E. Folate deficiency
F. Infectious mononucleosis
G. Acute lymphoblastic leukaemia
H. Multiple myeloma
I. Hepatocellular carcinoma
A

D. Hodgkin’s lymphoma

This is a lymphoma and the main differential is between Hodgkin’s and Non-Hodgkin’s. It is actually very difficult to differentiate the two clinically but there is only one option here and the question is testing your ability to tell that is is lymphoma, at this stage. Possible differentiating signs or symptoms include the bimodal age distribution of Hodgkin’s and that Hodgkin’s is associated with pruritis and alcohol-triggered pain. But again, the two are clinically similar. Lymphoma may like this case present with systemic symptoms of fever, drenching night sweats (which classically require patients to change their clothes) and weight loss due to the hypermetabolic state. There may also be SOB, cough, general malaise and abdominal discomfort (hepatomegaly, splenomegaly, lymphadenopathy, bowel involvement), headache, change in mental status, ataxia (if the CNS is involved), fatigue and chest pain (from anaemia), bone pain (from bone involvement) etc. There are many risk factors such as EBV, which is associated with Hodgkin’s lymphoma, Burkitt’s lymphoma, AIDS-related primary CNS lymphoma and nasal NK/T-cell lymphoma. Treatment depends on the histological subtype and the severity of symptoms at presentation.

99
Q

70 year old male. On examination he has non-tender lymphadenopathy. Lymph node biopsy shows Strenberg-Reed cells.

What is the most likely diagnosis?
A. B12 deficiency
B. Iron deficiency
C. Acute myeloblastic leukaemia
D. Hodgkin's lymphoma
E. Folate deficiency
F. Infectious mononucleosis
G. Acute lymphoblastic leukaemia
H. Multiple myeloma
I. Hepatocellular carcinoma
A

D. Hodgkin’s lymphoma

This is again a case of lymphoma. Reed-Sternberg cells are binucleate cells characteristically seen in Hodgkin’s lymphoma. Hodgkin’s is typically localised to a single group of nodes (normally the cervical and/or supraclavicular) and extranodal involvement is rare. Mediastinal involvement is common. Spread is contiguous and B symptoms may be present such as a low grade fever, weight loss and night sweats. Pruritis may be found in approximately 10% of cases but has no prognostic significance. 50% of cases is associated with EBV infection and distribution is bimodal with peaks in young and old. EBV antigens are found in 20-40% of Reed-Sternberg cells and there is a strong association with a history of EBV infection. There is classically pain in lymph nodes on alcohol consumption.

100
Q

An 8 year old boy presents with haematuria, frothy urine and oliguria. On investigation, proteinuria, and red cell casts in the urine are confirmed. He has recently recovered from Streptococcal tonsillitis.

What is the most likely diagnosis?
A. Nephrogenic Diabetes Inspidus
B. Fanconi Syndrome
C. Acute Nephritic Syndrome
D. Acute Pyelonephritis
E. Nephrocalcinosis
F. Alport’s syndrome
G. Chronic Pyelonephritis
H. Nephrotic Syndrome
I. Goodpasture’s syndrome
J. Urinary tract Obstruction
A

C. Acute Nephritic Syndrome

Nephritic syndrome is typically defined by acute kidney injury, hypertension and an active urinary sediment (RBCs and RBC casts). This is post-infectious glomerulonephritis caused by group A beta-haemolytic streptococcus with renal endothelial cell damage. Serological markers would expect to show antibodies to streptococcus and low complement and treatment here is with antibiotics.

As a note, causes of nephritic syndrome (with sub-nephrotic range proteinuria) include: IgA nephropathy, post-infectious GN and rapidly progressive GN. Nephrotic syndrome causes include minimal change disease, focal and segmental glomerulosclerosis, membranous nephropathy, deposition diseases and membranoproliferative GN. This will be covered in more detail in the explanation to question 3.

101
Q

A teenage boy presents with haematuria. It has been noted that his hearing has been worsening over recent years. He has a family history of early death.

What is the most likely diagnosis?
A. Nephrogenic Diabetes Inspidus
B. Fanconi Syndrome
C. Acute Nephritic Syndrome
D. Acute Pyelonephritis
E. Nephrocalcinosis
F. Alport’s syndrome
G. Chronic Pyelonephritis
H. Nephrotic Syndrome
I. Goodpasture’s syndrome
J. Urinary tract Obstruction
A

F. Alport’s syndrome

Alport’s syndrome is a rare familial nephropathy due to abnormal type IV collagen and may either be X-linked, autosomal recessive or autosomal dominant. Gross haematuria is a common presenting feature, often precipitated in a child by an infection. The age of onset of chronic renal failure is variable in this condition and it is frequently associated with sensorineural hearing loss, like in this case here. Complications such as renal failure and deafness typically occur more often in young males with the X-linked form of disease. The FH of early death may be linked to progression to end-stage renal failure. The mainstay of treatment is to monitor and treat renal disease. Female carriers of X-linked mutations have a lifetime risk of developing hypertension and renal disease which is significant. In EMQs if a child, especially male, has renal disease associated with deafness, then it is likely to be Alport’s. There is also a characteristic central retinopathy present in 40-70% of males which is said to be pathognomic of the disease.

102
Q

A previously well 51 year old lady presents complaining of gradually increasing swelling of her legs. On examination, she is found to have pitting oedema in the ankles and lower legs. On investigation, significant hypoalbuminaemia, proteinuria and previously unnoticed hypercholesterolaemia are noted.

What is the most likely diagnosis?
A. Nephrogenic Diabetes Inspidus
B. Fanconi Syndrome
C. Acute Nephritic Syndrome
D. Acute Pyelonephritis
E. Nephrocalcinosis
F. Alport’s syndrome
G. Chronic Pyelonephritis
H. Nephrotic Syndrome
I. Goodpasture’s syndrome
J. Urinary tract Obstruction
A

H. Nephrotic Syndrome

This lady has developed nephrotic syndrome, which is defined by the presence of proteinuria (>3.5g/24 hours), hypoalbuminaemia (<30g/L) oedema, and hyperlipidaemia. This is a constellation of several symptoms which can have many causes. The likely cause differs depending on how old the patient is and whether the patient has co-morbidities like diabetes. The most common cause in children is minimal change disease, whereas the most common cause in young adults is focal segmental glomerulosclerosis. Membranous nephropathy is most common in older people and you should suspect diabetic nephropathy as a cause in those with long standing DM. Note also that those who present with proteinuria without the other components needed to define nephrotic syndrome are described as having ‘nephrotic range proteinuria’. In terms of physiology, the glomerular filtration barrier is disrupted by disease leading to glomerular proteinuria (the scale of which is linked to how severe the condition is and renal function). The loss of albumin leads to hypoalbuminaemia. The liver then tries to compensate for the protein loss by increases albumin synthesis, and this also increases synthesis of LDL, vLDL and lipoprotein A, leading to lipid abnormalities. The patient is also in a hypercoagulable state due to the loss of coagulatiion inhibitors in urine and increased synthesis of clotting factors in the liver. A 24-hour urine protein collection would establish proteinuria and whether it is in the nephrotic range. If this is impractical then the protein/creatinine ratio can be obtained.

103
Q

A young man who has recently returned from travelling abroad presents complaining of pain and tenderness in the abdomen. On examination, this appears to be most severe in the loin, and he is pyrexial. He was found to have bacteriuria.

What is the most likely diagnosis?
A. Nephrogenic Diabetes Inspidus
B. Fanconi Syndrome
C. Acute Nephritic Syndrome
D. Acute Pyelonephritis
E. Nephrocalcinosis
F. Alport’s syndrome
G. Chronic Pyelonephritis
H. Nephrotic Syndrome
I. Goodpasture’s syndrome
J. Urinary tract Obstruction
A

D. Acute Pyelonephritis

Acute onset fever, flank pain and positive urine dipstick all point to the diagnosis of acute pyelonephritis. Urinalysis is highly sensitive but not very specific. In uncomplicated pyelonephritis, the most common cause is E. coli and gram stain will typically reveal gram negative rods, either E. coli, Proteus or Klebsiella. Gram positive cocci that could be implicated include enterococci and staphylococci. It is worth noting that older patients can often also present non-specifically. Treatment should start before culture results are received to prevent the patient from deteriorating, with empirical antibiotics.

104
Q

A 33 year old man presents with increased frequency of micturition and dysuria. He thinks his urine has a more unpleasant odour than usual. In his past history, he mentions that he was diagnosed with “reflux” as a child. On examination, the pain is identified as being in the suprapubic region; he was found to be hypertensive, and haematuria was confirmed on investigation.

What is the most likely diagnosis?
A. Nephrogenic Diabetes Inspidus
B. Fanconi Syndrome
C. Acute Nephritic Syndrome
D. Acute Pyelonephritis
E. Nephrocalcinosis
F. Alport’s syndrome
G. Chronic Pyelonephritis
H. Nephrotic Syndrome
I. Goodpasture’s syndrome
J. Urinary tract Obstruction
A

G. Chronic pyelonephritis

Chronic pyelonephritis is most commonly caused by chronic vesicoureteric reflux which this man gives a PMH of. Other risk factors include a history of acute pyelonephritis, which can often develop into the chronic type if inadequately treated or if acute cases are recurrent, renal obstruction (which may be due to urinary tract abnormalities, BPH or stones like a staghorn calculus) and DM. The damage is irreversible, unfortunately, and there is no specific treatment, and this condition may result in ESRF. Diagnosis is clinical and confirmed with imaging studies. The history of unpleasant smelling urine (which may be cloudy) along with urinary symptoms (suggesting UTI) and suprapubic pain (though there may be flank/back pain and tenderness) is also indicative. The patient’s condition may be complicated here by urinary tract infections and associated symptoms.

105
Q

A 45 year old, with severe asthma that has required recent hospital admission, has a moon-like face & truncal obesity. He complains of easy bruising & thin skin.

What is the most likely diagnosis?
A. Disseminated intravascular coagulation (DIC)
B. Henoch-Schonlein syndrome
C. Thrombotic thrombocytopenic purpura (TTP)
D. Haemorrhagic fever
E. Amyloidosis
F. Sepsis
G. Paroxysmal nocturnal haemoglobinuria (PNH)
H. Haemophilia
I. Pre-eclampsia
J. Malignant hypertension
K. Iatrogenic
L. Scurvy
M. Idiopathic thrombocytopenic purpura (ITP)
N. Cushing's Disease
O. Ehlers- Danlos syndrome
A

K. Iatrogenic

This asthmatic is likely on steroids (inhaled, or oral). This can lead to iatrogenic Cushing’s syndrome with signs and symptoms of hypercortisolism like moon-like facies, truncal obesity, easy bruising and thin skin. Other signs and symptoms include osteoporosis, proximal myopathy, hypertension, DM, acne, low libido, buffalo hump (dorsocervical fat pads), striae, facial plethora and menstrual irregularities. A low dose dexamethasone suppresion test can be done, or a 24 hour urinary free cortisol collection to diagnose Cushing’s syndrome. Note that Cushing’s disease is caused by a pituitary adenoma secreting ACTH.

106
Q

A 35 year old woman presents with purpura, fever, haemolytic anaemia, microvascular thrombosis, renal failure & mild stroke. Blood film shows fragmented red cells & clotting screen is normal.

What is the most likely diagnosis?
A. Disseminated intravascular coagulation (DIC)
B. Henoch-Schonlein syndrome
C. Thrombotic thrombocytopenic purpura (TTP)
D. Haemorrhagic fever
E. Amyloidosis
F. Sepsis
G. Paroxysmal nocturnal haemoglobinuria (PNH)
H. Haemophilia
I. Pre-eclampsia
J. Malignant hypertension
K. Iatrogenic
L. Scurvy
M. Idiopathic thrombocytopenic purpura (ITP)
N. Cushing's Disease
O. Ehlers- Danlos syndrome
A

C. Thrombotic thrombocytopenic purpura (TTP)

The title really tells you what this condition is. TTP is a diagnosis you should consider in any patient with anaemia and thrombocytopenia as 95% of cases are fatal if left untreated. The symptoms are usually non-specific though before plasma exchange was available as a treatment option, most patients developed 5 characteristic features (a pentad) of: microangiopathic haemolytic anaemia, thrombocytopenia with purpura, acute renal insufficiency, neurological abnormalities and fever. It is rare however for all of these to be present now. Most patients present aged 30-50 and there is usually a non-specific prodrome followed on the components of the pentad. The peripheral blood film here shows evidence of microangiopathic HA with schistocutes being present. Even though not reported, thrombocytopenia would also be seen on this film. The mainstay of treatment, as hinted before, is plasma-exchange therapy.

The main complications are renal and neurological. The main differential is haemolytic uraemic syndrome (more commonly seen in children), and some experts believe it is impossible to distinguish the two conditions as they are on the same spectrum of pathology, though others argue they are distinct entities. Care should be taken when diagnosing HUS as it would be really bad to withhold plasma-exchange if the diagnosis is TTP.

107
Q

A 56 year old vagrant presents with perifollicular skin bleeding & prolonged bleeding time.

What is the most likely diagnosis?
A. Disseminated intravascular coagulation (DIC)
B. Henoch-Schonlein syndrome
C. Thrombotic thrombocytopenic purpura (TTP)
D. Haemorrhagic fever
E. Amyloidosis
F. Sepsis
G. Paroxysmal nocturnal haemoglobinuria (PNH)
H. Haemophilia
I. Pre-eclampsia
J. Malignant hypertension
K. Iatrogenic
L. Scurvy
M. Idiopathic thrombocytopenic purpura (ITP)
N. Cushing's Disease
O. Ehlers- Danlos syndrome
A

L. Scurvy

Scurvy is basically vitamin C deficiency and is now a rare disorder and really only affects people who are subject to famine or wartime displacement in places like Africa. Vitamin C is needed for collagen synthesis so lack of this vitamin leads to impaired synthesis of collagen which can lead to complications. These complications include bleeding complications like spontaneous petechiae and bruising, friable gingiva and loose teeth, bone pain and joint effusions. Complications include heart failure, encephalopathy and entrapment neuropathies. To aid diagnosis, serum, leukocyte and whole blood ascorbic acid levels can be measured. This condition can be fatal if it is left untreated but complete recovery is expected on going vitamin C replacement in the form of ascorbic acid.

108
Q

A 35 year old man has hyperextensible joints & elastic skin. He also has purpura, & pseudotumours over his knees & elbows.

What is the most likely diagnosis?
A. Disseminated intravascular coagulation (DIC)
B. Henoch-Schonlein syndrome
C. Thrombotic thrombocytopenic purpura (TTP)
D. Haemorrhagic fever
E. Amyloidosis
F. Sepsis
G. Paroxysmal nocturnal haemoglobinuria (PNH)
H. Haemophilia
I. Pre-eclampsia
J. Malignant hypertension
K. Iatrogenic
L. Scurvy
M. Idiopathic thrombocytopenic purpura (ITP)
N. Cushing's Disease
O. Ehlers- Danlos syndrome
A

O. Ehlers- Danlos syndrome

Ehlers-Danlos is the commonest of a group of genetic connective tissue disorders. There are 6 types of this condition in which the hypermobility type (III) is the most common. This hypermobile 35 year old should make you think of this diagnosis, which actually often goes unnoticed and most people just assume that everyone is as flexible as them and do not assume that this is abnormal. Most do not develop symptoms or only develop minor symptoms. Skin manifestations can also give a clue to this diagnosis and include soft and silky skin, semi-transparent dermis and hyperelasticity, with patients also commonly displaying easy bruising, scarring and poor wound healing. There may also be CV, GI and autonomic features and the patient may be marfanoid or have features of chronic pain syndrome. The diagnosis is clinical and there is no genetic test that can confirm the hypermobility type, although collagen gene testing for the classic and vascular types are available. Most live unaffected healthy lives, although the vascular type is associated with reduced lifespan due to susceptibility to rupture of arteries or organs.

109
Q

A 20 year old man presents with colicky abdominal pain, arthritis & polyarthritis following a chest infection. He also has purpura over his buttocks & legs

What is the most likely diagnosis?
A. Disseminated intravascular coagulation (DIC)
B. Henoch-Schonlein syndrome
C. Thrombotic thrombocytopenic purpura (TTP)
D. Haemorrhagic fever
E. Amyloidosis
F. Sepsis
G. Paroxysmal nocturnal haemoglobinuria (PNH)
H. Haemophilia
I. Pre-eclampsia
J. Malignant hypertension
K. Iatrogenic
L. Scurvy
M. Idiopathic thrombocytopenic purpura (ITP)
N. Cushing's Disease
O. Ehlers- Danlos syndrome
A

B. Henoch-Schonlein syndrome

Henoch-Schonlein purpura is the most common vasculitis in childhood and in all cases there is a rash of palpable purpura which are typically non-blanching. If there is no rash, then it is not HSP. They are normally 2-10mm in diameter and arethe extravasation of blood into the skin. They can occur anywhere on the body but are usually concentrated on the lower extremities. Half of all patients have abdominal pain and arthralgias are commonly present (found in about 80%) and often associated with oedema. The joints most often affected are the knees and ankles. About half will show signs of renal disease such as proteinuria or haematuria. Risk factors for this condition include being male, age 3-15 and history of prior UTI. Complications can occur and the most common cause of death is renal failure.

110
Q

An overweight 48 year old Asian taxi driver complains of tiredness and blurring of vision which interfere with his job. His symptoms have been present for a few months and been slowly becoming worse. He tells you his mother died of diabetes but reports no polyuria, polydipsia or weight loss.

What is the most likely diagnosis?
A. Addison’s
B. Hyperprolactinaemia
C. Cushings
D. Graves Disease
E. Diabetes insipidus
F. Acromegaly
G. Diabetes Mellitus Type 1
H. Simmonds disease
I. Diabetes mellitus type 2
J. Hypothyroidism
K. Phaeochromocytoma
A

I. Diabetes mellitus type 2

Blurred vision is a common presentating complaint in T2DM and is due to elevated glucose. Fatigue is also a common feature and both being slowly and progressively worsening points towards a disease process such as T2DM. The diagnosis is given further weight here by the FH. There is a significant genetic component in T2DM. Additionally he is overweight which appears to be a precipitating factor leading to the clinical expression of DM. Several studies have the mean BMI at the time of diagnosis at around 31, and obesity is actually an intrinsic part of type 2 diabetes. Urinary symptoms are not as common as EMQs lead you to believe and their absence does not exclude this diagnosis. T2DM may also present with erectile dysfunction, urinary tract or yeast infections, dry and itchy skin, paraesthesias, unexplained weight loss, polydipsia and polyuria. Onset is generally preceded by a period of insulin resistance and hyperinsulinaemia before high glucose becomes detectable. There is a high concurrent risk of obesity, dyslipidaemia and hypertension. Lifestyle changes and metformin are indicated in most patients. BP and lipid management are also needed to reduce macrovascular risk and improve prognosis. BP and blood sugars should be managed to reduce the risk of microvascular complications such as retinopathy and nephropathy.

111
Q

A 32 year old solicitor has been trying to conceive for over a year with no success. She presents with oligomennorhoea over the last two years and significant weight loss. Upon examination she has a fine tremor and a slightly enlarged neck.

What is the most likely diagnosis?
A. Addison’s
B. Hyperprolactinaemia
C. Cushings
D. Graves Disease
E. Diabetes insipidus
F. Acromegaly
G. Diabetes Mellitus Type 1
H. Simmonds disease
I. Diabetes mellitus type 2
J. Hypothyroidism
K. Phaeochromocytoma
A

D. Graves Disease

This patient has hyperthyroidism which can present with menstrual irregularities in women such as oligomenorrhoea. The weight loss is a key diagnostic factor too and a tremor, usually fine, may also be present like this case. The slightly enlarged neck is a goitre, which is diffuse in Graves’ disease and in combination with the other signs and symptoms here is strongly diagnostic, although it is worth noting that the goitre may be difficult to appreciate in the elderly (especially if also kyphotic). There is no reason why this presentation should specifically be Graves’ disease but it is the only hyperthyroid state on the list. In countries where iodine defiency is not an issue, it is also the most common cause of hyperthyroidism. Peripheral manifestations such as ophthalmopathy, pretibial myxoedema and hyperthyroid acropachy do not occur with other causes of hyperthyroidism so if these were to be present then the cause of hyperthyroidism would be Graves’. Treatment aims to normalise thyroid function and is achieved by radioactive iodine, antithyroid medications or with surgery. They are all effective and relatively safe options. Symptomatic therapy is given with beta blockers such as propranolol.

112
Q

A 25 year old female presents with tiredness and oligomennorhoea. She has noticed an increase in weight over the last year with a loss in appetite. Upon examination she has waxy skin and has noticed a loss of body hair.

What is the most likely diagnosis?
A. Addison’s
B. Hyperprolactinaemia
C. Cushings
D. Graves Disease
E. Diabetes insipidus
F. Acromegaly
G. Diabetes Mellitus Type 1
H. Simmonds disease
I. Diabetes mellitus type 2
J. Hypothyroidism
K. Phaeochromocytoma
A

J. Hypothyroidism

This woman is in a hypothyroid state. Worldwide, the most common cause is iodine deficiency. Other causes include Hashimoto’s or secondary and tertiary hypothyroidism. It can also result from viral de Quervain’s thyroiditis or postpartum thyroiditis. Symptoms include those mentioned as well as depression, bradycardia, sluggish reflexes, constipation, cold intolerance and muscle cramps. Diagnosis is based on measurement of TSH and thyroid hormones. Treatment is by replacement of T4 with or without T3 in combination. If the patient has normal T3 and T4 but mildly elevated TSH, this is described as subclinical hypothyroidism.

113
Q

A 42 year old male presents with impotence and has noticed a loss of facial hair. He has also been trying for a baby with his wife for the last year with no success and is concerned about his fertility.

What is the most likely diagnosis?
A. Addison’s
C. Cushings
D. Graves Disease
E. Diabetes insipidus
F. Acromegaly
G. Diabetes Mellitus Type 1
H. Simmonds disease
I. Diabetes mellitus type 2
J. Hypothyroidism
K. Phaeochromocytoma
A

B. Hyperprolactinaemia

Hyperprolactinaemia is the most common endocrine disorder of the HPA axis and is a condition of elevated serum prolactin. It is not in itself an end diagnosis and it is important to bear this in mind. Prolactin is a peptide secreted and synthesised by lactotrophs in the anterior pituitary gland. This man is infertile due to reduced sperm production and may be impotent too due to erectile dysfunction. He also has signs of hypogonadism with the hair growth being lost (and testicles may also be small). There is also commonly decreased libido and may be galactorrhoea, gynaecomastia and reduced bone density. It is easier to understand the symptoms once you appreciate the role of prolactin. In women, prolactin stimulates breast epithelial cell proliferation and induces the production opf milk, as well as promoting the formation and action of the corpus luteum and suppressing pulsatile GnRH release. In men it directly inhibits spermatogenesis and steroidogenesis with receptors found on Sertoli and Leydig cells in the testes. Symptoms in women hence include oligo/amenorrhoea, menstrual disturbances, hirsutism, vaginal dryness, acne and painful sex.

The patient should be evaluated to look for an underlying cause which could be hypothyroidism and other diseases like chronic renal failure and cirrhosis. Drugs may also be present with elevate prolactin. Of course, this could also be due to a tumour, which you would suspect particularly if the patient has symptoms of mass effect such as headaches, visual field defects and seizures. Note that Simmonds’ disease is panhypopituitarism which would include other features too.

114
Q

A 19 year old is concerned about her performance in forthcoming examinations due to her constant need to urinate. She says that she is always very thirsty and produces large amounts of very dilute urine. Her random blood glucose is 4.7 mM

What is the most likely diagnosis?
A. Addison’s
B. Hyperprolactinaemia
C. Cushings
D. Graves Disease
E. Diabetes insipidus
F. Acromegaly
G. Diabetes Mellitus Type 1
H. Simmonds disease
I. Diabetes mellitus type 2
J. Hypothyroidism
K. Phaeochromocytoma
A

E. Diabetes insipidus

This is diabetes insipidus where there is polydipsia, polyuria, and hypotonic urine in large quantities. The patient has normal blood glucose excluding DM as a cause. Central DI is due to defective synthesis or release of AVP and will hence respond to DDAVP (desmopressin). Nephrogenic DI, on the other hand, occurs due to renal insensitivity to AVP. Hence, it will not respond to DDAVP and trearment is with adequate fluids, salt restriction and thaizide diuretics (paradoxically). Risks for nephrogenic DI include lithium use, CKD and chronic hypercalcaemia or hypokalaemia.

115
Q

A 28 yr old amateur footballer presents with numbness of the anterolateral aspect of his L leg and dorsum of foot. In the history he reveals that he had a plaster cast on that leg removed yesterday. On examination you find foot drop and weakness of ankle eversion.

What is the most likely diagnosis?
A. TIA
B. Brain Abscess
C. L Frontal Meningioma
D. Motor Neurone Disease
E. L VIII nerve Neurofibroma (acoustic neuroma)
F. R Parietal Glioma
G. Lateral Lumbar disc protrusion
H. Common Peroneal Nerve palsy
I. Adult Hydrocephalus
J. Meningitis
A

H. Common Peroneal Nerve palsy

Peroneal neuropathy is the most common mononeuropathy of the lower extremity and usually presents as foot drop. The peroneal nerve, if you remember from year 2 anatomy, is derived from L4-S1 roots and these fibres travel through the lumbosacral plexus and eventually in the sciatic nerve. Within the sciatic nerve, the fibres of the common peroneal nerve run separate to the tibial fibres and near the fibular neck, this nerve divides to give terminal branches as well as superficial and deep peroneal nerves. Do you remember which muscles these nerves supply? The most common way this nerve is damaged is by injury at the neck of the fibula due to compression. For example, in this case with the plaster cast, but also surgical positioning, crossing legs or trauma. So if you think about what the peroneal nerve actually does, then you will appreciate there will be weakness of foot eversion and sensory disturbance over the lateral calf as well as the dorsum of the foot. Toe and ankle dorsiflexion will also be affected (hence the foot drop). If the lesion is proximal to the fibular neck, the short head of biceps femoris will also be affected, however this is small print. EMG will be able to confirm the diagnosis.

116
Q

A 39 yr old female presents with headache and vomiting and also complains of losing her balance. On examination she scores poorly on the mini mental test and has papilloedema.

What is the most likely diagnosis?
A. TIA
B. Brain Abscess
C. L Frontal Meningioma
D. Motor Neurone Disease
E. L VIII nerve Neurofibroma (acoustic neuroma)
F. R Parietal Glioma
G. Lateral Lumbar disc protrusion
H. Common Peroneal Nerve palsy
I. Adult Hydrocephalus
J. Meningitis
A

I. Adult Hydrocephalus

This is normal pressure hydrocephalus – a diagnosis you should suspect in any patient who presents with a gait apraxia (losing her balance here). There may also be urinary symptoms such as urgency, frequency or urge incontinence. This patient also demonstrates cognitive impairment with a poor MMSE score and. These symptoms tend to be insidious in onset over months or years. There is not significantly raised CSF pressure though perhaps this should not be named ‘normal pressure’ as elevated CSF pressure may be seen but just not significantly so.

117
Q

A 58 yr old female zoo-keeper complains of loss of sensation in L arm and leg and L weakness for the past 3 months. On further questioning she describes episodes of tingling in L limbs. On examination you find L homonymous field defect.

What is the most likely diagnosis?
A. TIA
B. Brain Abscess
C. L Frontal Meningioma
D. Motor Neurone Disease
E. L VIII nerve Neurofibroma (acoustic neuroma)
F. R Parietal Glioma
G. Lateral Lumbar disc protrusion
H. Common Peroneal Nerve palsy
I. Adult Hydrocephalus
J. Meningitis
A

F. R Parietal Glioma

This question is testing whether you can work out the location of the lesion opposed to knowing what a glioma is (basically a tumour derived from glial cells as I’m sure you have already guessed). There is left sided weakness and tingling and a left homonymous field defect. The lesion is therefore on the right side. The other conditions listed are not consistent with this patient’s presentation.

118
Q

A 62 yr old male porter complains of low back pain with radiation down the buttock and lower limb. He said it got worse this morning when lifting a patient out of bed. On examination he has weakness of plantar flexion and can hardly raise his leg. He has absent reflexes.

What is the most likely diagnosis?
A. TIA
B. Brain Abscess
C. L Frontal Meningioma
D. Motor Neurone Disease
E. L VIII nerve Neurofibroma (acoustic neuroma)
F. R Parietal Glioma
G. Lateral Lumbar disc protrusion
H. Common Peroneal Nerve palsy
I. Adult Hydrocephalus
J. Meningitis
A

G. Lateral Lumbar disc protrusion

This is a herniated lumbar disc, which is where the disc material (nucleus pulposus or annulus fibrosis) is displaced beyond the intervertebral disc space. There is a male preponderance of this condition and the highest prevalence is between the age of 30 and 50. There is radiating lower back pain here, referred frequently to the paraspinal muscles, buttocks and back of the thighs. The pain is worse with axial loading so activities such as lifting a patient out of bed, or any sitting/standing makes it worse whereas it is relived on lying flat. The neurological deficits here result from nerve root dysfunction and additionally the patient may have cauda equina dysfunction in some cases. MRI is the imaging modality of choice as it gives a uniquely good detail of disc status. Treatment depends on the individual case and may either be surgical such as decompression or non-surgical with lifestyle measures, drugs and physiotherapy.

119
Q

A 45 yr old male lawyer presents with worsening headaches. His family tell you that he cannot talk properly and that his personality has changed over past 2 yrs. On examination he is apyrexial and you find papilloedema and R hemiparesis.

What is the most likely diagnosis?
A. TIA
B. Brain Abscess
C. L Frontal Meningioma
D. Motor Neurone Disease
E. L VIII nerve Neurofibroma (acoustic neuroma)
F. R Parietal Glioma
G. Lateral Lumbar disc protrusion
H. Common Peroneal Nerve palsy
I. Adult Hydrocephalus
J. Meningitis
A

C. L Frontal Meningioma

A meningioma is predominantly a benign tumour of the meninges occuring at both cranial and spinal sites, although these can ocassionally be atypical or malignant. The symptoms are due to local mass effect due to compression of neural structures, such as the headaches, altered speech and personality here and raised ICP. The headache may be progressive, focal or general and are more common in larger tumours.Given the neurological deficit here we can place the rough location of the tumour to the frontal lobe area. There is expressive or Broca’s aphasia here with damage to the motor association cortex in the left frontal lobe. Additionally the perrsonality change is also associated with frontal lobe dysfunction – remember the case of Phineas Gage (now he drinks, swears and flies in a rage!). Additionally these may present with a seizure. Diagnosis is made on MRI which gives a characteristic appearance of a contrast-enhancing tumour, possible with surrounding oedema. Asymptomatic lesions may simply be followed up with serial observation, however this symptomatic case will require surgical resection.

120
Q

A 28 yr old man comes into A+E complaining of sudden chest pain with increasing breathlessness. On examination there is reduced chest wall movements on the right with a mediastinal shift towards the left.

What is the most likely diagnosis?
A. Cancer of the lung
B. Pneumothorax
C. Bronchiectasis
D. Pulmonary embolism
E. Pneumonia
F. Pleural effusion
G. Asthma
H. COPD
I. Inhaled foreign body
J. Pulmonary oedema
A

B. Pneumothorax

This man has the signs and symptoms of a right sided tension pneumothorax and you would expect to find the trachea shifted over to the contralateral side too. The extreme SOB he seems to be suffering and the mediastinal shift here indicate a likely tension pneumothorax. This requires immediate decompression which is achieved by the insertion of a 14G IV cannula into the pleural space at the midclavicular line either at the 2nd or the 3rd intercostal space. This should not be delayed to await radiographic confirmation. Chest pain occurs on the same side as the pneumothorax and the degree of SOB depends on the size of the pneumothorax as well as whether there is any pre-existing lung disease. On examination, you would expect to find the ipsilateral hemithorax to be hyperexpanded (the extent of which again depends on the size) and hyper-resonant to percussion. Ipsilaterally, there would also be absent or diminished breath sounds.

121
Q

A 60 yr old lady complains of chest pain, which she has had for a few days. On examination one side was found to have reduced chest wall movement, a dull percussion note, reduced vocal resonance and absent breath sounds

What is the most likely diagnosis?
A. Cancer of the lung
B. Pneumothorax
C. Bronchiectasis
D. Pulmonary embolism
E. Pneumonia
F. Pleural effusion
G. Asthma
H. COPD
I. Inhaled foreign body
J. Pulmonary oedema
A

F. Pleural effusion

This question tests your ability to differentiate the causes of respiratory exam findings and to recognise that this is consistent with a pleural effusion. A pleural effusion would result in reduced movement of the chest wall with the trachea either central or pushed away from the side with the effusion. The percussion note would be stony dull, although it can be difficult to clinically distinguish stony dull from dull, particularly if you are less experienced. Breath sounds would be decreased, with bronchial breathing reported at the surface of the effusion. An aegophony may also be heard above an effusion. There are no added sounds associated with an effusion. In terms of vocal resonance or tactile vocal fremitus, the thing to be aware of is that normal VR is heard over areas of vesicular breathing, decreased VR over areas with decreased breath sounds and increased VR over areas of bronchial breathing. It makes sense really. So in this case vocal resonance is reduced as breath sounds are absent (except as mentioned before, the aegophony above an effusion). It is worth learning a list of some of the main pathologies which cause a pleural effusion.

People in past years have asked why this is not a pneumonia. A pneumonia would give increased vocal resonance and bronchial breathing (as long as the airway is patent). Why is this not COPD? COPD gives a normal percussion note.

122
Q

A 72 yr old man who has been a life long smoker goes to his G.P. complaining of a cough producing copious amounts of thick, foul-smelling, green sputum, breathlessness and wheeze. On examination there is clubbing and coarse crackles over the lung bases.

What is the most likely diagnosis?
A. Cancer of the lung
B. Pneumothorax
C. Bronchiectasis
D. Pulmonary embolism
E. Pneumonia
F. Pleural effusion
G. Asthma
H. COPD
I. Inhaled foreign body
J. Pulmonary oedema
A

C. Bronchiectasis

Bronchiectasis is permanent bronchi dilatation due to bronchial wall damage and loss of elasticity. It is often as a consequence of recurrent/severe infections and most present with chronic productive mucopurulent cough. Many patients will have crackles on auscultation of the lungs and high-pitched inspiratory squeaks and pops are also commonly heard. SOB is present in the majority of patients, especially with exertion (and is a good correlate of the severity on CT) and more than half will have recurrent episodes of fever, which is also a sign of an acute exacerbation. Daily sputum production is present in two thirds of patients and bloody sputum in half. The most common identifiable cause is CF and clubbing is a sign of bronchiectasis. Chest CT is the diagnostic test. Diagnosis is aided by sputum analysis. Have a think about what you would expect to hear on ascultation of the chest.

123
Q

An 84 yr old lady, who has been a heavy smoker, and lived in the city all her life, visits her G.P. She complains of cough, chest pain, haemoptysis, breathlessness, unintentional weight loss and malaise. On examination she has clubbing.

What is the most likely diagnosis?
A. Cancer of the lung
B. Pneumothorax
C. Bronchiectasis
D. Pulmonary embolism
E. Pneumonia
F. Pleural effusion
G. Asthma
H. COPD
I. Inhaled foreign body
J. Pulmonary oedema
A

A. Cancer of the lung

Consistent respiratory symptoms combined with a history of heavy smoking and unintentional weight loss points strongly towards lung cancer. First line treatment aims at surgical resection if possible. Small cell lung cancer is treated with chemotherapy and is associated with SIADH and ectopic ACTH. Non-small cell lung cancer is more often associated with clubbing. Squamous cell carcinoma is associated with PTHrp release and is treated with radiotherapy. Adenocarcinomas are usually located peripherally in the lung and are more common in non-smokers although most cases are still associated with smoking. The paraneoplastic syndromes may include Lambert-Eaton myasthenic syndrome.

124
Q

A 20 yr old man complains of episodic wheezing, cough and shortness of breath, which gets worse at night and in the early morning.

What is the most likely diagnosis?
A. Cancer of the lung
B. Pneumothorax
C. Bronchiectasis
D. Pulmonary embolism
E. Pneumonia
F. Pleural effusion
G. Asthma
H. COPD
I. Inhaled foreign body
J. Pulmonary oedema
A

G. Asthma

This man has asthma where symptoms are typically worse at night or first thing in the morning. Symptoms include SOB, cough and wheeze which can all be precipitated by exposure to allergens, cold air, smoke or with emotions like laughing too hard. Diagnosis is supported by PEFR variation of at least 20% over 3 days in a week over several weeks or an increase of at least 20% to treatment. Look up the BTS guidelines for more information.

125
Q

A 12 year old boy presents with shortness of breath, coughing and wheezing while playing football in the park on a cold evening. On examination there is reduced chest expansion and prolonged expiratory time.

What is the most likely diagnosis?
A. Asthma
B. Pleural effusion
C. Lung Cancer
D. Allergic Alveolitis
E. Pulmonary Embolism
F. COPD
G. Bronchiectasis
H. Anaphylaxis
I. Idiopathic pulmonary fibrosis
J. Pneumothorax
K. Heart failure
A

A. Asthma

SOB, cough and wheeze, made worse with cold air and exercise suggests asthma. Other asthma triggers include respiratory infection, environmental tobacco smoke and emotion. Examination can show an expiratory wheeze but may be normal. Here it shows prolonged expiratory time (consistent with the obstructive pattern seen in asthma) and reduced chest expansion. Treatment is step-wise based on BTS guidelines. It is worth noting that in severe exacerbations, the chest may be silent. Night symptoms occur in more severe asthma. Diagnosis is supported by PEFR variation of at least 20% over 3 days in a week over several weeks or an increase of at least 20% to treatment. Look up the BTS guidelines for more information. Children have different guidelines.

126
Q

A 68 year old man presents to his GP with breathlessness at rest, wheezing, cough and a sputum production. On examination he was found to use his accessory muscles of respiration and had prolonged expiration. He says he has smoked 25 a day since he was 15.

What is the most likely diagnosis?
A. Asthma
B. Pleural effusion
C. Lung Cancer
D. Allergic Alveolitis
E. Pulmonary Embolism
F. COPD
G. Bronchiectasis
H. Anaphylaxis
I. Idiopathic pulmonary fibrosis
J. Pneumothorax
K. Heart failure
A

F. COPD

Smoking is the most important risk factor, accounting for 90% of COPD. COPD has an insidious onset and usually presents in older people with a history of cough, wheeze and SOB. Examination findings are also suggestive of COPD with prolonged expiration (obstructive pattern) and the use of accessory muscles. Patients with COPD are at a higher risk of infections and are vaccinated against influenza annually and pneumococcal pneumonia every 5 years. Spirometry is the gold standard for diagnosis, with FEV1/FVC ratio <70% with no evidence of reversibility (unlike asthma) being indicative.

127
Q

A 25 year old man comes to A&E complaining of severe central chest pain, SOB and haemoptysis. He is shocked and found to be tachycardic. He also has central cyanosis.

What is the most likely diagnosis?
A. Asthma
B. Pleural effusion
C. Lung Cancer
D. Allergic Alveolitis
F. COPD
G. Bronchiectasis
H. Anaphylaxis
I. Idiopathic pulmonary fibrosis
J. Pneumothorax
K. Heart failure
A

E. Pulmonary Embolism

Patients with a high clinical suspicion of PE should be anticoagulated while waiting a definitive diagnosis unless contraindicated. Options include LMWH, unfractionated heparin or a factor Xa inhibitor. The underlying pathophysiology is based on Virchow’s triad. SOB and chest pain are common symptoms and there may also be haemoptysis. Strong risk factors include DVT, obesity, surgery in the past 2 months, prolonged bed rest, malignancy, previous VTE, pregnancy and the thrombophilias such as factor V Leiden. ECG may be normal, or may show tachycardia, new RAD, new RBBB or the classical S wave in I, Q wave with T inversion in III. Various clinical probability scores exist for PE and D-dimer can be used to exclude PE as a diagnosis.

A two-level Wells score is used to guide management:

If the score is 5 or more, arrange an immediate CTPA, or give LMWH if immediate CTPA is not available

If the score is 4 or less, perform a d-dimer test. if the test is positive arrange an immediate CTPA or give LMWH if immediate CTPA cannot be given.

If the d-dimer is negative, consider an alternative diagnosis.

LMWH should be continued for at least 5 days (or until the INR is above 2 for 24 hours, whichever is longer), and warfarin should be added within 24 hours of confirmation of the diagnosis, and continued for at least 3 months.
NB there are guideline variations for pregnant women and cancer patients.

128
Q

A farmer goes to his doctor and complains of fever, malaise, cough and shortness of breath. Physical examination reveals tachypnoea, coarse end-expiratory crackles and wheezes

What is the most likely diagnosis?
A. Asthma
B. Pleural effusion
C. Lung Cancer
D. Allergic Alveolitis
E. Pulmonary Embolism
F. COPD
G. Bronchiectasis
H. Anaphylaxis
I. Idiopathic pulmonary fibrosis
J. Pneumothorax
K. Heart failure
A

D. Allergic Alveolitis

This is extrinsic allergic alveolitis (EAA), a hypersensitivity pneumonitis. There is history here of exposure to organic dust (avian proteins) with birds. There are a variety of syndromes this could be – pidgeon breeder’s lung, bird fancier’s lung and budgerigar fancier’s disease. History and examination findings depend on whether the EAA is acute, subacute or chronic. Chronic presents like idiopathic pulmonary fibrosis and there may be clubbing. The most important treatment element is to avoid antigen exposure.

129
Q

A 28 year old male presents to his GP with a worsening cough which started a few months ago, due to this he has now seen blood in his sputum which he also says is foul smelling. On examination he also has clubbing and cyanosis.

What is the most likely diagnosis?
A. Asthma
B. Pleural effusion
C. Lung Cancer
D. Allergic Alveolitis
E. Pulmonary Embolism
F. COPD
G. Bronchiectasis
H. Anaphylaxis
I. Idiopathic pulmonary fibrosis
J. Pneumothorax
K. Heart failure
A

G. Bronchiectasis

Bronchiectasis is permanent bronchi dilatation due to bronchial wall damage and loss of elasticity. It is often as a consequence of recurrent/severe infections and most present with chronic productive mucopurulent cough. Daily sputum production is present in two thirds of patients and bloody sputum in half. The most common identifiable cause is CF and clubbing is a sign of bronchiectasis. Haemoptysis is present in around 50%, as mentioned, and is usually mild and thought to come from bronchial arteries or bronchial-pulmonary anastomoses. Chest CT is the diagnostic test. Diagnosis is aided by sputum analysis. Have a think about what you would expect to hear on ascultation of the chest.

130
Q

A 64 year old shop assistant complains of breathlessness. Examination reveals central cyanosis, clubbing and fine end inspiratory crackles.

What is the most likely diagnosis?
A. Asthma
B. Pleural effusion
C. Lung Cancer
D. Allergic Alveolitis
E. Pulmonary Embolism
F. COPD
G. Bronchiectasis
H. Anaphylaxis
J. Pneumothorax
K. Heart failure
A

I. Idiopathic pulmonary fibrosis

Idiopathic pulmonary fibrosis (previously known as cryptogenic fibrosing alveolitis) progresses over several years and is characterised by pulmonary scar tissue formation and dyspnoea. Patients complain of a non-productive cough and typically reproducible and predictable SOB on exertion. Contact with small organic or inorganic dust particles is thought to be implicated in the cascade of events leading to IPF and another risk factor is cigarette smoking which significantly increases the risk of IPF. The mean age of diagnosis is 60-70. End expiratory basal crackles are found on examination. These are described as ‘Velcro-like’ in quality. IPF is also associated with clubbing. The lack of an obvious allergen in the question and the patient’s age leads you away from chronic EAA towards IPF.

131
Q

A 71-year-old man is admitted to the burns unit with 45% burns to the body. He is sedated, given diamorphine and a course of prophylactic antibiotics in addition to aggressive fluid resuscitation and dressings. The next day he experiences several bouts of haematemesis. FBC: Hb 9.2 g/dl, MCV=90fl, WCC 15.1 x 109/l, Plt 410 x 109/l; INR: 1.0

What is the mmost likely diagnosis?
A. Zollinger-Ellison syndrome
B. Peptic ulcer disease (duodenal/gastric ulcer)
C. Mallory-Weiss tear
D. Diverticular disease
E. Ulcerative colitis
F. Oesophagitis
G. Gastric leiomyoma
H. Gastric carcinoma
I. Pancreatitis
J. Hiatus hernia
K. Oesophageal varices
L. Gastric erosions
M. Oesophageal carcinoma
A

L. Gastric erosions

Gastric erosions can occur as a side effect of burns which accounts for this man’s haematemesis. This explains the microcytic anaemia too and the raised WCC can be due to wound infection (though neutropenia and thrombocytopenia may be indicators of sepsis in this patient with severe burns, if seen). This is a serious burn which is most effectively managed in a regional burn centre. Burn extent can be estimated fairly accurately using a Lund-Browder chart which compensates for changes in body proportions with age, or a simpler method would involve the ‘rule of nines’. This patient will have been treated with wound cooling, cleaning, dressing, pain management and tetanus prophylaxis.

132
Q

A 30-year-old woman investigated for one year for recurrent peptic ulceration was admitted with haematemesis. Ranitidine had previously failed to control symptoms and she is presently taking omeprazole 40 mg. Endoscopy shows a large 2 cm actively bleeding ulcer in the duodenum. CT scanning shows a 2 cm mass in the pancreas.

What is the mmost likely diagnosis?
A. Zollinger-Ellison syndrome
B. Peptic ulcer disease (duodenal/gastric ulcer)
C. Mallory-Weiss tear
D. Diverticular disease
E. Ulcerative colitis
F. Oesophagitis
G. Gastric leiomyoma
H. Gastric carcinoma
I. Pancreatitis
J. Hiatus hernia
K. Oesophageal varices
L. Gastric erosions
M. Oesophageal carcinoma
A

A. Zollinger-Ellison syndrome

Zollinger-Ellison syndrome is a gastrin producing neuroendocrine tumour which causes the hypersecretion of gastric acid and subsequent PUD. Gastrin directly stimulates parietal cell secretion and also causes expansion of the mass of parietal cells to cause an increase in basal acid output and maximal acid output. The condition can either be sporadic or associated with MEN1 (one third of MEN1 cases have ZES and 20% of ZES have MEN1). The tumours are thought to arise from delta cells in the pancreas and account for about 25-40% of all gastrinomas. The rest come mainly from enteroendocrine cells of the small bowel while some 5% can arise from other intraabdominal areas. Common presentations include the one seen, with refractory peptic ulcer disease, abdominal pain, diarrhoea and gastro-oesophageal disease. Treatment involves controlling gastric hypersecretion with PPIs. The diagnosis is confirmed by discontinuing PPIs and measuring gastrin levels again. Surgery is only indicated for localised disease. Metastatic disease is the most common cause of morbidity and mortality in this condition and around 55-90% of gastrinomas are malignant and will usually metastasize to the lymph nodes and liver.

133
Q

A 42-year-old man was brought into A&E following several episodes of vomiting fresh blood. The patient is drowsy and a poor historian. Investigations include: FBC: Hb 8.1 g/dl, MCV 106 fl, Plt 167 x 109/l, WCC 11.7 x 109/l,: INR 2.1

What is the mmost likely diagnosis?
A. Zollinger-Ellison syndrome
B. Peptic ulcer disease (duodenal/gastric ulcer)
C. Mallory-Weiss tear
D. Diverticular disease
E. Ulcerative colitis
F. Oesophagitis
G. Gastric leiomyoma
H. Gastric carcinoma
I. Pancreatitis
J. Hiatus hernia
K. Oesophageal varices
L. Gastric erosions
M. Oesophageal carcinoma
A

K. Oesophageal varices

This is a case of haematemesis secondary to oesophageal varices. Oesophageal varices are a direct result of portal hypertension, which occurs as a progressive complication of cirrhosis. The other options here do not fit this presentation. MCV which is a measure of RBC size, increases with excessive alcohol intake. The bleeding here is acute so the patient has not developed IDA. Low platelet count may be indicative of portal hypertension. The INR, which is a measure of prothrombin time, is also elevated, which represents a decline in hepatic synthetic function due to cirrhosis. An acute bleed like this case can be managed with resuscitation, terlipressin or somatostatin (or analogues) and endoscopic ligation. Shunt therapy and antibiotic prophylaxis can also be deployed if necessary. Diagnosis and surveillance by endoscopy is an important part of management of this condition and in terms of prophylaxis against variceal bleeding before it has occured, non-selective beta blockers and/or endoscopic ligation can be used.

134
Q

A 75-year-old man presents with several episodes of coffee ground vomiting. Further questioning reveals a five month history of epigastric discomfort, nausea, anorexia and weight loss. FBC: Hb 7.9 g/dl, MCV 76.6 fl, WCC 5.3 x 109/l, Plts 333 x 109/l; INR 1.1

What is the mmost likely diagnosis?
A. Zollinger-Ellison syndrome
B. Peptic ulcer disease (duodenal/gastric ulcer)
C. Mallory-Weiss tear
D. Diverticular disease
E. Ulcerative colitis
F. Oesophagitis
G. Gastric leiomyoma
H. Gastric carcinoma
I. Pancreatitis
J. Hiatus hernia
K. Oesophageal varices
L. Gastric erosions
M. Oesophageal carcinoma
A

H. Gastric carcinoma

This is cancer of the stomach which has presented with weight loss, abdominal discomfort located in the epigastrium and gastric bleeding, which is a common complication. Coffee ground vomit refers to vomit where the iron in RBCs has been exposed to gastric acid for some time before being vomited up, resulting in oxidisation and causing the vomit to resemble ground vomit. Some patients will also present with melaena, which is tarry black stools due to oxidised blood excreted in stool. Risk factors for the development of gastric carcinoma include pernicious anaemia, Helicobacter pylori and exposure to N-nitroso compounds commonly used as a food additive in cured meats. Men are twice as likely to get gastric cancer and it is most prevalent in the 50-70 age range. Smoking is also a risk factor. Nausea seen in this patent is also a diagnostic factor, but is not common. Additionally, those with more proximal and gastro-oesophageal junction tumours may present with dysphagia. There is a microcytic anaemia here associated with blood loss.

135
Q

A 21-year-old medical student comes in to A&E after the participating in a ‘beer race’. After vomiting several times he notices bright red blood in the vomitus. He had only consumed 11-and-a-half pints of beer during the event. FBC: Hb 14.2 g/dl, MCV 85 fl, WCC 8.2 x 109/l, Plt 450 x 109/l; INR 1.0

What is the mmost likely diagnosis?
A. Zollinger-Ellison syndrome
B. Peptic ulcer disease (duodenal/gastric ulcer)
C. Mallory-Weiss tear
D. Diverticular disease
E. Ulcerative colitis
F. Oesophagitis
G. Gastric leiomyoma
H. Gastric carcinoma
I. Pancreatitis
J. Hiatus hernia
K. Oesophageal varices
L. Gastric erosions
M. Oesophageal carcinoma
A

C. Mallory-Weiss tear

This is a Mallory-Weiss tear. This occurs after a rise in abdominal pressure which induces a tear in the oesophageal mucosa, causing subsequent GI bleeding. It commonly presents with haematemesis after an episode of retching/vomiting/coughing/straining. Hence, risk factors include anything which can cause vomiting like heavy alcohol use, which is commonly the case in EMQs. Also, other conditions would include food poisoning, bowel obstruction, hyperemesis gravidarum, bulimia, the chronic cough of COPD, meningitis etc… you name it really. Classically, MWT presents with a small self limiting episode of haematemesis. Definitive diagnosis is made by OGD. Treatment is supportive because most cases, as mentioned, are self limiting and emergency treatment is not offered unless the patient is showing signs of clinical instability. If the patient is actively bleeding, treatment will be with therapeutic endoscopy in most cases, and very very few cases will require more intervention such as angiography with embolisation.

136
Q

A 21 year old man with a family history of sudden death before the age of forty presents to his GP with LVF. On examination there is an ejection systolic murmur present and echocardiography shows a thickened ventricular septum.

What is the most likely diagnosis?
A. Mitral Regurgitation
B. Aortic stenosis
C. Mitral Stenosis
D. Aortic Regurgitation
E. Complete Heart Block
F. Supraventricular tachycardia
G. Ventricular Septal defect
I. Atenolol toxicity
A

H. Hypertrophic obstructive cardiomyopathy

This patient has likely HOCM. HOCM is the most likely cardiomyopathy in this age and the most frequent cause of sudden cardiac death in younger people. The FH of sudden death is suggestive of this diagnosis which is confirmed by echocardiography showing asymmetric septal hypertrophy and LVH. LVH is the cardinal feature and the classic pattern of asymmetrical septal hypertrophy is defined as a diameter >1.3-1.5 times the posterior wall diameter in diastole. A systolic ejection murmur is a common finding which is audible at the lower left sternal edge and accentuated on standing and exercise, and lessened by lying flat or squatting. This has a benign prognosis is most people though symptomatic patients are treated medically with beta blockade, CCBs or disopyramide. Chest pain on exertion is a common presentation in those who are symptomatic, as is dyspnoea on exertion, palpitations (such as due to AF) and a history of either pre-syncope or syncope (due to LV outflow obstruction). Inheritance is autosomal dominant with a variable penetrance.

137
Q

A 74 year old woman presents to her GP with LVF. Examination shows that she has an irregularly irregular pulse and a rumbling mid-diastolic murmur heard at the apex.

What is the most likely diagnosis?
A. Mitral Regurgitation
B. Aortic stenosis
C. Mitral Stenosis
D. Aortic Regurgitation
E. Complete Heart Block
F. Supraventricular tachycardia
G. Ventricular Septal defect
H. Hypertrophic obstructive cardiomyopathy
I. Atenolol toxicity
A

C. Mitral Stenosis

Practically every single case of mitral stenosis is caused by rheumatic heart disease. The process tends to also cause regurgitation. LA enlargement and increased age commonly also cause atrial fibrillation which is seen here. Women are three times more likely than men to get mitral stenosis. The murmur of mitral stenosis is characteristically a grade 1-2 low pitch murmur heard in mid-diastole which has a rumbling nature. There can be an associated malar flush, tapping apex beat and a diastolic thrill palpable at the apex, in the 5th intercostal space in the MCL. The first heart sound is also characteristically loud and often this is the most striking feature on ascultation. It is a difficult murmur to pick up so if you are ever asked at this stage to spot this murmur, it will most likely be based on the loud S1.

138
Q

A 58 year old man who presented with an inferior MI four hours ago suddenly becomes extremely breathless and tired. His hands and feet look pale. Pulmonary oedema is seen on CXR. Auscultation reveals a new, pansystolic murmur at the apex which radiates into the axilla.

What is the most likely diagnosis?
B. Aortic stenosis
C. Mitral Stenosis
D. Aortic Regurgitation
E. Complete Heart Block
F. Supraventricular tachycardia
G. Ventricular Septal defect
H. Hypertrophic obstructive cardiomyopathy
I. Atenolol toxicity
A

A. Mitral Regurgitation

MR can occur as a complication of MI which may cause structural damage to the mitral valve apparatus. MR is loudest at the apex and radiates to the axilla and tends to be around grade 4. It is associated with a systolic thrill at the apex. TTE is the investigation of choice for diagnosis. Chronic MR is associated with a laterally displaced apex beat with LV dilatation. This case of acute MR in the setting of an acute MI is very serious and has here led to high LA pressure and pulmonary oedema secondary to reduced LA compliance. There are symptoms here of sudden congestive cardiac failure with SOB and there is associated peripheral vasoconstriction. Occasionally no murmur is heard. This is not a VSD because the murmur of MR here is most prominent in the mitral area and there is axillary radiation based on the regurgitant jet. VSD also gives a pansystolic murmur, which is generally easily heard, and is loudest at the left parasternal region.

139
Q

A 67 year old with uncontrolled hypertension presents with LVF. She has a collapsing pulse, a laterally displaced apex beat and a soft early diastolic murmur best heard at the left sternal edge.

What is the most likely diagnosis?
A. Mitral Regurgitation
B. Aortic stenosis
C. Mitral Stenosis
D. Aortic Regurgitation
E. Complete Heart Block
F. Supraventricular tachycardia
G. Ventricular Septal defect
H. Hypertrophic obstructive cardiomyopathy
I. Atenolol toxicity
A

D. Aortic Regurgitation

Aortic regurgitation is the leakage of blood back into the LV in diastole. The collapsing pulse is also known as a water hammer or Corrigan’s pulse and describes the rapid rise and quick ‘collapse’ of the arterial pulse resulting in a wide pulse pressure. The murmur in AR is early diastolic in mild cases and increases to pansystolic in severe cases. Risk factors include a bicuspid valve, rheumatic fever, endocarditis, anklylosing spondylitis and Marfan’s. The uncontrolled hypertension in this patient is a weak risk factor, which can lead to aortic root dilation and inadequate closure of the aortic valve leaflets. Other commonly seen signs in EMQs, although uncommon in clinical practice include Traube’s, Quincke’s, Duroziez’s and de Musset’s sign. Occasionally although uncommon, an Austin Flint murmur may be heard which is a rumbling mid-diastolic murmur best heard at the apex, produced by the regurgitant jet hitting the LV endocardium. Its presence indicates severe AR and the absence of a loud S1 or an opening snap distinguishes this from the murmur of mitral stenosis.

140
Q

An 80 year old man who has recently been started on verapamil with the hope of controlling his angina presents with LVF. An ECG shows dissociation between P waves and QRS complexes. The QRS complexes are broad and occurring at a rate of 30/min. ST segments are normal.

What is the most likely diagnosis?
A. Mitral Regurgitation
B. Aortic stenosis
C. Mitral Stenosis
D. Aortic Regurgitation
E. Complete Heart Block
F. Supraventricular tachycardia
G. Ventricular Septal defect
H. Hypertrophic obstructive cardiomyopathy
I. Atenolol toxicity
A

E. Complete Heart Block

The ECG here is diagnostic of complete heart block, though it is worth noting that the degree of heart block does not necessarily correlate with the symptoms experienced. Remember that in first degree AV block, there is a fixed but extended PR interval. In second degree AV block Mobitz type 1, there is progressive PR lengthening and eventual loss of AV conduction for one beat, and in Mobitz type 2 there is occasional loss of AV conduction for a beat which is preceded and followed by fixed unchanging PR intervals. In third degree AV block seen here though, there is no consistent PR relationship. The QRS complexes are broad due to a ventricular escape rhythm – the ventricles are acting as their own pace maker resulting in bradycardia because AV block is complete and signals are not getting to the ventricular myocardium from the SA node. Verapamil here is a risk factor as AV nodal blocking medications such as beta blockers, CCBs, digitalis and adenosine are commonly implicated in heart block. Some anti-arrhythmics have also been responsible. Other risks include increased vagal tone, underlying cardiovascular disease, hypertension and CHF.

141
Q

A 40 year old receptionist present with a short history of central chest pain made worse by inspiration (or lying flat) and is made better by sitting forward.

What is the most likely diagnosis?
A. Pericarditis
B. Unstable angina
C. Mitral regurgitation
D. MI
E. Digoxin effect
F. Cardiac tamponade
G. Stable angina
H. Mitral stenosis
I. Atrial fibrillation
J. Myocarditis
K. Aortic stenosis
A

A. Pericarditis

These patients have presented with pericarditis. Symptoms include a sharp and severe chest pain retrosternally which is worse on inspiration and when supine, relieved by sitting forwards. The classical finding on examination is a friction rub which is said to sound like ‘walking on snow’. There may be diffuse ST elevations on ECG, classically saddle-shaped, an effusion on echocardiography and blood results suggesting inflammation. Complications include tamponade and constrictive pericarditis. Prior viral infection is a risk factor with the most common pericardial infection being viral. Bacterial purulent pericarditis also occurs. The inflammation is due either to direct viral attack or immune mediated damage. Other risk factors include male gender, post-MI (both ‘early’ and Dressler’s), post-pericardiotomy syndrome, neoplasm from local tumour invasion, uraemia and autoimmune conditions such as RA and SLE.

142
Q

An elderly woman present to A+E with chest pain radiating to her arm, which woke her from her sleep and occurs at minimal exertion (or rest). The pain is similar to her normal angina but is not relieved by her GTN spray. The ECG shows ST depression.

What is the most likely diagnosis?
A. Pericarditis
B. Unstable angina
C. Mitral regurgitation
D. MI
E. Digoxin effect
F. Cardiac tamponade
G. Stable angina
H. Mitral stenosis
I. Atrial fibrillation
J. Myocarditis
K. Aortic stenosis
A

B. Unstable angina

This is likely to be unstable angina but may well be an NSTEMI and you will only be able to tell when the cardiac biomarkers come in and you see if troponin is raised. Note though that if the patient has had an MI in the last 2 weeks, then CK-MB or serum myoglobin should be taken as troponin may remain persistently elevated (although a second rise by 20% of more is still consistent with re-infarction). The presentation here is pretty much indistinguishable from NSTEMI except when given the information that this woman has a history of stable angina, and the pain is a similar kind of pain which is occuring now on minimal exertion and even at rest, you can suspect UA over NSTEMI. The ECG is consistent with both diagnoses really, showing ST segment depression. There may also be T wave inversion and the ECG may also be normal. Acute management of UA includes the use of antiplatelet drugs and antithrombotic therapy to reduce myocardial damage and complications. Long term management is aimed at reducing risk factors and using medication to prevent this from reoccuring. Risk factors include your typical cardiovascular risks such as smoking, hypertension, DM, hyperlipidaemia, FH and PMH of coronary artery disease.

143
Q

A patient presents to A+E with dyspnoea, fatigue and palpitations. Auscultation reveals a pan-systolic murmur at apex radiating to the axilla.

What is the most likely diagnosis?
A. Pericarditis
B. Unstable angina
C. Mitral regurgitation
D. MI
E. Digoxin effect
F. Cardiac tamponade
G. Stable angina
H. Mitral stenosis
I. Atrial fibrillation
J. Myocarditis
K. Aortic stenosis
A

C. Mitral regurgitation

MR is loudest at the apex and radiates to the axilla and tends to be around grade 4. It is associated with a systolic thrill at the apex. TTE is the investigation of choice for diagnosis. Chronic MR is associated with a laterally displaced apex beat with LV dilatation. Mitral valve prolapse is a strong risk factor for development of MR.

144
Q

A 40 year old man presents to A+E having chest pain. It started 40 mins ago and has got worse. The pain is in his chest and radiates to his neck. The ECG shows ST elevation in leads II, III and aVF.

What is the most likely diagnosis?
A. Pericarditis
B. Unstable angina
C. Mitral regurgitation
D. MI
E. Digoxin effect
F. Cardiac tamponade
G. Stable angina
H. Mitral stenosis
I. Atrial fibrillation
J. Myocarditis
K. Aortic stenosis
A

D. MI

This patient’s chest pain sounds is due to an MI. Chest pain is classically severe and heavy in nature (often described as crushing), located centrally with possible radiation to the left arm or jaw and lasts for >20 minutes. SOB due to pulmonary congestion and sweating due to high sympathetic output are also common symptoms. Risk factors incorporate the standard set of cardiovascular risks such as smoking, high BP, DM, obesity and dyslipidaemia. An ECG is indicated which shows an inferior STEMI here. STEMI, new LBBB or confirmed posterior MI is an indication for PCI/thrombolysis. It is worth noting that RV infarction is present in 40% of inferior infarcts so in this case, right sided ECG leads should also be obtained.

145
Q

A 40 year old man ECG showed a widespread saddle-shaped ST elevation.

What is the most likely diagnosis?
A. Pericarditis
B. Unstable angina
C. Mitral regurgitation
D. MI
E. Digoxin effect
F. Cardiac tamponade
G. Stable angina
H. Mitral stenosis
I. Atrial fibrillation
J. Myocarditis
K. Aortic stenosis
A

A. Pericarditis

These patients have presented with pericarditis. Symptoms include a sharp and severe chest pain retrosternally which is worse on inspiration and when supine, relieved by sitting forwards. The classical finding on examination is a friction rub which is said to sound like ‘walking on snow’. There may be diffuse ST elevations on ECG, classically saddle-shaped, an effusion on echocardiography and blood results suggesting inflammation. Complications include tamponade and constrictive pericarditis. Prior viral infection is a risk factor with the most common pericardial infection being viral. Bacterial purulent pericarditis also occurs. The inflammation is due either to direct viral attack or immune mediated damage. Other risk factors include male gender, post-MI (both ‘early’ and Dressler’s), post-pericardiotomy syndrome, neoplasm from local tumour invasion, uraemia and autoimmune conditions such as RA and SLE.

146
Q

An 45 year old single man presents with slight confusion and has had difficulty walking recently. He tells you that he fell over on the way home from the pub a few days ago.

What is the most likely diagnosis?
A. Stroke
B. Cluster Headaches
C. Subdural Haemorrhage
D. Extradural Haemorrhage
E. Sinusitis
F. Transient Ischaemic Attack
G. Tension Headache
H. Migraine
I. Subarachnoid Haemorrhage
A

C. Subdural Haemorrhage

A subdural occurs due to blood collecting between the dura mater and the arachnoid mater surrounding the brain. It may be arterial or venous although is most often venous. The disease course varies but this patient’s presentation fits. The cause is trauma – a fall in this case, most likely due to the patient’s drunken state. Advanced age is associated with chronic subdurals. It is important in the examination to look for signs of trauma such as scalp abrasions and bruises. This patient is symptomatic and surgical options include twist-drill craniotomy with drainage (a bedside procedure where a hand drill is used to gain access to the subdural space and then a catheter is placed to act as a drain). Standard craniotomy is also an option, as is the creation of a burr hole. Remember that extradural haematomas classically have a ‘lucid interval’ and occur in younger patients, usually with an associated skull fracture, and CT of the haematoma does not cross suture lines.

147
Q

An 80 year old woman who is a smoker was brought into A&E from a residential home where her carers noticed that she had difficulty swallowing and that she also had difficulty moving her left arm and leg for the past few days.

What is the most likely diagnosis?
A. Stroke
B. Cluster Headaches
C. Subdural Haemorrhage
D. Extradural Haemorrhage
E. Sinusitis
F. Transient Ischaemic Attack
G. Tension Headache
H. Migraine
I. Subarachnoid Haemorrhage
A

A. Stroke

Weakness on one side and the difficulty swallowing makes this likely to be a stroke. If you have a think about the motor pathways you will realise that this is a right sided stroke. It is important is perform a CT head exclude a haemorrhagic aetiology and consider thrombolysis with tPA if within the 4.5 hour window and there are no contraindications. Thrombolysis is done with alteplase at 10% bolus, 90% infusion at a dose of 0.9 mg/kg. Presentation after the 4.5 hour window is managed with aspirin. The Bamford/Oxford Stroke Classification subtypes ischaemic stroke according to vascular territory of infarction. After initial management, stroke care involves the ethos of an MDT environment with rehabilitation.

148
Q

A 25 year old man while playing cricket was hit on the head with the ball. He recovered enough to finish off the game. He then presented to A&E 8 hours later with a severe headache and vomiting. He rapidly loses consciousness.

What is the most likely diagnosis?
A. Stroke
B. Cluster Headaches
C. Subdural Haemorrhage
D. Extradural Haemorrhage
E. Sinusitis
F. Transient Ischaemic Attack
G. Tension Headache
H. Migraine
I. Subarachnoid Haemorrhage
A

D. Extradural Haemorrhage

Here we have the ‘lucid interval’ classically associated with an extradural haematoma. There is blood buildup this time between the dura mater and the skull. Compressive signs may also be present such as the down and out pupil due to CN III compression. The bleed here is usually from arteries, under high pressure, causing raised intracranial pressure. In this case, there is a chance the brain stem has been compressed causing his LOC.

149
Q

A 40 year old lady who has been a smoker for 25 years and has poorly controlled hypertension suddenly collapsed at a dinner party. She had complained of a severe headache at the back of her head a few minutes earlier.

What is the most likely diagnosis?
A. Stroke
B. Cluster Headaches
C. Subdural Haemorrhage
D. Extradural Haemorrhage
E. Sinusitis
F. Transient Ischaemic Attack
G. Tension Headache
H. Migraine
A

I. Subarachnoid Haemorrhage

SAH (bleeding into the subarachnoid space) presents with sudden severe headache patients will often describe as the worst headache of their life, and can often be so bad that they feel like they’ve been kicked in the back of the back. Hypertension is an important risk factor. Half of all patients lose consciousness and eye pain with exposure to light can also be seen. Altered mental status is common. SAH occurs most commonly in the 50-55 age group and affects women and black people more than men and white people. The most common cause of non-traumatic SAH is an aneurysm which ruptures. Conditions which predispose to aneurysm formation and SAH include adult PKD, Marfan’s, NF1 and Ehlers-Danlos. Cerebral aneurysms arise around the circle of Willis. A CT scan is indicated, and if unrevealing, this should be followed by an LP. Cerebral angiography can confirm the presence of aneurysms. The patient should be stabilised and this followed by surgical clipping or endovascular coil embolisation, the choice is subject to much current controversy sparked by relatively recent research. Complications can commonly occur and include rebleeding, hydrocephalus and vasospasm.

150
Q

A 65 year old man has complained of losing vision in one eye which lasts for a few hours and then goes back to normal. He says its like a ‘a curtain descending over my field of view’.

What is the most likely diagnosis?
A. Stroke
B. Cluster Headaches
C. Subdural Haemorrhage
D. Extradural Haemorrhage
E. Sinusitis
F. Transient Ischaemic Attack
G. Tension Headache
H. Migraine
I. Subarachnoid Haemorrhage
A

F. Transient Ischaemic Attack

A TIA is colloquially called a ‘mini stroke’ with symptoms typically lasting under an hour. Here there is a description of focal neurological deficit unilaterally in the form of amaurosis fugax. There may also be other signs like unilateral weakness or sensory loss, hemianopsia, aphasia and cranial nerve defects, to name but a few. An antiplatelet drug such as aspirin is effective secondary prevention if the patient is not already anticoagulated. The patient will be anticoagulated if they have a likely or known cardioembolic source such as AF. Clopidogrel is an alternative in those who do not tolerate aspirin.

151
Q

A 20 year old student presented to student health with a severe headache and photophobia, and complained that she has seen zig-zag lines 2 hours previously. She is apyrexial. Whilst in the surgery the girl becomes increasingly drowsy.

What is the most likely diagnosis?
A. Migraine
B. Tension headache
C. Trigeminal neuralgia
D. Acute glaucoma
E. Encephalitis
F. Analgesic headache
G. Meningitis
H. Raised ICP
I. Sinusitis
J. Tooth abscess
A

A. Migraine

Migraine is a chronic condition, with genetic determinants, which usually presents in early to mid life. The typical migraine aura this patient describes (which can be visual, sensory or speech symptoms) which can occur during or before the headache, is pathognomic, but is not seen in the majority of patients. The aura can be positive phenomena (for example this patient seeing flashing lights) or negative phenomena (for example visual loss). Nausea, photophobia and disability (the headache gets in the way with the patient’s ability to function) accompanying a headache also suggest a migraine diagnosis. The headache of a migraine tends to be prolonged if untreated, and tends to be unilateral and pounding (but does not have to be). Tests aim to rule out other differentials, although if the history is compatible and neurological examination is unremarkable, further testing is not needed.

152
Q

A 35 year old Management Consultant presented to her GP with a month long history of ongoing headache. She described the pain as like a “tight band around her head.” The doctor noted that she looked stressed and exhausted.

What is the most likely diagnosis?
A. Migraine
B. Tension headache
C. Trigeminal neuralgia
D. Acute glaucoma
E. Encephalitis
F. Analgesic headache
G. Meningitis
H. Raised ICP
I. Sinusitis
J. Tooth abscess
A

B. Tension headache

A tension headache is commonly triggered by stress and mental tension (also, fatigure and missing meals), hence the name. It is more common in females and those in middle age, and there is a link with lower socioeconomic status, although this does not necessarily represent causation. Symptoms include a dull, non-pulsatile and constricting bilateral pain, which is often described as a band across the patient’s head. It is not severe or disabling but classically worsens as the day progresses. This headache normally responds well to simple analgesics.

153
Q

A 54 year old woman went to her GP following a couple of weeks of intermittent intense pain on one side of her face. The lady informed the doctor that the pain occurred particularly when she washed her face.

What is the most likely diagnosis?
A. Migraine
B. Tension headache
C. Trigeminal neuralgia
D. Acute glaucoma
E. Encephalitis
F. Analgesic headache
G. Meningitis
H. Raised ICP
I. Sinusitis
J. Tooth abscess
A

C. Trigeminal neuralgia

Trigeminal neuralgia occurs as episodes of severe unilateral pain in the distribution of CNV, usually lasting seconds, with no pain occuring between these episodes. Examination is often unremarkable. The pain is described as sharp, intense, stabbing or burning. It can be triggered commonly by actions such as eating, tooth brushing, cold and touch. Shaving and eating seem to be common in EMQs. Most people are asymptomatic between attacks although the severity of the pain makes these patients live in constant fear. TN is more common in MS and incidence increases with age. Post-herpetic TN is also possible. The mainstay of treatment is medical, with antiepileptics such as carbamazepine (which is the only medicine which is proven in RCTs and is therefore typically first line). If medical treatment fails, surgical options do exist such as microvascular decompression.

154
Q

An 80 year old man was brought to A&E by his daughter with aching pain around his eye which radiates to his forehead. His daughter told the doctor that her father had been vomiting all morning. He tells you his vision is blurry.

What is the most likely diagnosis?
A. Migraine
B. Tension headache
C. Trigeminal neuralgia
D. Acute glaucoma
E. Encephalitis
F. Analgesic headache
G. Meningitis
H. Raised ICP
I. Sinusitis
J. Tooth abscess
A

D. Acute glaucoma

Acute closed-angle glaucoma can often present like this, with a change in vision with other severe acute symptoms such as eye pain, headache and N&V. This occurs when the iris comes into contact with the trabecular meshwork, which obstructs aqueous outflow from the eye such that intraocular pressure increases. This damages the optic nerve with loss of axons, leading to loss of visual field, which if untreated can progress to complete blindness. Typical changes that can be seen on fundoscopy include large optic disc cup. Immediate treatment is medical and aims at relieving symptoms and dropping IOP. Carbonic anhydrase inhibitors, topical beta blockers or alpha 2 adrenergic agents may be used, and typically in combination. In terms of pharmacology, CA inhibitors decrease aqueous humour formation. Hyper-osmotic agents are used when pressures are exceedingly high, or the patient is unresponsive to medical treatment. After the acute attack resolves, the patient should receive definitive surgical treatment within 24-48 hours to persistently create an open angle. This can be performed by laser peripheral iridotomy where a laser makes a hole in the iris to allow aqueous to bypass the pupil.

155
Q

A 21 year old medical student presented to her GP following a year of intense headaches that occurred once or twice a month. They were unilateral and the girl described experiencing some visual symptoms.

What is the most likely diagnosis?
A. Migraine
B. Tension headache
C. Trigeminal neuralgia
D. Acute glaucoma
E. Encephalitis
F. Analgesic headache
G. Meningitis
H. Raised ICP
I. Sinusitis
J. Tooth abscess
A

A. Migraine

Migraine is a chronic condition, with genetic determinants, which usually presents in early to mid life. The typical migraine aura this patient describes (which can be visual, sensory or speech symptoms) which can occur during or before the headache, is pathognomic, but is not seen in the majority of patients. The aura can be positive phenomena (for example this patient seeing flashing lights) or negative phenomena (for example visual loss). Nausea, photophobia and disability (the headache gets in the way with the patient’s ability to function) accompanying a headache also suggest a migraine diagnosis. The headache of a migraine tends to be prolonged if untreated, and tends to be unilateral and pounding (but does not have to be). Tests aim to rule out other differentials, although if the history is compatible and neurological examination is unremarkable, further testing is not needed.

156
Q

A 50 year old female complains of palpitations, weight loss and insomnia. On examination she has a goitre and exophthalmos.

What is the most likely diagnosis?
A. Iatrogenic Hyperprolactinaemia
B. Phaeochromocytoma
C. Plummers Disease
D. Nephrogenic Diabetes Insipidous
E. Graves Disease
F. Hyperparathyroidism
G. Sheehans Syndrome
H. Acromegaly
I. Cushing Syndrome
J. Cranial Diabetes Insipidus
K. Addisons Disease
L. Diabetic Ketoacidosis
M. Hypercalcaemia
N. Conn’s Syndrome
O. Hypothyroidism
A

E. Graves Disease

This patient has hyperthyroidism. More specifically Graves’ disease (peripheral manifestations such as ophthalmopathy do not occur with other causes of hyperthyroidism). Treatment of Graves’ aims to normalise thyroid function and is achieved by radioactive iodine, antithyroid medications or with surgery. They are all effective and relatively safe options. Symptomatic therapy is given with beta blockers such as propranolol.

157
Q

A 34-year old female, with known Type 1 Diabetes Mellitus and autoimmune-mediated hypothyroidism complains to her GP of constant tiredness and weakness. She has also recently noted various patches of pigmentation over her body. On examination her GP notes a postural drop in BP.

What is the most likely diagnosis?
A. Iatrogenic Hyperprolactinaemia
B. Phaeochromocytoma
C. Plummers Disease
D. Nephrogenic Diabetes Insipidous
E. Graves Disease
F. Hyperparathyroidism
G. Sheehans Syndrome
H. Acromegaly
I. Cushing Syndrome
J. Cranial Diabetes Insipidus
K. Addisons Disease
L. Diabetic Ketoacidosis
M. Hypercalcaemia
N. Conn’s Syndrome
O. Hypothyroidism
A

K. Addisons Disease

Hyperpigmentation points towards Addison’s disease. It occurs due to excess ACTH production and can be mucosal or cutaneous and is more pronounced in the palms, knuckles and around scars. MSH is a byproduct of the production of ACTH from the cleavage of POMC. Sodium is low and potassium elevated in Addison’s. Anorexia and weight loss is observed in all patients. Vomiting is present in 75% of patients and nausea is a common finding. Additionally, postural hypotension may be present as in this case. The presence of other autoimmune diseases is a risk factor for the development of Addison’s and this patient has known auto-immune mediated hypothyroidism and T1DM characterised by the immune-mediated destruction of pancreatic beta cells. Diagnosis of Addison’s can be made on an ACTH stimulation test (synacthen test) whereby serum cortisol remains low despite the administration of synthetic ACTH. In an emergency, treatment should not be delayed by diagnostic testing.

158
Q

A 40 year old man complains of constant attacks of sweating, nausea and panic attacks. CT scanning of his abdomen showed a small mass in his adrenal gland.

What is the most likely diagnosis?
A. Iatrogenic Hyperprolactinaemia
B. Phaeochromocytoma
C. Plummers Disease
D. Nephrogenic Diabetes Insipidous
E. Graves Disease
F. Hyperparathyroidism
G. Sheehans Syndrome
H. Acromegaly
I. Cushing Syndrome
J. Cranial Diabetes Insipidus
K. Addisons Disease
L. Diabetic Ketoacidosis
M. Hypercalcaemia
N. Conn’s Syndrome
O. Hypothyroidism
A

B. Phaeochromocytoma

Phaeochromocytomas presents with paroxysmal episodes of palpitations, anxiety, excessive sweating, pallor and hypertension. The patient may also complain of headaches. It can be inherited in MEN2, von Hippel-Lindau syndrome and NF1. Diagnosis is based on raised urinary and serum catecholamines, metanephrines and normetanephrines. 24 hour urinary VMA will be elevated. CT is used to localise the tumour which is adrenal in this case. Treatment includes medical with the use of phenoxybenzamine, phentolamine and surgical options. Surgical excision is carried out under alpha and beta blockade to protect against the release of catecholamines into circulation when the tumour is being manipulated. The 10% rule is often quoted: 10% are bilateral, 10% malignant, 10% extraadrenal and 10% hereditary.

159
Q

Whilst in hospital a 24 year old male who had been recently admitted to hospital following a RTA whereby he received significant head injuries was noted to have massive polyuria and polydipsia. A water deprivation test had to be abandoned after the patient lost 3kg in weight.

What is the most likely diagnosis?
A. Iatrogenic Hyperprolactinaemia
B. Phaeochromocytoma
C. Plummers Disease
D. Nephrogenic Diabetes Insipidous
E. Graves Disease
F. Hyperparathyroidism
G. Sheehans Syndrome
H. Acromegaly
I. Cushing Syndrome
J. Cranial Diabetes Insipidus
K. Addisons Disease
L. Diabetic Ketoacidosis
M. Hypercalcaemia
N. Conn’s Syndrome
O. Hypothyroidism
A

J. Cranial Diabetes Insipidus

Central DI is due to defective synthesis or release of AVP and will hence respond to DDAVP (desmopressin) which is also the treatment of choice. Nephrogenic DI, on the other hand, occurs due to renal insensitivity to AVP. Hence, it will not respond to DDAVP and treatment is with adequate fluids, salt restriction and thaizide diuretics (paradoxically). This man has cranial DI due to the recent traumatic brain injury which he has suffered. Central DI most commonly occurs following this kind of injury and is most frequently transient in nature, although 7% of cases report permanent central DI. A water deprivation test can be used to confirm DI as a diagnosis although it is potentially dangerous and must be done with caution. It is abandoned if the patient loses 3% of their body weight. Patients are deprived for fluids for 8 hours and serum osmolarity, urine volume and urine osmolarity are measured every hour. If urine osmolarity is shown to be less than serum osmolarity following dehydration then this confirms the diagnosis of DI.

160
Q

A 40-year old female presents with tiredness, weight gain, cold intolerance, oligomenorrhoea and loss of libido.

What is the most likely diagnosis?
A. Iatrogenic Hyperprolactinaemia
B. Phaeochromocytoma
C. Plummers Disease
D. Nephrogenic Diabetes Insipidous
E. Graves Disease
F. Hyperparathyroidism
G. Sheehans Syndrome
H. Acromegaly
I. Cushing Syndrome
J. Cranial Diabetes Insipidus
K. Addisons Disease
L. Diabetic Ketoacidosis
M. Hypercalcaemia
N. Conn’s Syndrome
O. Hypothyroidism
A

O. Hypothyroidism

This is hypothyroidism responsible for tiredness, weight gain, menstrual problems and cold intolerance. Sheehan’s syndrome is pituitary infarction secondary to a post-partum haemorrhage with hypotension. Worldwide, the most common cause of hypothyroidism is iodine deficiency. Other causes include Hashimoto’s or secondary and tertiary hypothyroidism. It can also result from viral de Quervain’s thyroiditis or postpartum thyroiditis. Diagnosis is based on measurement of TSH and thyroid hormones. Treatment is by replacement of T4 with or without T3 in combination. If the patient has normal T3 and T4 but mildly elevated TSH, this is described as subclinical hypothyroidism.

161
Q

A 22-year old male, with a 2 month history of thirst, weight loss, polyuria and polydipsia presents to the local casualty department with extreme nausea and vomiting. Although the patient had no known respiratory pathology he appeared to be hyperventilating. Arterial blood gas samples were taken. PaCO2 : 2.4 kPa (4.8-6.1) PaO2 13 kPa (10-13.3) pH 7.01 (7.35-7.45) Bicarbonate 7mmoll/L (22-26)

What is the most likely diagnosis?
A. Iatrogenic Hyperprolactinaemia
B. Phaeochromocytoma
C. Plummers Disease
D. Nephrogenic Diabetes Insipidous
E. Graves Disease
F. Hyperparathyroidism
G. Sheehans Syndrome
H. Acromegaly
I. Cushing Syndrome
J. Cranial Diabetes Insipidus
K. Addisons Disease
L. Diabetic Ketoacidosis
M. Hypercalcaemia
N. Conn’s Syndrome
O. Hypothyroidism
A

L. Diabetic Ketoacidosis

This is a case of DKA in a type 1 diabetic. In T1DM there is absolute insulin deficiency. This is a case of metabolic ketoacidosis with deep sighing respiratory compensation as lungs try to compensate for the acidosis (Kussmaul breathing). The ABG panel confirms the presence of a metabolic acidosis with respiratory compensation. In reality, venous blood samples can be done and your patient will thank you as they hurt less and there is a lower risk of infection. You just need to bear in mind that venous pH is usually 0.03 units lower than arterial pH. Initial treatment of DKA aims at correcting severe volume depletion, again with IV saline infusion at a rate of 1-1.5L for the first hour. When glucose reaches 11.1mmol, fluid should be changed to 5% dextrose to prevent hypoglycaemia. Bicarbonate therapy may be necessary in adults with pH <7 or bicarbonate level <5mmol/L but this is not the main consideration. The main issue is the severe volume depletion which needs to be rapidly corrected.

162
Q

50 year old man who feels TATT (tired all the time). He recently noticed pigmentation of his skin and has lost some weight.

What is the most likely diagnosis?
A. Glandular fever
B. SIADH secretion
C. Anaemia
D. Chronic renal failure
E. Colorectal carcinoma
F. AIDS
G. Diabetes mellitus
H. Psychological distress
I. Addison's disease
J. Hypothyroidism
A

I. Addison’s disease

Hyperpigmentation in the palmar creases points towards Addison’s disease. Hyperpigmentation due to excess ACTH production can be mucosal or cutaneous and is more pronounced in the palms, knuckles and around scars. MSH is a byproduct of the production of ACTH from the cleavage of POMC. Anorexia, fatigue and weight loss is observed in all patients. Sodium is low and potassium elevated. Vomiting is present in 75% of patients and nausea is a common finding. Additionally, postural hypotension may be present. The presence of other autoimmune diseases is a risk factor for the development of Addison’s. Diagnosis of Addison’s can be made on an ACTH stimulation test (synacthen test) whereby serum cortisol remains low despite the administration of synthetic ACTH. In an emergency, treatment should not be delayed by diagnostic testing.

163
Q

16 year old girl who feels TATT. She had a sore throat prior to this but it has now gone. She is concerned because she is unable to study for upcoming exams. She has noticed her boyfriend had similar symptoms.

What is the most likely diagnosis?
A. Glandular fever
B. SIADH secretion
C. Anaemia
D. Chronic renal failure
E. Colorectal carcinoma
F. AIDS
G. Diabetes mellitus
H. Psychological distress
I. Addison's disease
J. Hypothyroidism
A

A. Glandular fever

This is infectious mononucleosis or glandular fever and is caused by EBV. It is characterised by fever, pharyngitis and lymphadenopathy. Enlargement of the spleen begins in the first week and lasts 3-4 weeks, occuring in half of all cases. Risk factors for EBV transmission include kissing and sex. The fact her boyfriend has had similar symptoms should raise your suspicion. A FBC will show an atypical lymphocytosis. Confirmation of IM involves detection of the existence of heterophile antibodies using the Paul Bunnell monospot. A more accurate test is a serological test detecting EBV specific antibodies. Treatment is usually symptomatic but IM can carry rare but potentially life threatening complications.

164
Q

15 year old boy who is TATT. He is also suffering from polyuria, and thus nocturia 3-4 times per night, and polydipsia.

What is the most likely diagnosis?
A. Glandular fever
B. SIADH secretion
C. Anaemia
D. Chronic renal failure
E. Colorectal carcinoma
F. AIDS
G. Diabetes mellitus
H. Psychological distress
I. Addison's disease
J. Hypothyroidism
A

G. Diabetes mellitus

Polyuria, polydipsia, nocturia in a boy who is tired all the time should make you think of T1DM. Insulin is needed alongside dietary changes and exercise. Insulin regimes aim to mimic physiological insulin release with a basal-bolus dosing. There is an option between using a pump and having multiple daily injections. It is worth noting that there is a high incidence of diabulimia among young people with T1DM who give themselves less insulin than they need in order to lose weight (they lose weight, ‘look good’ but trash their bodies).

165
Q

32 year old homosexual man who is TATT which is associated with weight loss and purple lesions on the skin.

What is the most likely diagnosis?
A. Glandular fever
B. SIADH secretion
C. Anaemia
D. Chronic renal failure
E. Colorectal carcinoma
F. AIDS
G. Diabetes mellitus
H. Psychological distress
I. Addison's disease
J. Hypothyroidism
A

F. AIDS

AIDS (acquired immunodeficiency syndrome) is caused by HIV, which is a retrovirus. To give you an indication of risk here, there is a risk of 50 infections per 10,000 exposed to an infected source in unprotected receptive anal intercourse. The risk with receptive vaginal intercourse is 10 infections per 10,000 exposures. Obviously, people have sex more often than the one off, as is human nature, so what seems like a small risk per sexual encounter adds up. IVDU needle sharing has a risk of 67 per 10,000, a needle-stick is 30 per 10,000 (equal to having receptive vaginal intercourse 3 times with an HIV positive man, so be careful on the wards, though this statistic does depend on factors like the size of the needle) and the risk associated with vertical transmission is associated with maternal viral load (the risk goes if you can suppress the viral load with anti-retrovirals). The thing to note is that the association with homosexuality is based on the increased risk of transmission from receptive anal compared to receptive vaginal, and if you happen to be a homosexual male and contract HIV, you’re unlikely to pass it on with vaginal intercourse. There should not be negative stigma attached to HIV. There are two types, HIV 1 which is the main virus responsible and HIV 2 which is restricted to parts of West Africa. Weight loss is common in HIV and if more than 10% body weight is lost or BMI reduces to 18.5, this is an indication of more severe immunocompromise. Weight loss in HIV may result from malnutrition, co-existent TB infection or HIV wasting syndrome, the latter being an AIDS defining illness. The purple lesions seen here are due to Kaposi’s sarcoma, which is a neoplasm derived from mesenchymal tissue, associated with HHV-8 infection. This is an AIDS defining infection. There are WHO (stage 1-4) and CDC criteria used in clinical staging of HIV. This patient needs to have a CD4 count, HBV and HCV screen, VDRL (syphilis), tuberculin skin test (TB) and CXR. HIV viral load will also be assessed. Prophylaxis and immunisations should be considered against infections such as hepatitis, influenza, PCP and TB. HAART needds to be initiated as he has developed AIDS. Classes of antiretrovirals include NRTIs, NNRTIs, protease inhibitors, fusion inhibitors and integrase inhibitors.

166
Q

22 year old lady who feels TATT. She has also gained a large amount of weight, feels constipated and feels very cold all the time. HR = 45bpm.

What is the most likely diagnosis?
A. Glandular fever
B. SIADH secretion
C. Anaemia
D. Chronic renal failure
E. Colorectal carcinoma
F. AIDS
G. Diabetes mellitus
H. Psychological distress
I. Addison's disease
J. Hypothyroidism
A

J. Hypothyroidism

There is weight gain, cold intolerance, fatigue and constipation as well as bradycardia which all point to hypothyroidism. Worldwide, the most common cause is iodine deficiency. Other causes include Hashimoto’s or secondary and tertiary hypothyroidism. It can also result from viral de Quervain’s thyroiditis or postpartum thyroiditis. Symptoms include those mentioned as well as depression, bradycardia, sluggish reflexes, constipation, cold intolerance and muscle cramps. Diagnosis is based on measurement of TSH and thyroid hormones. Treatment is by replacement of T4 with or without T3 in combination. If the patient has normal T3 and T4 but mildly elevated TSH, this is described as subclinical hypothyroidism.

167
Q

Mrs E, a 26 year old American lady arrives in A&E with severe central chest pain. She is also shocked, pale and sweaty. You notice she has many large suitcases in the bay as you go over to examine her. An overwrought Mr E implores you to save the life of his wife and their unborn child.

What is the most likely diagnosis?
A. Costochondritis
B. Herpes zoster
C. Acute coronary syndrome
D. Ectopic pregnancy
E. Fibromyalgia
F. Cyclical breast pain
G. Pleuritis
H. Panic disorder
I. Pneumothorax
J. Oesophageal reflux
K. Pulmonary embolus
L. Transient ischaemic attack
A

K. Pulmonary embolus

The underlying pathophysiology of PE is based on Virchow’s triad. SOB is a common symptom and there may also be pleuritic chest pain and haemoptysis. Most patients also describe a feeling of apprehension. Pregnancy is a strong risk and there has also presumably been long-haul air travel here. Other strong risk factors include recent surgery, DVT, obesity, prolonged bed rest, malignancy, previous VTE and the thrombophilias such as factor V Leiden. The oral contraceptive pill is also associated with an increased risk of VTE but is a weak risk factor. CXR may be normal or may have findings suggestive of PE such as band atelectasis, hemidiaphragm elevation, Fleischner’s sign, Westermark’s sign and Hampton hump. ECG may be normal, or may show tachycardia, new RAD, new RBBB or the classical S wave in I, Q wave with T inversion in III. Various clinical probability scores exist for PE and D-dimer can be used to exclude PE as a diagnosis.

168
Q

Mr L, a 43 year old investment banker, who is as wide as he is tall, has been brought into A&E at 2am on Sunday morning after collapsing at the Kebab Machine Takeaway on Shepherds Bush Road. Apparently, Mr L had been complaining of tight chest pain and anxiety earlier in the evening, but had had a cigarette to help him calm down.

What is the most likely diagnosis?
A. Costochondritis
B. Herpes zoster
C. Acute coronary syndrome
D. Ectopic pregnancy
E. Fibromyalgia
F. Cyclical breast pain
G. Pleuritis
H. Panic disorder
I. Pneumothorax
J. Oesophageal reflux
K. Pulmonary embolus
L. Transient ischaemic attack
A

C. Acute coronary syndrome

ACS refers to acute myocardial ischaemia caused by atherosclerotic disease and encompasses STEMI, NSTEMI and unstable angina. Once you are aware that this is what acute coronary syndrome encompasses then this diagnosis becomes obvious. This man is clearly overweight and is seen munching a kebab so there are clear cardiovascular risk factors here. The tight chest pain followed by collapse make this sound like a myocardial infarction. Those with identified STEMI should be considered for immediate reperfusion therapy by thrombolytics or percutaneous coronary intervention. Those with NSTEMI or unstable angina do not benefit from this. The pathophysiology underlying all three of these disease processes involves disruption of vulnerable or high risk plaques leading to platelet activation and thrombus formation. Blood flow is disrupted. Most complain of chest pain which is described as substernal pressure, heaviness, squeezing, burning or tightness. It may either localise or radiate to the arms, shoulders, back, neck or jaw and is usually reproduced by exertion, eating, exposure to cold or emotional stress. ECG and serum biomarkers like troponin will be useful in confirming the diagnosis.

169
Q

Miss A, a 63 year old attends the Rapid Diagnostic Clinic for Breast Cancer at Charing Cross. She is concerned over an area of tenderness on her left breast. On examination, you find that the lower medial quadrant of the right breast feels sore when you palpate deeply. There is no palpable lump, and both mammography and ultrasound are normal.

What is the most likely diagnosis?
A. Costochondritis
B. Herpes zoster
C. Acute coronary syndrome
D. Ectopic pregnancy
E. Fibromyalgia
F. Cyclical breast pain
G. Pleuritis
H. Panic disorder
I. Pneumothorax
J. Oesophageal reflux
K. Pulmonary embolus
L. Transient ischaemic attack
A

A. Costochondritis

This is costochondritis, or Tietze’s syndrome (which describes constochondritis accompanied by chest wall swelling), which presents with insidious onset of anterior chest wall pain which is made worse by certain movements of the chest and deep inspiration. The key sign here is that there is pain when palpating the costochondral joints, particularly the 2nd to the 5th and the diagnosis is clinical. Tests are done to exclude other diagnoses here such as breast pathology. First line treatment is with NSAIDs. Oral NSAIDs are preferred in a primary care setting and a beneficial response confirms the diagnosis. If NSAIDs or local corticosteroid injection (usually performed by a specialist) fail to make the symptoms better than you should seek further investigations and consider a wider differential diagnosis which include conditions like pleuritis, ACS, PE, rib fracture and GORD.

170
Q

Mrs I, a plump 65 year old lady presents with burning chest pain. She complains that the pain has stopped her working in her allotment, as it is more likely to come on when she tends to her marrows. She also finds it difficult to sleep at night unless she uses 3 pillows.

What is the most likely diagnosis?
A. Costochondritis
B. Herpes zoster
C. Acute coronary syndrome
D. Ectopic pregnancy
E. Fibromyalgia
F. Cyclical breast pain
G. Pleuritis
H. Panic disorder
I. Pneumothorax
J. Oesophageal reflux
K. Pulmonary embolus
L. Transient ischaemic attack
A

J. Oesophageal reflux

This burning chest pain is comes on when this lady bends over while tending her allotment and when she lies down at night to sleep. This indicates GORD and there is heartburn here and acid regurgitation. No atypical symptoms are present in this lady but these can include cough, laryngitis, asthma or dental erosion. The diagnosis is clinical here and a therapeutic trial of PPI can both be diagnostic and serve as initial treatment. UGI endoscopy is not needed unless there are complications you want to investigate for, and for atypical, persistent or relapsing symptoms, or if there are alarm features like weight loss or anaemia that need looking into. Complications that are worrying include stricture formation, Barrett’s oesophagus (metaplasia) or oesophageal carcinoma. Upon stopping PPIs there is a high rate of relapse.

171
Q

Mr N, a tall and thin young student presents at A&E with unilateral left sided chest pain and shortness of breath. He is pale and tachycardic. The upper left lobe is silent on auscultation.

What is the most likely diagnosis?
A. Costochondritis
B. Herpes zoster
C. Acute coronary syndrome
D. Ectopic pregnancy
E. Fibromyalgia
F. Cyclical breast pain
G. Pleuritis
H. Panic disorder
I. Pneumothorax
J. Oesophageal reflux
K. Pulmonary embolus
L. Transient ischaemic attack
A

I. Pneumothorax

A primary pneumothorax occurs in young people without any known lung conditions. Chest pain, likely to be pleuritic in nature, SOB, pallor, tachycardia and examination findings here are consistent. Having a tall and slender build like this patient is a recognised risk factor. Other risks include smoking, FH, Marfan’s, young age, male and conditions like CF and TB. The main investigation is a CXR and pneumothoraces are classified by the BTS as large (>2cm visible rim between the lung margin and the chest wall) or small (<2cm). If the patient is clinically stable, they can be observed and given oxygen – an invasive approach is not necessary and the oxygen will increase the rate of pneumothorax reabsorption. If large however, percutaneous needle aspiration is required (IV cannula 2nd intercostal space, or 3rd, at the MCL). A CXR should be obtained after this procedure. If this fails, a chest drain should be inserted. If the CXR is normal, then you may start to consider other differentials, and indeed a CXR may even show evidence of PE if this was the case.

172
Q

A 65 year old man presents with severe, continuous burning left sided chest pain. This has lasted for the last 48hrs. The ECG of the patient is normal and 12 hour troponin tests is negative. The patient has been taking prednisolone for the last 4 months.

What is the most likely diagnosis?
A. Costochondritis
B. Herpes zoster
C. Acute coronary syndrome
D. Ectopic pregnancy
E. Fibromyalgia
F. Cyclical breast pain
G. Pleuritis
H. Panic disorder
I. Pneumothorax
J. Oesophageal reflux
K. Pulmonary embolus
L. Transient ischaemic attack
A

B. Herpes zoster

Chronic corticosteroid use is a strong risk factor here by causing immunosuppresion. Other risks include advanced age, HIV, chemotherapy and malignancies. Herpes zoster infection is caused by reactivation of primary VZV infection due to a decline in virus-specific cell-mediated immunity. It presents with a pain which is usually described by patients as burning or stabbing and is followed generally after 2-3 days (but can be weeks) by a vesicular rash in the affected dermatome. The eruption begins as an erythematous maculopapular rash which is followed by the presence of clear vesicles. The location of the pain depends on the affected nerve. The diagnosis can be made on typical clinical symptoms such as dermatomal pain and eruption of vesicles in the same area. PCR can be used to confirm the diagnosis and treatment is mainly aimed at reducing pain with painkillers and using antiviral drugs such as acyclovir to inhibit viral replication. It is worth noting that if the trigeminal nerve is affected, the rash may cause corneal ulceration and the patient presents with pain in the affected eye and reduced vision. The most commonly involved ganglia are the thoracic and trigeminal nerves.

173
Q

A 17 year old girl presents to A&E having recently (< 5 hours ago) taken a massive overdose of paracetamol in an attempt to take her own life. 2 hours after taking the tablets she was told that actually the girl she saw with her boyfriend was in fact his cousin and not a furtive romantic interest.

What is the most appropriate treatment?
A. N-acetylcysteine
B. Pralidoxime
C. Naloxone
D. Venesection
E. Protamine
F. Desferrioxamine
G. Methionine
H. Dimercaprol
I. Flumazenil
J. Ethanol
K. Vitamin K
L. Penicillamine
M. Dicobalt edetate
A

A. N-acetylcysteine

This is actually not too unreal a case. I happened to be in A&E on Valentine’s day and witnessed an overdose and self harm based around quite similar circumstances. Paracetamol OD can occur after a single large OD or repeated ODs. Often, the patient is asymptomatic at initial presentation but if untreated may cause liver injury over the 2-4 days after ingestion, including fulminant liver failure. Paracetamol is the most frequent intentional OD drug in this country. The risk of liver damage is increased after taking drugs which induce CYP 450. Inducers include St John’s wort, barbiturates, phenytoin, tetracycline, chronic alcohol use and carbamazepine. A serum paracetamol level is important to order as early as possible, but at the earliest 4 hours post-ingestion.Treatment if indicated is with N-acetylcysteine with the level based on a paracetamol treatment graph. NAC will reduce hepatic damage caused by NAPQI. There is insufficient endogenous glutathione to neutralise this in an overdose and NAC binds and neutralises NAPQI. In patients allergic to NAC, oral methionine can be used but it is not the first line choice here.

174
Q

An opioid addict self-refers to A&E complaining of very severe loin pain, and mentions a strong past history of renal stones. The patient requests opioid analgesia and the doctor administers diacetylmorphine. The patient notices the doctor has made a miscalculation by administering ten times the required dose, but is happy not to point this out. When nobody is looking the addict self-discharges from the department. He only gets as far as the observation ward before collapsing with bradypnoea and soon becoming comatose.

What is the most appropriate treatment?
A. N-acetylcysteine
B. Pralidoxime
C. Naloxone
D. Venesection
E. Protamine
F. Desferrioxamine
G. Methionine
H. Dimercaprol
I. Flumazenil
J. Ethanol
K. Vitamin K
L. Penicillamine
M. Dicobalt edetate
A

C. Naloxone

Signs of opiate OD include CNS depression, miosis (pinpoint pupils) and apnoea. Naloxone is indicated both therapeutically and diagnostically. If there is a response, then it is diagnostic. Another diagnosis should be sought if the patient is unresponsive. IV is the preferred route of administration although naloxone can be given IM or SC if IV access cannot be established. Ventilatory support is key with 100% oxygen.

175
Q

A 63 year old woman on regular dialysis falls and fractures her hip. She undergoes surgery and postoperatively receives heparin, morphine, simvastatin, calcichew and co-amoxiclav. 3 days later she is found to be very drowsy and unrousable.

What is the most appropriate treatment?
A. N-acetylcysteine
B. Pralidoxime
C. Naloxone
D. Venesection
E. Protamine
F. Desferrioxamine
G. Methionine
H. Dimercaprol
I. Flumazenil
J. Ethanol
K. Vitamin K
L. Penicillamine
M. Dicobalt edetate
A

C. Naloxone

Signs of opiate OD include CNS depression, miosis (pinpoint pupils) and apnoea. Naloxone is indicated both therapeutically and diagnostically. If there is a response, then it is diagnostic. Another diagnosis should be sought if the patient is unresponsive. IV is the preferred route of administration although naloxone can be given IM or SC if IV access cannot be established. Ventilatory support is key with 100% oxygen.

176
Q

A rural aubergine farmer is brought to A&E and is severely ill with dizziness, headache, miosis, nausea, hypersalivation, vomiting and bradycardia. The patient is unable to communicate much but mentions a spill of volatile organophosphorus insecticides, which occured about 9 hours ago. He is given atropine sulphate and also one other drug as an adjunct.

What is the most appropriate treatment?
A. N-acetylcysteine
B. Pralidoxime
C. Naloxone
D. Venesection
E. Protamine
F. Desferrioxamine
G. Methionine
H. Dimercaprol
I. Flumazenil
J. Ethanol
K. Vitamin K
L. Penicillamine
M. Dicobalt edetate
A

B. Pralidoxime

Organophosphate poisoning can occur due to occupational or accidental exposure, or if you were being attacked by Saddam Hussein’s regime. Treatment is with resuscitation and supportive care, removal of contaminated clothing, washing the skin and starting full atropinisation IV. Over treament is very much preferred to under treatment and the skin should feel dry when the patient has received adequate atropine. Pupils will also be dilated. Pralidoxime is often given as an adjunct in very severe cases but evidence does not support its routine use, unless you’ve just been attacked by a nerve agent by terrorists, which is unlikely in this agricultural worker. But, given this question tells you he has been given an adjunct, the only feasible one on the list is pralidoxime. It is not however routinely used.

177
Q

A 49 year old man of northern European origin presents to an endocrinologist with symptoms and signs of diabetes mellitus, and also complains of fatigue and arthralgia. On extensive further investigation he is noted to have cardiomyopathy, testicular atrophy and hepatomegaly. The patient also mentions that his skin seems to have darkened somewhat over the past decade, despite no increase in sun exposure.

What is the most appropriate treatment?
A. N-acetylcysteine
B. Pralidoxime
C. Naloxone
D. Venesection
E. Protamine
F. Desferrioxamine
G. Methionine
H. Dimercaprol
I. Flumazenil
J. Ethanol
K. Vitamin K
L. Penicillamine
M. Dicobalt edetate
A

D. Venesection

This is the classic case of the ‘bronzed diabetic’. No prizes for working out this man has haemochromatosis. This condition is autosomal recessive (HFE gene mutation) and is a disorder of inappropriately increased iron absorption and increased iron release (from erythrophagocytosis, whi ch is pretty much what you gather from the name). Iron gets deposited in organs – the liver, heart, adenohypophysis, pancreas, joints… leading to organ dysfunction. In the liver it can lead to cirrhosis due to periportal hepatocyte iron accumulation, it causes the myocardium to thicken and the heard may be enlarged to give cardiomyopathy and heart failure and in the pancreas it can lead to diabetes. Hypogonadism may result from involvement of the anterior pituitary. Presenting features include DM present in half at the time of diagnosis and skin bronzing (which may progress to grey or brown with slate-grey patches in the mouth). The goal of treatment is to remove excess iron. There are two options for this on the list – iron chelation with desferrioxamine or venesection. Venesection is the preferred intervention (for stage 2 and above disease i.e. late stage) with iron chelation reserved for those with a contraindication to phlebotomy (such as anaemia or impossible blood draw). Remember that vitamin C increases intestinal absorption of dietary iron (although in low doses during therapy with desferrioxamine, may increase the availability of iron for chelation).

178
Q

A house officer accidentally overdoses a patient with heparin. Realising his mistake he asks for an APTT (which comes back as significantly prolonged, and is likely to worsen).

What is the most appropriate treatment?
A. N-acetylcysteine
B. Pralidoxime
C. Naloxone
D. Venesection
E. Protamine
F. Desferrioxamine
G. Methionine
H. Dimercaprol
I. Flumazenil
J. Ethanol
K. Vitamin K
L. Penicillamine
M. Dicobalt edetate
A

E. Protamine

This is what the GMC defines as terrible medical practice. Protamine sulphate is given to those with an overdose of unfractionated heparin and is given by IV injection. Let us hope that the house officer gets the dose of protamine right, or he could really be in trouble. 7) Look back at question 4 here. This patient is also iron overloaded due to excess iron in the blood transfusion bags. This has led to testicular atrophy due to iron deposition in the anterior pituitary casuing hypogonadism. However, in this case, venesection should not be deployed as the patient is anaemic so an iron chelating agent will be used instead, in this case, desferrioxamine which has to be given IV or SC. There is an alternative drug called deferasirox which is an oral chelator and in cases such as this patient’s case, it has not shown inferiority although it has not yet been evaluated in hereditary haemochromatosis. It is worth bearing this alternative in mind because nobody likes to be stabbed with a needle every now and then and compliance will be better with oral therapy.

179
Q

An 18 year old with Sickle Cell disease (who required weekly transfusions) presented with tiredness and arthralgia. Examination revealed testicular atrophy.

What is the most appropriate treatment?
A. N-acetylcysteine
B. Pralidoxime
C. Naloxone
D. Venesection
E. Protamine
F. Desferrioxamine
G. Methionine
H. Dimercaprol
I. Flumazenil
J. Ethanol
K. Vitamin K
L. Penicillamine
M. Dicobalt edetate
A

F. Desferrioxamine

This patient has iron overload causing organ dysfunction, and so they require iron chelation.

180
Q

An 18 year old girl complains of her appearance. She is much too fat, she says. She also complained of missed periods and hairiness. On physical examination you find her to be 10kg overweight.

What is the most appropriate investigation?
A. Random blood glucose
B. T3, T4, and TSH levels
C. Dexamethasone suppression test
D. Abdominal ultrasound
E. Synacthen test
F. Urine dipstick
G. Water deprivation test
H. Liver function tests
I. Serum aldosterone
J. HbA1c
K. Dietary history
L. Serum creatinine
A

D. Abdominal ultrasound

This patient’s history makes you think PCOS. Hirsutism, irregular and infrequent periods and weight gain are all features, as are acne, scalp hair loss and infertility. Hypertension is sometimes associated with this syndrome. On examination, sweaty/oily skin may be found and acanthosis nigricans may also be seen. An ultrasound is by no means the first test to order (but is the best option from the list), as 1/4 of normal women and women with other problems like hyperprolactinaemia may also have polycystic ovaries, and they are present in 3/4 of those with PCOS. Serum androgens can be measured or the diagnosis can be made clinically.

181
Q

A 44 year old woman presents with tachycardia, atrial fibrillation, double vision and swelling above her ankles. She has lid lag on examination.

What is the most appropriate investigation?
A. Random blood glucose
B. T3, T4, and TSH levels
C. Dexamethasone suppression test
D. Abdominal ultrasound
E. Synacthen test
F. Urine dipstick
G. Water deprivation test
H. Liver function tests
I. Serum aldosterone
J. HbA1c
K. Dietary history
L. Serum creatinine
A

B. T3, T4, and TSH levels

This woman has hyperthyroidism, more specifically Graves’ disease. In countries where sufficient iodine intake is not an issue, Graves’ disease is the most common cause of hyperthyroidism. Peripheral manifestations such as ophthalmopathy, pretibial myxoedema and hyperthyroid acropachy do not occur with other causes of hyperthyroidism. Acropachy manifests as clubbing with soft tissue swelling. Pretibial myxoedema is almost always associated with ophthalmopathy. Thyroid function tests are therefore the correct match here. Have a think about what they would show in Graves’ disease.
Treatment aims to normalise thyroid function and is achieved by radioactive iodine, antithyroid medications or with surgery. They are all effective and relatively safe options. Symptomatic therapy is given with beta blockers such as propranolol.

182
Q

A 12 year old boy is brought into A&E unconscious following a collapse. He has a history of 4 weeks weight loss, polyuria and polydipsia.

What is the most appropriate investigation?
A. Random blood glucose
B. T3, T4, and TSH levels
C. Dexamethasone suppression test
D. Abdominal ultrasound
E. Synacthen test
F. Urine dipstick
G. Water deprivation test
H. Liver function tests
I. Serum aldosterone
J. HbA1c
K. Dietary history
L. Serum creatinine
A

A. Random blood glucose

This is type 1 diabetes mellitus (young person with weight loss, polyuria and polydipsia) with likely collapse due to DKA. In T1DM there is absolute insulin deficiency. In DKA there is a reduction in the net effective concentration of insulin with elevation of hormones like glucagon, cortisol and GH which are counter-regulatory. Initial treatment of DKA aims at correcting severe volume depletion with IV saline infusion at a rate of 1-1.5L for the first hour. When glucose reaches 11.1mmol, fluid should be changed to 5% dextrose to prevent hypoglycaemia. Bicarbonate therapy may be necessary in adults with pH <7 or bicarbonate level <5mmol/L but this is not the main consideration. The main issue is the severe volume depletion which needs to be rapidly corrected.

183
Q

An obese 68 year old man presents with 2 kg weight loss, polyuria and polydipsia. He collapsed yesterday and was found by his carer this morning.

What is the most appropriate investigation?
A. Random blood glucose
B. T3, T4, and TSH levels
C. Dexamethasone suppression test
D. Abdominal ultrasound
E. Synacthen test
F. Urine dipstick
G. Water deprivation test
H. Liver function tests
I. Serum aldosterone
J. HbA1c
K. Dietary history
L. Serum creatinine
A

A. Random blood glucose

This is type 2 diabetes mellitus. Obesity is a precipitating factor leading to the clinical expression of diabetes. This could well be a hyperosmolar hyperglycaemic state characterised by severe hyperglycaemia, hyperosmolarity and volume depletion, causing his collapse.

184
Q

A 34 year old man presents with insidious onset weakness and weight loss. On examination, he has hyperpigmentation of the palmar creases and postural hypotension.

What is the most appropriate investigation?
A. Random blood glucose
B. T3, T4, and TSH levels
C. Dexamethasone suppression test
D. Abdominal ultrasound
E. Synacthen test
F. Urine dipstick
G. Water deprivation test
H. Liver function tests
I. Serum aldosterone
J. HbA1c
K. Dietary history
L. Serum creatinine
A

E. Synacthen test

Hyperpigmentation in the palmar creases points towards Addison’s disease. Hyperpigmentation due to excess ACTH production can be mucosal or cutaneous and is more pronounced in the palms, knuckles and around scars. MSH is a byproduct of the production of ACTH from the cleavage of POMC. Sodium is low and potassium elevated. Vomiting is present in 75% of patients and nausea is a common finding. Additionally, postural hypotension may be present. The presence of other autoimmune diseases is a risk factor for the development of Addison’s. Diagnosis of Addison’s can be made on an ACTH stimulation test (synacthen test) whereby serum cortisol remains low despite the administration of synthetic ACTH. In an emergency, treatment should not be delayed by diagnostic testing.

185
Q

A 42 year old man has hypertension, hyperglycaemia, myopathy, thinning of skin and truncal obesity.

What is the most appropriate investigation?
A. Random blood glucose
B. T3, T4, and TSH levels
C. Dexamethasone suppression test
D. Abdominal ultrasound
E. Synacthen test
F. Urine dipstick
G. Water deprivation test
H. Liver function tests
I. Serum aldosterone
J. HbA1c
K. Dietary history
L. Serum creatinine
A

C. Dexamethasone suppression test

There is truncal obesity, hyperglycaemia, hypertension, proximal myopathy and thinning of the skin in Cushing’s due to hypercorticolism. Cushing’s disease is due to an ACTH secreting pituitary adenoma and is responsible for most cases of Cushing’s syndrome. A low dose 1mg overnight dexamethasone suppresion test can be done, or a 24 hour urinary free cortisol collection to diagnose Cushing’s syndrome. Plasma ACTH should guide further investigation. If ACTH is suppressed, the problem is likely to be with the adrenals. If it not suppressed, pituitary or ectopic disease is more likely.

186
Q

A 62 year old man presents with fatigue, breathlessness and anorexia. On examination his JVP is noted as being elevated, he has hepatomegaly and swollen ankles and bilateral basal crepitations.

What is the most likely diagnosis?
A. Stable angina
B. Myocardial infarction
C. Subacute endocarditis
D. Congestive cardiac failure
E. Right heart failure
F. Atrial fibrillation
G. Decubitus angina
H. Left ventricular failure
A

D. Congestive cardiac failure

The signs and symptoms this patient has points to CCF (congestive cardiac failure). CCF is a term used for patients who are breathless with oedema (signs of LVF and RVF). Key cardiovascular risk factors this patient may give a history of include hypertension, MI, DM and dyslipiaemia. SOB with possible orthopnoea due to the sudden increase in pre-load, indicates LV failure. The bilateral basal crepitations, heard in late expiration is consistent with pulmonary oedema and is a major Framingham criteria. Neck vein distension is also a major Framingham criteria for diagnosis. Ankle oedema and hepatomegaly is a are minor criteria for diagnosis. Other major criteria for diagnosis include S3 gallop, cardiomegaly and hepatojugular reflux. For all patients, initial investigations should include ECG, CXR, TTE and bloods including BNP levels. CXR may reveal pulmonary vascular redistribution to the upper zones, Kerley B lines, an increased CTR (cardiomegaly) and pleural effusion.

187
Q

A 55 year old man with a history of systemic hypertension presents to A&E with breathlessness on exertion and orthopnoea.

What is the most likely diagnosis?
A. Stable angina
B. Myocardial infarction
C. Subacute endocarditis
D. Congestive cardiac failure
E. Right heart failure
F. Atrial fibrillation
G. Decubitus angina
H. Left ventricular failure
A

H. Left ventricular failure

This patient has presented only with symptoms of left ventricular dysfunction. You should know the distinction between left ventricular failure and right ventricular failure. RVF leads to a backlog of blood and congestion of the systemic capillaries. This causes peripheral oedema and ascites and hepatomegaly may develop. Nocturia may be a symptom as fluid returns from the legs when the patient lies down flat. LVF causes congestion in the pulmonary circulation so the symptoms are respiratory. As seen in this patient, there is SOB on exertion and orthopnoea. This is why you can ask patients in a cardiac history how many pillows they sleep with. PND can also occur as well as ‘cardiac asthma’.

188
Q

A diabetic, 66 year old lady presents to A&E with breathlessness, sweating, nausea and vomiting. She is feeling very distressed. She has no pain. On inspection she appears pale, sweaty and grey.

What is the most likely diagnosis?
A. Stable angina
B. Myocardial infarction
C. Subacute endocarditis
D. Congestive cardiac failure
E. Right heart failure
F. Atrial fibrillation
G. Decubitus angina
H. Left ventricular failure
A

B. Myocardial infarction

This diabetic is having a silent MI without chest pain. Silent MIs are more common in the elderly and those with DM probably due to autonomic neuropathy.

Tachycardia is a common feature of MI especially anterior wall MI. Chest pain of MI is classically severe and heavy in nature, located centrally with possible radiation to the left arm or jaw and lasts for >20 minutes. SOB due to pulmonary congestion and sweating due to high sympathetic output are common symptoms. This patient also has pallor which is due to a high sympathetic output. Risk factors incorporate the standard set of cardiovascular risks such as smoking, high BP, DM, obesity and dyslipidaemia. Patients with DM are at increased risk of CAD by a variety of mechanisms which are not fully known.

If ECG shows STEMI, new LBBB or confirmed posterior MI then PCI/thrombolysis is indicated. It is worth noting that RV infarction is present in 40% of inferior infarcts so if ST elevation is seen in II, III and aVF, right sided ECG leads should be obtained. Cardiac biomarkers include CK-MB and troponin.

189
Q

A 49 year old man presents to A&E with a 2 week history of a ‘tight’ central chest pain radiating to the jaw experienced when he is lying down.

What is the most likely diagnosis?
A. Stable angina
B. Myocardial infarction
C. Subacute endocarditis
D. Congestive cardiac failure
E. Right heart failure
F. Atrial fibrillation
G. Decubitus angina
H. Left ventricular failure
A

G. Decubitus angina

Usually as a complication of heart failure. This patient has chest pain which occurs on lying down, which is decubitus angina by definition.

190
Q

A 45 year old man comes to A&E with shortness of breath, giving a history of decreased exercise tolerance. On examination the patient is noted as having an irregular pulse.

What is the most likely diagnosis?
A. Stable angina
B. Myocardial infarction
C. Subacute endocarditis
D. Congestive cardiac failure
E. Right heart failure
F. Atrial fibrillation
G. Decubitus angina
H. Left ventricular failure
A

F. Atrial fibrillation

This patient has hyperthyroidism. More specifically, Graves’ disease (peripheral manifestations such as ophthalmopathy do not occur with other causes of hyperthyroidism). Treatment of Graves’ aims to normalise thyroid function and is achieved by radioactive iodine, antithyroid medications or with surgery. They are all effective and relatively safe options. Symptomatic therapy is given with beta blockers such as propranolol. This patient has AF which has occured as a result of his hyperthyroid state which affects around 10% of untreated patients. Irregular HR is the hallmark feature of AF. Have a think about what the ECG would show.

191
Q

70 year old male smoker with a 5 year history of productive cough presents with breathlessness. On examination you find hyperinflation and diminished breath sounds when examining the chest.

What is the most likely diagnosis?
A. COPD
B. Pneumonia
C. Pulmonary embolus
D. Asthma
E. Mitral regurgitation
F. Thyrotoxicosis
G. Anxiety
H. Epiglottitis
I. Mitral stenosis
J. Left ventricular failure
K. Pneumothorax
L. Anaemia
M. Aspirin poisoning
A

A. COPD

Smoking is the most important risk factor, accounting for 90% of COPD. COPD has an insidious onset and usually presents in older people with a history of cough, wheeze and SOB. Patients with COPD are at a higher risk of infections and are vaccinated against influenza annually and pneumococcal pneumonia every 5 years. Spirometry is the gold standard for diagnosis, with FEV1/FVC ratio <70% with no evidence of reversibility (unlike asthma) being indicative. These examination findings are consistent with emphysematous hyperexpanded lungs consistent with COPD.

192
Q

40 year old female presents with 3 week history of shortness of breath. She also complains of chronic fatigue. On examination she is pale with a pulse of 120 bpm.

What is the most likely diagnosis?
A. COPD
B. Pneumonia
C. Pulmonary embolus
D. Asthma
E. Mitral regurgitation
F. Thyrotoxicosis
G. Anxiety
H. Epiglottitis
I. Mitral stenosis
J. Left ventricular failure
K. Pneumothorax
L. Anaemia
M. Aspirin poisoning
A

L. Anaemia

Anaemia is defined by haemoglobin concentration (<12 females and <14 males or there abouts – different books quote slightly different figures for normal). The most common cause and a likely cause in this patient is iron deficiency and females are affected more so than males. Anaemias are classified into microcytic, normocytic and macrocytic by MCV which guides the investigation of the cause of anaemia. Pallor is a finding on examination. There may be koilonychia, angular stomatitis and glossitis. SOB is a non-specific complaint which is not particularly common though patients often complain of fatigue (like this patient) and weird cravings. Dysphagia may also be a presentation of IDA with swallowing difficulties at the level of the cricoid consistent with Plummer-Vinson syndrome – an uncommon presentation but somewhat seen in EMQs. This patient’s anaemia will need to be investigated as the cause is unclear.

193
Q

85 year old male presents with shortness of breath and confusion. On examination there is decreased expansion on the left side and the patient resp rate is 35/min.

What is the most likely diagnosis?
A. COPD
B. Pneumonia
C. Pulmonary embolus
D. Asthma
E. Mitral regurgitation
F. Thyrotoxicosis
G. Anxiety
H. Epiglottitis
I. Mitral stenosis
J. Left ventricular failure
K. Pneumothorax
L. Anaemia
M. Aspirin poisoning
A

B. Pneumonia

Remember that older people present with atypical symptoms such as confusion, lethargy and general deterioration. There is SOB, decreased expansion and respiratory distress. This is likely to be pneumonia which is the best option on the list. The other differentials would not explain the confusion as well as the other symptoms. The most specific and sensitive test is a CXR (PA and lateral) and initial treatment of a CAP is empirical with antibiotics. Management is guided by the patient’s CURB-65 score.

194
Q

75 year old recently widowed male smoker with a history of angina presents with shortness of breath. He has also vomited and complains of a ringing in his ears. On examination the patient has a BP of 80/50 mmHg and fine crackles at both lung bases.

What is the most likely diagnosis?
A. COPD
B. Pneumonia
C. Pulmonary embolus
D. Asthma
E. Mitral regurgitation
F. Thyrotoxicosis
G. Anxiety
H. Epiglottitis
I. Mitral stenosis
J. Left ventricular failure
K. Pneumothorax
L. Anaemia
M. Aspirin poisoning
A

M. Aspirin poisoning

This patient has angina so probably has a stash of aspirin. Tinnitis is common in the early stages of acute salicylate poisoning and reflects CNS toxicity. There may also be deafness and both are reversible. GIT decontamination should be considered as an adjunct on arrival to A&E and activated charcoal can be given. The mainstay of treatment is alkaline diuresis induced by an infusion of sodium bicarbonate. In cases of severe poisoning, it is still started as a bridge to haemodialysis.

195
Q

80 year old female with a history of rheumatic fever in childhood and palpitations presents with shortness of breath. On examination she has an irregularly irregular pulse of 120bpm and loud first heart sound

What is the most likely diagnosis?
A. COPD
B. Pneumonia
C. Pulmonary embolus
D. Asthma
E. Mitral regurgitation
F. Thyrotoxicosis
G. Anxiety
H. Epiglottitis
I. Mitral stenosis
J. Left ventricular failure
K. Pneumothorax
L. Anaemia
M. Aspirin poisoning
A

I. Mitral stenosis

Practically every single case of mitral stenosis is caused by rheumatic heart disease. The major criteria for rheumatic fever can be remember by CASES: carditis, arthritis, Sydenham’s chorea, erythema marginatum and subcutaneous nodules. The process tends to also cause regurgitation. Mitral stenosis is characteristically a grade 1-2 low pitch murmur heard in mid-diastole which has a rumbling nature and there is no radiation. There can be an associated malar flush, tapping apex beat and a diastolic thrill palpable at the apex, in the 5th intercostal space in the MCL. The first heart sound is also characteristically loud and often this is the most striking feature on ascultation. It is a difficult murmur to pick up so if you are ever asked at this stage to spot this murmur, it will most likely be based on the loud S1. Mitral stenosis is associated with AF caused by LA enlargement (seen by the irregularly irregular pulse).

196
Q

A 3 year old girl presented with several bruises over her body and tiredness. On examination she appeared pale and had petechial haemorrhages. Blood tests revealed low Hb and high WCC and a thrombocytopenia. Peripheral blood film showed the presence of blast cells.

What is the most likely diagnosis?
A. Acute lymphoblastic leukaemia
B. Primary polycythaemia
C. Disseminated intravascular coagulation
D. Pernicious anaemia
E. Haemophilia B
F. Aplastic anaemia
G. Haemophilia A
H. Iron deficiency anaemia
I. Chronic myeloid leukaemia
J. Crohn's disease
K. Chronic lymphocytic leukaemia
A

A. Acute lymphoblastic leukaemia

ALL typically presents in children with bone marrow involvement and the associated symptoms. It is primarily a disease of children with 75% of all cases occuring in those under the age of 6. T-cell lineage ALL may cause mediastinal masses and associated symptoms such as stridor, wheeze and SVCO. Bone marrow infiltration leads to a pancytopenia leading to anaemia (reduced red blood cells), haemorrhage (reduced platelets) and infections (reduced mature white blood cells). In ALL, bone marrow is replaced by lymphoblasts. There may also be spread to CNS and testes. ALL is associated with Down’s syndrome. In contrast, CLL presents in older adults and is often asymptomatic, discovered by chance when a FBC is ordered. Smear/smudge cells are seen in peripheral blood smear. CLL is associated with a warm-type AIHA and there is peripheral blood lymphocytosis.

197
Q

A 68 year old lady presenting with tiredness, vertigo, tinnitus and bleeding. Further history revealed itching, especially after a hot bath and a PMH of angina. On examination she appeared plethoric with a BP of 170/110 and splenomegaly. Blood tests revealed increases in all of the following: Hb, PCV, red cell mass, WCC and platelets. PaO2 was normal.

What is the most likely diagnosis?
A. Acute lymphoblastic leukaemia
B. Primary polycythaemia
C. Disseminated intravascular coagulation
D. Pernicious anaemia
E. Haemophilia B
F. Aplastic anaemia
G. Haemophilia A
H. Iron deficiency anaemia
I. Chronic myeloid leukaemia
J. Crohn's disease
K. Chronic lymphocytic leukaemia
A

B. Primary polycythaemia

Polycythaemia rubra vera is a disease of middle and older age and is strongly associated with the JAK2V617 mutation. Pruritis is a common feature and is often severe and evoked by contact with water. Presentation can be with features of thrombosis and/or features of haemorrhage. Facial redness and fullness is commonly observed and splenomegaly is a common finding. It is a myeloproliferative disorder with raised Hct(PCV), Hb and RBC count. Blood hence becomes very viscous. There is a clear link between Budd-Chiari syndrome and subsequent PRV. Treatment is with venesection. Around 30% will go on to develop myelofibrosis. Presentation with all of the classic symptoms is uncommon but this is an EMQ after all. Familial cases of PRV exist but are rare.

198
Q

A 28 year old woman who has suffered a miscarriage presented with a nosebleed and large bruises. She is acutely ill and shocked. Her blood tests show very prolonged APTT and PT. Fibrinogen is decreased and she has a severe thrombocytopenia.

What is the most likely diagnosis?
A. Acute lymphoblastic leukaemia
B. Primary polycythaemia
C. Disseminated intravascular coagulation
D. Pernicious anaemia
E. Haemophilia B
F. Aplastic anaemia
G. Haemophilia A
H. Iron deficiency anaemia
I. Chronic myeloid leukaemia
J. Crohn's disease
K. Chronic lymphocytic leukaemia
A

C. Disseminated intravascular coagulation

DIC is a syndrome where coagulation pathways activate resulting in intravascular thrombosis, platelet and clotting factor depletion. The underlying disorder needs to be treated and FFP with platelet concentrate needs to be given. The cause here is obstetric and could for example be due to placenta abruptio. Release or exposure of tissue material into circulation leads to activation of the external coagulation pathway (involving factor VIIa). Generalised bleeding evidence by at least 3 unrelated sites is highly suggestive of DIC. Here there is bruising and nosebleeds. There may be systemic signs of circulatory collapse (this woman is in shock and there may also be oliguria and tachycardia) and of micro and macrovascular thrombosis (such as purpura fulminans and gangrene). A platelet infusion should be considered and FFP is preferred for replacement of clotting factors and clotting inhibitors. Complications of DIC include life-threatening bleed, ARF and gangrene.

199
Q

A 6 year old boy presenting with bruising and painful swollen knees. Clotting screen showed a high APTT and low factor 9. FBC, PT and vWF levels were normal.

What is the most likely diagnosis?
A. Acute lymphoblastic leukaemia
B. Primary polycythaemia
C. Disseminated intravascular coagulation
D. Pernicious anaemia
E. Haemophilia B
F. Aplastic anaemia
G. Haemophilia A
H. Iron deficiency anaemia
I. Chronic myeloid leukaemia
J. Crohn's disease
K. Chronic lymphocytic leukaemia
A

E. Haemophilia B

Haemophilia is a bleeding disorder which is usually inherited and is characterised by the deficiency of clotting factor VIII (in haemophilia A) or factor IX (in haemophilia B). It occurs almost exclusively in men due to an X-linked pattern of inheritance and can be mild, moderate or severe based on the level of clotting factor. The most common type of haemorrhage is musculoskeletal and treatment aims at replacement of the deficient clotting factor. A complication of treatment which is a major issue is the development of inhibitory antibodies against the infused factor VIII or IX. APTT will be usually prolonged and a factor VIII and/or factor IX assay should be requested to confirm the diagnosis. Here it shows a low factor 9 which gives the diagnosis.

200
Q

A 30 year old afro-Caribbean patient comes to A&E with a rash on his cheeks, nose and his shins. The CXR shows bilateral hilar lymphadenopathy.

What is the most likely diagnosis?
A. Sarcoidosis
B. Streptococcal pneumoniae
C. Bronchiectasis
D. Bronchial carcinoma
E. Trauma
F. Pulmonary TB
G. PE
H. COPD
I. Mycoplasma pneumoniae
J. Primary pulmonary hypertension
K. LVF
A

A. Sarcoidosis

Sarcoidosis is a chronic multisystem disease with an unknown aetiology. The lesions on his shins are erythema nodosum and are tender erythematous nodules. Lupus pernio is another typical skin manifestation of sarcoidosis presenting with indurated plaques with discoloration on the face which this person unluckily also has. CXR will typically show bilateral hilar lymphadenopathy and CXR findings are used in the staging of disease. Additionally, serum calcium and ACE levels may be raised. A transbronchial biopsy is essential for diagnosis in most cases and shows the presence of non-caseating granulomas. Black people have a higher lifetime risk of sarcoidosis, as do those of Scandinavian origin. The mainstay of treatment for severe disease involves systemic corticosteroids.

201
Q

A 49 year old lady comes to A&E with severe haemoptysis. She has a history of continuous production of foul smelling khaki coloured sputum and she had a past medical history of whooping cough as a child.

What is the most likely diagnosis?
A. Sarcoidosis
B. Streptococcal pneumoniae
C. Bronchiectasis
D. Bronchial carcinoma
E. Trauma
F. Pulmonary TB
G. PE
H. COPD
I. Mycoplasma pneumoniae
J. Primary pulmonary hypertension
K. LVF
A

C. Bronchiectasis

Sarcoidosis is a chronic multisystem disease with an unknown aetiology. The lesions on his shins are erythema nodosum and are tender erythematous nodules. Lupus pernio is another typical skin manifestation of sarcoidosis presenting with indurated plaques with discoloration on the face which this person unluckily also has. CXR will typically show bilateral hilar lymphadenopathy and CXR findings are used in the staging of disease. Additionally, serum calcium and ACE levels may be raised. A transbronchial biopsy is essential for diagnosis in most cases and shows the presence of non-caseating granulomas. Black people have a higher lifetime risk of sarcoidosis, as do those of Scandinavian origin. The mainstay of treatment for severe disease involves systemic corticosteroids.

202
Q

A male 45 year old patient from India presented with a 2 week history of tiredness, loss of weight and haemoptysis. The left lung base was stony dull on percussion.

What is the most likely diagnosis?
A. Sarcoidosis
B. Streptococcal pneumoniae
C. Bronchiectasis
D. Bronchial carcinoma
E. Trauma
F. Pulmonary TB
G. PE
H. COPD
I. Mycoplasma pneumoniae
J. Primary pulmonary hypertension
K. LVF
A

F. Pulmonary TB

It is important to have a high level of suspicion when evaluating patients with risk factors who present with suggestive symptoms. Night sweats, fever, weight loss, malaise, cough, haemoptysis and erythema nodosum are all suggestive. In the first half of the 20th century, tuberculosis accounted for over 90% of cases of erythema nodosum. Other key risk factors for pulmonary TB include exposure to infection and returning from or being born in a high-risk region such as Asia, Africa and Latin America (India in this case). The examination findings here are consistent with a parapneumonic effusion. If TB is suspected, the patient should be placed in isolation and a CXR obtained with 3 sputum samples cultured for AFB being the gold standard of diagnosis. Culture takes several weeks so sputum smears will be done before culture results are known. Interferon-gamma release assays (IGRAs) are now used by some hospitals to rapidly determine a patient’s TB status. All patients who have TB should be tested for HIV within 2 months of diagnosis.

203
Q

60 year old patient presents with shortness of breath and haemoptysis. On examination the patient is tachycardic and tachypnoeic and has swollen ankles. On listening to the chest he has basal crepitations.

What is the most likely diagnosis?
A. Sarcoidosis
B. Streptococcal pneumoniae
C. Bronchiectasis
D. Bronchial carcinoma
E. Trauma
F. Pulmonary TB
G. PE
H. COPD
I. Mycoplasma pneumoniae
J. Primary pulmonary hypertension
K. LVF
A

K. LVF

LVF causes congestion in the pulmonary circulation so the symptoms are respiratory with evidence here of pulmonary oedema. As seen in this patient, there is SOB and there may also be orthopnoea. This is why you can ask patients in a cardiac history how many pillows they sleep with. PND can also occur as well as ‘cardiac asthma’. RVF leads to a backlog of blood and congestion of the systemic capillaries. This causes peripheral oedema and ascites and hepatomegaly may develop. Nocturia may be a symptom as fluid returns from the legs when the patient lies down flat. This patient does have peripheral oedema too so technically has CCF (congestive cardiac failure), but the best option on the list is LVF.

204
Q

A 69 year old lady presents with a sudden onset of fever, productive cough of purulent rusty coloured sputum. Examination of the chest showed signs of consolidation.

What is the most likely diagnosis?
A. Sarcoidosis
B. Streptococcal pneumoniae
C. Bronchiectasis
D. Bronchial carcinoma
E. Trauma
F. Pulmonary TB
G. PE
H. COPD
I. Mycoplasma pneumoniae
J. Primary pulmonary hypertension
K. LVF
A

B. Streptococcal pneumoniae

The rusty coloured sputum is hinting at a pneumococcal pneumonia.The patient has presented with common symptoms of fever and a productive cough. Examination findings are also consistent – have a think about what would actually be found while performing a respiratory examination on this patient. There may also be SOB, chills, rigors and pleuritic chest pain. The most specific and sensitive test is a CXR (PA and lateral) and initial treatment of a CAP is empirical with antibiotics. Often diagnosis is made solely on history and examination findings. Management is guided by the patient’s CURB-65 score.

205
Q

A 45 year old diabetic man collapses while painting his house after taking some ‘Rennies’ for painful indigestion and is brought into A&E sweating and in distress. His wife had to stop the car en route for him to vomit. Examination shows him sweating and tachycardic but is otherwise unremarkable.

What is the most likely diagnosis?
A. Meningitis
B. Appendicitis
C. Subarachnoid haemorrhage
D. Large bowel obstruction
E. Migraine
F. Myocardial infarction
G. Small bowel obstruction
H. Tension headache
I. Pancreatitis
J. Labyrinthitis
K. Gastroenteritis
L. Gastric ulcer
A

F. Myocardial infarction

This diabetic is having a silent MI without chest pain. All of the other symptoms are indicative such as vomiting, sweating and tachycardia. Silent MIs are more common in the elderly and those with DM probably due to autonomic neuropathy. If ECG shows STEMI, new LBBB or confirmed posterior MI then PCI/thrombolysis is indicated. It is worth noting that RV infarction is present in 40% of inferior infarcts so if ST elevation is seen in II, III and aVF, right sided ECG leads should be obtained. Cardiac biomarkers include CK-MB and troponin. Troponins rise 4-6 hrs after onset of infarction and peak at 18-24 hours and may persist for 7-10 days.

206
Q

A student teacher presents on your take after school. She has noticed that she is drowsy and irritable. She has a throbbing headache that is unilateral and lasts for a few hours. The headache is associated with some visual disturbance and she reports seeing zigzag lines. She felt nauseous and has vomited. On examination her pulse is slow but regular and her blood pressure is increased.

What is the most likely diagnosis?
A. Meningitis
B. Appendicitis
C. Subarachnoid haemorrhage
D. Large bowel obstruction
E. Migraine
F. Myocardial infarction
G. Small bowel obstruction
H. Tension headache
I. Pancreatitis
J. Labyrinthitis
K. Gastroenteritis
L. Gastric ulcer
A

E. Migraine

Migraine is a chronic condition, with genetic determinants, which usually presents in early to mid life. The typical migraine aura this patient describes (which can be visual, sensory or speech symptoms) which can occur during or before the headache, is pathognomic, but is not seen in the majority of patients. The aura can be positive phenomena (for example seeing flashing lights) or negative phenomena (for example visual loss). Nausea, photophobia and disability (the headache gets in the way with the patient’s ability to function) accompanying a headache suggest a migraine diagnosis. The headache of a migraine tends to be prolonged if untreated, and tends to be unilateral and pounding (but does not have to be). Tests aim to rule out other differentials, although if the history is compatible and neurological examination is unremarkable, further testing is not needed. Treatment of this chronic condition aims at treating acute attacks to restore function. Triptans can be used in specialist care. These are 5HT1 agonists. Effective initial treatment in a primary care setting can involve NSAIDs, which are available OTC. Treatment should be taken as soon as a patient realises they are having an attack and may need to be repeated after the attack. A few patients who have frequent, severe or disabling headaches may require daily prophylaxis such as anticonvulsants, TCAs and beta blockers.

207
Q

A widowed 55 year old man complaining of a severe pain in his abdomen is admitted while drunk. He finds the pain eases when he sits forward as it radiates to his back. He relates to you that he has been vomitting heavily. On examination he is pyrexial and tachycardic, with a rigid abdomen.

What is the most likely diagnosis?
A. Meningitis
B. Appendicitis
C. Subarachnoid haemorrhage
D. Large bowel obstruction
E. Migraine
F. Myocardial infarction
G. Small bowel obstruction
H. Tension headache
I. Pancreatitis
J. Labyrinthitis
K. Gastroenteritis
L. Gastric ulcer
A

I. Pancreatitis

This patient has pancreatitis. He has vomited heavily and is describing mid-epigastric pain radiating around to the back. This description of the pain itself is almost diagnostic. This pain is classically relieved if the patient curls up and is worse with movement. Complicated haemorrhagic pancreatitis may exhibit Cullen’s sign, Grey-Turner’s sign and Fox’s sign. Make sure you know what these are and you are familiar with the causes of acute pancreatitis – it seems likely to be alcohol in this instance. Those caused by hypocalcaemia may display Chvostek’s sign and Trousseau’s sign. Key to diagnosis is serum amylase or lipase levels which are massively elevated. Prognostic criteria are outlined in Ranson’s criteria applied on admission and after 48 hours, or the modified Glasgow score which you can find in your Oxford Handbook. An abdominal CT is however the most sensitive and specific study and findings may include enlargement of the pancreas with irregular contours, necrosis, pseudocysts and peripancreatic fat obliteration.

208
Q

A 25 year old man gives a history of loss of appetite and nausea over 2 days with profuse vomiting. He also has colicky abdominal pain with increased bowel sounds. Past medical history reveals that he had an exploratory laporotomy 2 years ago.

What is the most likely diagnosis?
A. Meningitis
B. Appendicitis
C. Subarachnoid haemorrhage
D. Large bowel obstruction
E. Migraine
F. Myocardial infarction
G. Small bowel obstruction
H. Tension headache
I. Pancreatitis
J. Labyrinthitis
K. Gastroenteritis
L. Gastric ulcer
A

G. Small bowel obstruction

Bowel obstruction has several causes such as adhesions or cancer in the older patient. In this case the PMH of recent surgery suggests adhesions as a likely aetiology. The profuse vomiting, distended and colicky painful abdomen and increased tinkling high pitched bowel signs are all indicative. The proximal segment of bowel dilates and distal bowel collapses. Completely obstructed patients generally require surgery. If, on AXR, air is seen to be seeping past the obstruction then the obstruction is partial. As a standard, all patients should be made NBM and given supplemental oxygen, IV fluids and NG decompression (to reduce flow/gastric contents/air towards the obstruction), unless they are rushed off for an emergency laparotomy because, for example, they have complete SBO and are peritonitic.

209
Q

A patient who is unsteady on their feet reports to you a history over 3 days of 20 minute unsettling spells of nausea and vomiting where their unsteadiness increases and they feel they are veering sideways. On examination they are pale and sweaty but little else is remarkable.

What is the most likely diagnosis?
A. Meningitis
B. Appendicitis
C. Subarachnoid haemorrhage
D. Large bowel obstruction
E. Migraine
F. Myocardial infarction
G. Small bowel obstruction
H. Tension headache
I. Pancreatitis
J. Labyrinthitis
K. Gastroenteritis
L. Gastric ulcer
A

J. Labyrinthitis

Labrynthitis is an inflammatory condition affecting the labyrinth in the cochlea and the vestibular system of the inner ear. The most common cause is viral in origin although bacterial laryrinthitis occurs as a complication of otitis media or meningitis. The typical presentation is one including vertigo, imbalance and hearing loss. The typical description is of acute rotational vertigo which may last up to 7 hours. This is often associated with N&V. Risk factors include recent URTI or other viral infections such as VZV, CMV, mymps, measles, rubella and HIV. Treatment is typically symptomatic and involves the use of vestibular suppresants (benzodiazepines such as diazepam) and antiemetics (such as promethazine or metaclopramide). Labyrinthitis is the only condition on the list which would cause this constellation of symptoms this patient is describing. In reality, this presentation could of course be vestibular neuritis.

210
Q

A 70 year old man presents with suprapubic pain. He complains that he has not passed any urine in 3 days. On examination he has a distended bladder.

What is the most likely diagnosis?
A. Colorectal cancer
B. Urinary tract stones: bladder outflow obstruction
C. UTI
D. Crohn’s disease
E. Ovarian cyst
F. Ulcerative colitis
G. Pseudomembranous colitis
H. Endometritis
I. Bacterial gastroenteritis
J. Bladder cancer
K. Prostatic hypertrophy
L. Urinary tract stones: ureteric colic/stricture
M. Endometriosis
N. Labour
A

K. Prostatic hypertrophy

This is benign prostatic hyperplasia. There is hyperplasia of smooth muscle and prostate enlargement. BPH presents with both symptoms of frequency, urgency and nocturia as well as problems voiding (poor stream, terminal dribbling, straining). It is the voiding symptoms present in this man (voiding problems increase with age) who has urinary retention (an acute complication) associated with bladder pain and distension. The first thing to do here is to catheterise this man to relieve the urinary retention. Medical therapy can be initiated with an alpha blocker and 5-alpha-reductase inhibitors and hopefully this man will not need further intervention

211
Q

A 45 year old smoker presents with painful haematuria. He has a history of recurrent UTI. He explains to the medical student that although he is now a taxi driver, he worked in the rubber industry for 20 years.

What is the most likely diagnosis?
A. Colorectal cancer
B. Urinary tract stones: bladder outflow obstruction
C. UTI
D. Crohn’s disease
E. Ovarian cyst
F. Ulcerative colitis
G. Pseudomembranous colitis
H. Endometritis
I. Bacterial gastroenteritis
J. Bladder cancer
K. Prostatic hypertrophy
L. Urinary tract stones: ureteric colic/stricture
M. Endometriosis
N. Labour
A

J. Bladder cancer

Gross haematuria is the primary symptom of bladder cancer. Over 80% present with haematuria which is the primary presenting complaint in this condition. The gross painless haematuria is classically painless and present throughout the entire urinary stream, however while painful the rest of the symptoms fit. It is also worth noting that carcinoma in situ commonly presents with dysuria and frequency and can easily be confused with prostatitis. Risk factors include smoking, exposure to carcinogens such as the aromatic amines used in rubber and dye industries, age >55, pelvic radiation and Schistosomiasis resulting in SCC (related to chronic inflammation – so other risks also include UTI, stones etc). Bladder cancer is the most common cancer in Egypt, for the latter reason. Cystoscopy and urinary cytology are key in diagnosis. Low grade tumours are papillary and easy to see on cystoscopy whereas high grade tumours and carcinoma in situ are often difficult to visualise. Resection provides diagnosis and primary treatment in one step.

212
Q

A 55 year old lady presents with bloody diarrhoea and intense abdominal pain. She has just recently recovered from a chest infection.

What is the most likely diagnosis?
A. Colorectal cancer
B. Urinary tract stones: bladder outflow obstruction
C. UTI
D. Crohn’s disease
E. Ovarian cyst
F. Ulcerative colitis
G. Pseudomembranous colitis
H. Endometritis
I. Bacterial gastroenteritis
J. Bladder cancer
K. Prostatic hypertrophy
L. Urinary tract stones: ureteric colic/stricture
M. Endometriosis
N. Labour
A

G. Pseudomembranous colitis

Gross haematuria is the primary symptom of bladder cancer. Over 80% present with haematuria which is the primary presenting complaint in this condition. The gross painless haematuria is classically painless and present throughout the entire urinary stream, however while painful the rest of the symptoms fit. It is also worth noting that carcinoma in situ commonly presents with dysuria and frequency and can easily be confused with prostatitis. Risk factors include smoking, exposure to carcinogens such as the aromatic amines used in rubber and dye industries, age >55, pelvic radiation and Schistosomiasis resulting in SCC (related to chronic inflammation – so other risks also include UTI, stones etc). Bladder cancer is the most common cancer in Egypt, for the latter reason. Cystoscopy and urinary cytology are key in diagnosis. Low grade tumours are papillary and easy to see on cystoscopy whereas high grade tumours and carcinoma in situ are often difficult to visualise. Resection provides diagnosis and primary treatment in one step.

213
Q

A 50 year old lady presents to A&E in excruciating pain. It radiates from the flank to the illiac fossa and labium. She cannot lie still, and is pale, sweaty and vomiting. After a few hours the pain has subsided.

What is the most likely diagnosis?
A. Colorectal cancer
B. Urinary tract stones: bladder outflow obstruction
C. UTI
D. Crohn’s disease
E. Ovarian cyst
F. Ulcerative colitis
G. Pseudomembranous colitis
H. Endometritis
I. Bacterial gastroenteritis
J. Bladder cancer
K. Prostatic hypertrophy
M. Endometriosis
N. Labour
A

L. Urinary tract stones: ureteric colic/stricture

This patient has renal colic which classically presents with severe flank pain radiating to the groin. Microscopic haematuria is present in up to 90% of cases. Up to 85% of stones are visible on a plain KUB although urate stones are radiolucent. If the stone is radio-opaque, calcification will be seen within the urinary tract. In pregnancy, a renal USS is first line. The IVP has now been replaced by the CT scan which is the new diagnostic standard otherwise. A non-contrast helical (or spiral) CT is preferred due to high sensitivity and specificity and acurately determines presence, site and size of stones. Stones are analysed after they are extracted or when they are expelled to check their composition. It is worth noting that in all females of child bearing age, a urine pregnancy test is necessary to exclude an ectopic pregnancy.

214
Q

An elderly lady is found collapsed and confused at home by her district nurse. She had been fit and well until 3 days ago when she started behaving oddly and yesterday her neighbour found her wandering around in the road in her dressing gown. On examination, she has a low grade fever, a tender abdomen and an unpleasant smell.

What is the most likely diagnosis?
A. Hepatic encephalopathy
B. Alcohol withdrawal
C. Phenytoin toxicity
D. Diabetic ketoacidosis
E. Subdural haematoma
F. Ecstasy overdose
G. Dementia
H. Meningitis
I. Schizophrenia
J. UTI
A

J. UTI

10% of women >70 have a UTI. This is the cause of this elderly lady’s confusion. Her set of symptoms (confusion, fever, abdominal tenderness) are not explained by any of the other conditions given on the list. A UTI, in uncomplicated cases is most commonly caused by E coli. It is diagnosed with a dipstick and urine MC+S from an MSU sample. Have a think about what the dipstick would show. Antibiotic therapy should be guided by local sensitivities and guidelines, or MC+S results. Nitrofurantoin or co-trimoxazole could be used.

215
Q

A 62 year old hotel owner is in hospital for investigation of his jaundice. When you come to see him in the morning he is slurring his speech and doesn’t seem to know where he is. You cant help noticing that his abdomen is grossly distended and his breath smells strangely sweet.

What is the most likely diagnosis?
A. Hepatic encephalopathy
B. Alcohol withdrawal
C. Phenytoin toxicity
D. Diabetic ketoacidosis
E. Subdural haematoma
F. Ecstasy overdose
G. Dementia
H. Meningitis
I. Schizophrenia
J. UTI
A

A. Hepatic encephalopathy

This patient has decompensated chronic liver disease (he is in liver failure) which has resulted in neurological symptoms associated with hepatic encephalopathy. The brain is exposed to ammonia which bypasses the liver by portosystemic shunting. It is a diagnosis of exclusion and tests will need to be conducted to rule out other potential causes of confusion. The findings of jaundice, ascites and fetor hepaticus (liver failure) are all signs of liver disease. Think about the other signs you might see like spider naevi and palmar erythema. This patient may also have asterixis which is a coarse flapping tremor. HE is likely caused by a host of factors. This patient’s LFTs will be abnormal and he is likely to have coagulopathy too (PT will be elevated).

216
Q

A 21 year old diabetic student is brought into A&E by his friends. They say he has been acting weirdly all night and wondered if he was on drugs. A couple of hours ago he developed a fever and started vomiting. When you meet him he seems very irritable and is complaining that the lights are too bright. Examination reveals a erythematous rash over his back.

What is the most likely diagnosis?
A. Hepatic encephalopathy
B. Alcohol withdrawal
C. Phenytoin toxicity
D. Diabetic ketoacidosis
E. Subdural haematoma
F. Ecstasy overdose
G. Dementia
H. Meningitis
I. Schizophrenia
J. UTI
A

H. Meningitis

This patient has meningitis. Universities are common sites of outbreaks due to crowding. Commonly there will be a headache, fever and nuchal rigidity. There may also be an altered mental status, confusion, photophobia and vomiting. Kernig’s sign is uncommon but is positive when attempts to extend the leg are met with resistance when the patient is supine with the thigh flexed to 90 degrees. Another uncommon sign is Brudzinski’s sign and a petechial/purpuric rash, typically associated with meningococcal meningitis. CT head should be considered before LP if there is any evidence of raised ICP. An LP will confirm the diagnosis with bacterial meningitis showing a low CSF glucose, elevated CSF protein and positive CSF culture/gram stain or meningococcal antigen.

217
Q

You are called to see a 45 year old man who is known to have suffered a subarachnoid haemorrhage two months ago. In hospital he was prescribed an anti-convulsant to reduce the risk of seizure and has continued to take it, despite being discharged a fortnight ago. Over the past week he has become increasingly confused, lethargic and ataxic. On examination, he has nystagmus and an intention tremor and shows past-pointing.

What is the most likely diagnosis?
A. Hepatic encephalopathy
B. Alcohol withdrawal
C. Phenytoin toxicity
D. Diabetic ketoacidosis
E. Subdural haematoma
F. Ecstasy overdose
G. Dementia
H. Meningitis
I. Schizophrenia
J. UTI
A

C. Phenytoin toxicity

PHT has an unpredictable pharmacokinetic behaviour. It is 80-90% bound to albumin (competitive binding by drugs such as salicylates increase free PHT). It is metabolised by hepatic mixed function oxidase and metabolism can be either induced or inhibited by drugs which share the same hepatic enzymes. It has a narrow therapeutic range of around 40-100 micromol/l and there is a lot of individual variation in the plasma concentration achieved with a fixed dose and unwanted effects tend to occur >100 micromol/l. The symptoms include those seen like nystagmus, confusion, headache, ataxia and vertigo. Chronic use is also associated with unwanted effects like gum hyperplasia and use is associated with fetal malformations like cleft palate (associated with epoxide formation in metabolism).

218
Q

A 37 year old man has had half his ear bitten off in a fight and is admitted under the plastic surgeons. After being on the ward for a day and a half he becomes extremely agitated, claiming to see snakes and spiders crawling up the walls. Examination shows him to be tachycardic and sweaty but is otherwise unremarkable.

What is the most likely diagnosis?
A. Hepatic encephalopathy
B. Alcohol withdrawal
C. Phenytoin toxicity
D. Diabetic ketoacidosis
E. Subdural haematoma
F. Ecstasy overdose
G. Dementia
H. Meningitis
I. Schizophrenia
J. UTI
A

B. Alcohol withdrawal

This is a case of alcohol withdrawal experienced by some 40% of alcohol abusers who subsequently come off alcohol. Scary auditory and visual hallucinations can occur along with the symptoms described including tachycardia, anxiety, sweating, tremor, nausea, retching and a mild pyrexia. Acutely, this should be treated with a BDZ such as chlordiazepoxide. It can progress to delirium tremens with seizures, hallucinations, coma and death.

219
Q

A 52 year old obese gentleman presents complaining of headaches in the morning and feeling very sleepy during the day which has been affecting his work. His colleagues have noticed he’s becoming increasingly irritable.

What is the most likely diagnosis?
A. Alcoholic liver disease
B. Chronic myeloid leukaemia
C. Primary biliary cirrhosis
D. Chronic lymphocytic leukaemia
E. Food poisoning
F. Systemic lupus erythematosus
G. Rheumatoid arthritis
H. Obstructive sleeps apnoea
A

H. Obstructive sleeps apnoea

This is obstructive sleep apnoea and is associated with obesity. Symptoms this patient has are typical. There may also be gasping and snoring during sleep, unrefreshing sleep and excessive sleepiness during the day, which explains why this man is feel sleepy or falling asleep at work. Diagnosis is confirmed with polysomnography (also known as a sleep study). Non-invasive interventions include positive airway pressure can be used and there is the option of surgical treatment if indicated. There are many complications of untreated OSA such as MI, stroke and premature death.

220
Q

A 45 year old lady goes to see her GP complaining of tiredness that has been going on for the past month or so. She’s noticed she has been feeling hotter and sweatier than usual, and on further questioning tells her GP that she’s lost some weight recently. On examination she has a high temperature and splenomegaly.

What is the most likely diagnosis?
A. Alcoholic liver disease
B. Chronic myeloid leukaemia
C. Primary biliary cirrhosis
D. Chronic lymphocytic leukaemia
E. Food poisoning
F. Systemic lupus erythematosus
G. Rheumatoid arthritis
H. Obstructive sleeps apnoea
A

B. Chronic myeloid leukaemia

This is CML which tends to present in the 30-60 age group. At presentation 1/3 may be asymptomatic though if symptomatic, it presents with symptoms including fever, weight loss and night sweats. There is myeloid stem cell proliferation and presents with raised neutrophils, metamyelocytes and basophils. CML is associated with the philadelphia chromosome characterised by t(9;22) of bcr-abl. There tends to be massive splenomegaly which is the most common physical finding on examination. This conditon may transform to AML or ALL in what is known as a ‘blast crisis’. CML responds to imatinib, which is an anti-bcr-abl antibody and gives long term remission in most patients. The Philadelphia chromosome is the pathognomic feature, which is a reciprocal translocation between chromosomes 9 and 22 which results in an abnormal chromosome 22. The BCR gene on 22 is fused with the ABL gene from 9 resulting in the BCR-ABL fusion oncogene. BCR-ABL is an active tyrosine kinase which phosphorylates and alters the activity of downstream signal transduction proteins. Treatment is hence with a tyrosine kinase inhibitor such as imatinib which inhibits the BCR-ABL tyrosine kinase.

221
Q

A 50 year old lady presents to A&E with a 2 day history of abdominal pain, bloody diarrhoea and fever. She was previously well and in fact attended her daughter’s wedding 5 days previously.

What is the most likely diagnosis?
A. Alcoholic liver disease
B. Chronic myeloid leukaemia
C. Primary biliary cirrhosis
D. Chronic lymphocytic leukaemia
E. Food poisoning
F. Systemic lupus erythematosus
G. Rheumatoid arthritis
H. Obstructive sleeps apnoea
A

E. Food poisoning

This is infectious gastroenteritis which is self-limiting and a definitive diagnosis, if needed, is based on isolating the organism from a stool culture. This is based on the short history in a previously well patient. Treatment is supportive with fluid and electrolyte replacement and antibiotics are generally used only for patients with risk factors for severe disease or those with extra-GI complications.

222
Q

A 35 year old African lady has been feeling unwell generally and depressed. Recently she had a fever, a rash on her face that looks like a butterfly shape, and has been experiencing some pain in her joints.

What is the most likely diagnosis?
A. Alcoholic liver disease
B. Chronic myeloid leukaemia
C. Primary biliary cirrhosis
D. Chronic lymphocytic leukaemia
E. Food poisoning
F. Systemic lupus erythematosus
G. Rheumatoid arthritis
H. Obstructive sleeps apnoea
A

F. Systemic lupus erythematosus

Systemic lupus erythematosus is a multi-system generalised disorder most commonly affecting women of reproductive age. It is characterised by the presence of ANA and most frequently involves the skin and joints. Most present with tiredness, the typical rash as seen here and/or musculoskeletal symptoms. Arthralgia is common and can be similar to RA but is non-erosive. The classic rash of lupus is the malar (butterfly) rash which presents as erythema over the cheeks and bridge of the nose, sparing the nasolabial folds. A photosensitive rash is classic, which occurs after sun exposure and there may also be a discoid rash which presents as erythematous raised patches with keratotic scaling. Symptoms and signs may accumulate over time.

223
Q

A 48 year old homeless man comes into A&E with abdominal distension and jaundice. He is disoriented and drowsy.

What is the most likely diagnosis?
A. Alcoholic liver disease
B. Chronic myeloid leukaemia
C. Primary biliary cirrhosis
D. Chronic lymphocytic leukaemia
E. Food poisoning
F. Systemic lupus erythematosus
G. Rheumatoid arthritis
H. Obstructive sleeps apnoea
A

A. Alcoholic liver disease

This man has hepatic encephalopathy which is a syndrome caused by either acute or chronic hepatic insufficiency. There is altered mental state here and the cause is multi-factorial. There is brain exposure to ammonia that has bypassed the liver by portosystemic shunting. This patient has evidence of hepatic dysfunction with ascites and jaundice implying decompensated cirrhosis, likely to be alcoholic in cause. Treatment is supportive with the aim to correct any precipitating factors.